You are on page 1of 187

Temel Matematik

1
PDF processed with CutePDF evaluation edition www.CutePDF.com
Saylar zerine
Dnyada uygarlklarn en eski gnlerinden beri nesnelerin saylar nem kazanmtr.
Toplanan rnler, alnacak vergiler, kiilerin serveti hep saylmas gereken byklkler
olmutur. Saylar, eski Mezpotamya uygarlklarnda gelitirilmi fakat bu bilgiler gnmze
dolayl yollarda ulaabilmilerdir.
Mezopotamya uygarlklarnn devam olarak kabul edilen Msrda, saylarn
uygulanmas ile ilgili kstl bilgiler bulunmaktadr. Yeni bulunan papirsler, eski Msr
uygarlnda sanldndan daha kapsaml bir matematik kltr olduunu ortaya karmtr.
zellikle, ok yeni bulunan bir papirs, adeta temel matematiin balagc saylabilir.
Uygarln Msrdan sonra devam olan eski Yunan matematikilerinin Msrda renim grm
olmalar ancak yeni olarak zerinde durulmakta olan bir gerektir. Yine de, gerek eski Msr,
gerek eski Yunan, gerekse eski Roma uygarlnda sadece yaz kaytlar yaplmtr. rnek
olarak
Antakya ilinden u kadar rn alnmtr.
eklinde kaytlar tutulmutur. Gerek eski Yunan, gerekse eski Roma alfabesinde saylar
metinsel harflerdir ve toplama karma ilemlerine uygun deildir. Eski devirlerde, saylar
saylabilir say sistemine gre uygulanmtr. Saylabilir say sistemi sonu ak olan
1 , 2, 3 , 4 , 5 , 6 .....
Kmesidir.Kmeler benzer zellikte nesneleri ieren gruplardr. Bu saylar sadece kayt ileri
iin kullanlmtr. Bu nedenle eski Yunan uygarlnda sadece Geometri gelimitir.
Drt ilem yaplmasna uygun olan sfr saysn ieren onlu say sistemi, eski Babilliler
tarafndan bulunmu, fakat yaygnlaamadan Babil tahrip olmutur. Bu say sistemi,
Hindistanda bir gzlemevi direktrnn abalar ile hatrlanm ve Abbasiler zamannda
Badat niversitesinde kullanlmaya balanmtr. Gnmzn onlu say sistemi olan bu say
sistemi "Arap Rakkamlar" olarak tannr.
Badat niversitesinde temel matematik ilemlerini gerekletiren ve "Cebir" adn
verdii matematik biliminin kurucusu, AbAbdallh Muammad ibn Msal-Khwrizm dir. Bu
bilim dal Endls Emevileri tarafndan Avrupaya tanm ve Rnesansn kaynaklarndan biri
olmutur. El Harzemi ad ile tannan bu bilim adamna sayg bildirgesi olarak, bilgisayar
bilimlerinde bir deerin hesaplanma yntemine "Algoritma" ad verilmitir.
2
Tamsaylar
Tamsaylar kmesine sfr saysnn eklenmesi ile Pozitif Tamsaylar elde edilir. Gerek
tamsaylar, gerekse pozitif saylar tam olarak metematik ilemlerin gerelletirilmesi iin
yetersizdir. rnek olarak, "beten alt karsa sonu ne olur ? " sorusuna verilecek bir yant
bulunamaz. Bu say sistemine negatif saylarn da eklenmesi ile drt ilem yaplmasna uygun
tamsaylar sistemi, bulunur. Bu sistemde saylar, bir eksen zerine dizili olarak dnlr:
-16 -15 -14 -13 -12 -11 -10 -9 -8 - 7 -6 -5 -4 -1 0 1 2 3 4 5 6 7 8 9 10 11 12 13 14 15 16
<------------------------------------------------------------------------------------->
ekil 1-1
Tamsaylar kmesi, eksi ve art sonsuz olarak dnlen saylar arasnda kalan tm
tamsaylar ierir. Bu tamsayla rkmes iinde ok eitli alt kmeler oluturulabilir. rnek
olarak "Doal Saylar Kmesi", 1 ve art sonsuz saylar dahil, aradaki tm tamsaylar
ieren bir kmedir. "Pozitif Saylar Kmesi", 0 dahil, art sonsuz dahil, aradaki tm
tamsaylar ieren bir kmedir. "Negatif Saylar Kmesi", negatif sonsuz dahil, 0 dahil,
aradaki tm tamsaylar ieren bir kmedir.
Eksi ve art sonsuz saylar dnsel saylardr. Her zaman, dnebildiimizden
daha byk bir art sonsuz ve dnebildiimizden daha kk bir eksi sonsuz says
bulunabilir.
Doal saylar pozitif saylardr ve gndelik yaamda pozitif iareti konulmadan
kullanlabilirler. Negatif saylarn negatif iaretle belirtilmesi gerekir.
Negatif saylar elle tutulamaz fakat gerektir. rnek olarak tarlanzn yllk vergisi
be kile buday ise ve ancak iki kile verebiliyorsanz, bir sonraki yl hissenize kalabilecek
budaydan kile daha alndnda negatif saylarn da gerek olduunu ac bir ekilde
anlarsnz.
Tamsaylarn bir doru zerinde diziliine "Tamsaylar Skalas" ad verilir. Skala
szc Trkeye iskele olarak gemitir. Eskiden kumalarn lld dz tahtalara
skala yani lt ad verilirdi. Skalaya uygun yani lte uygun olarak nitelendirme bir
performans ltdr ve Milano da "La Scala" operas performansn en yksek olduu yer
anlamna gelir. Tamasaylar skalasnn en byk deeri art sonsuzdur ve tm kme
elemanlar en kk deerden balanarak , en bye kadar skala zerine yerletirilir.
ekil 1-1 deki skalaya baklnca, en kk saynn eksi 16 olduu grlr. Bu ekilde, eksi
15 de eksi 14 den kktr. Bunu onbe kile borcun ondrt kile bortan daha byk
olduunu dnerek bulabilirsiniz. Bir say, eksi sonsuza ne kadar yaknsa okadar kk,
art sonsuza ne kadar yaknsa okadar byktr. Bir say, art sonsuzdan ne kadar uzaksa o
kadar kk, eksi sonsuzdan ne kadar uzaksa o kadar byktr.
3
Tamsaylarn Toplanmas ve karlmas
Toplama ve karma ilemlerinde iaretlere dikkat edilir. rnek olarak,
2 3 1 = 4 5 + 9 =
Tamsay arpm
Tamsay arpm, iaretler dikkate alnarak yaplr. rnek olarak,
15 ( ) 9 ( ) 135 = 9 7 ( ) 63 = 87 56 4872 =
arpma srasnda pozitif iaretlerin yazlmadna dikkat ediniz.
Parantezli ilemlerde, ,llk nce parantezin iindeki ilemler tamamlanr. Sonra
parantezin nndeki arpma ilemi gerelletirilir. Sonra, parantez iindeki terimlerin herbiri
parantezin nndeki terimle arplr. Eer parantezin nnde +1 varsa parantez alr ve
sonraki ilemler yaplr. Eer parantezin nnde -1 varsa, parantezin iindeki terimler -1 ile
arplr ve parantezin nnde +1 var hale getirilir.
3 5 8 + ( ) 9 = Alm : 5 8 + ( ) 3 = 3 3 9 =
3 5Lira 6USS + ( ) Sonu : 15 Lira 18USS
36 78 ( ) 42 = Alm : 36 78 ( ) 42 = 1 42 ( ) 42 =
Bir saynn sfr ile arpmnnn sonucu sfrdr.
5 0 0 =
4
Tamsaylarn Blnmesi
her zaman
Eer m= ve n sfrdan farkl iki tamsay iseler,
m
n
tamsay vermeyebilir. Buna ramen her tamsay mutlaka hem + veya eksi kendisine
veya art veya eksi 1 saysna blnebilir.
5
5
1 =
5
5
1 =
5
1
5 =
5
1
5 =
Bir say eer, art veya eksi kendisine veya art veya eksi 1 den baka bir sayya
blnemezse, o say asal bir saydr. rnek
2 3 5 7 11 13 17 19 23 29
Asal saylarn dndaki saylar. bileik saylardr ve asal saylar ile arplarak daha basit olarak
belirtilebilirler. Bunlara asal say katsaylar ad verilir. rnek,
8
4
=
4 2
2 2
= 2
Bir saynn sfra blnmesinin sonucu belirsiz bir durumu belirtir.
12
0
=
0
Belirsiz
Bir saynn sonsuza blnmesinin sonucu sfrdr.
45

0 =
45

0 =
Bunun anlam,
45
1000000000000
4.5 10
11
=
45
1000000000000
4.5 10
11
=
rnei ile daha kolay alglanabilir.
Bir saynn sonsuzla arpmnn sonucu sonsuzdur.
89 =
5
Rasyonel Saylar
Eer m ve n saylar , sfrdan farkl tamsaylar ise, rasyonel saylar kmesi, tm m/n orannn
sonucu olan elemanlar ierir. Bu durumda, rasyonel saylar kmesi, hem tamsaylar hem de
tamsay olmayan rasyonel saylar ierir.
Bir rasyonel saynn blnen ve bleni ayn say le arplrsa oran deimez. rnek,
8 72
4 72
2 =
8
72
2

4
72
2

2 =
Rasyonel saylar kmesi, tamsaylar kmesi ile ayn skalay paylar. Rasyonel saylar
kmesi eksi ve art sonsuz aras tm rasyonel saylar ierir.
Rasyonel saylarn gsterildii skala zerinde saylar birbirlerii le eit uzaklkta olan
aralklarla yeretirilir. Bu ekilde uzaklklar ve byklkler arasnda orant sabit
olacandan saylarn karlatrlmas daha kolaylkla yaplablir. 0 ile 1 arasndaki
mesafeye skala yani lt ad verilir. Eer lt 3.5 cm ise, 0 ile +1 aras uzunluk 3.5 cm ,
ayn ekilde, 0 ile -1 arasnda uzunluk yine 3.5 cm dir. 0 ile +2 arasndaki uzunluk 2 x 3.5
cm , ayn ekilde 0 ile -2 arasndaki uzunluk 2 x 3.5 = 7 cm dir. Her defasnda deiik
saylar ile belirtmek gereksiz ilem yaplmasna neden olacandan, tm uzunluklar skala
uzunluuna blnebilir ve 0 ile 1 aras uzunluk,

3.5
3.5
1 =
birim olarak adlandrlr. Bu ekilde 2 saysnn 0 dan (orijin) den uzakl,
7
3.5
2 =
birim olarak belirlenir. Bylece skala deeri istendii kadar seilerek, birbirlerinden
uzakl sabit oranlarda olan saysal yerleimler oluturulabilir. Bu ileme "normalizasyon"
ad verilir. Normalizasyon ilemlerinden grafik ve istatistik almalarnda geni lde
yararlanlr.
6
Eer iki rasyonel saynn bleni ayn sfrdan farkl bir say olan n
ise,
r
n
ve
s
n
bu durumda, ortaya kan rasyonel saylarnn byklkleri, sadece blnenlerin
byklklerinin karlatrlmas ile belirlenir. rnek olarak
15
4.5
ile
18
4.5
rasyonel saylarnn ikincisi ilkinden byktr. nk, 18 says, 15 saysndan byktr.
Bu durumda, rasyonel saylarnn birbrler ile kolaylkla karlatrlmasnn salanmas ve
aralarnda toplama ve karma ilemlerinin yaplabilmesi iin, rasyonel saylarn ortak blen ile
belirtilmesi yararl olmaktadr.
Blenleri sfrdan farkl olan her rasyonel saynn ortak bleni bulunur. Bu deer,
q
f
e
p
rasyonel saylar iin, f x p deeridir. rnek,
56
34
ile
119
68
arasnda ortak
blen,
34 68 2312 = eklinde bulunur. Bu
ekilde,
56
34
=
56
34
2312
34
2312
34
=
56 68
2312
=

3808
2312
Ayn ekilde,
119
68
=
119 34
2312
=
4046
2312
olarak yazlabilir.
7
Eer iki blen arasnda en kk ortak blen (least common divisor =lcm) bulunabilirse,
ilemler daha kolay gerelletirilebilir. rnek olarak, rasyonel say, asal say bileenlerine
ayrlabilir.
56
2
28 =
28
2
14 =
14
2
7 = 2 2 2 7 56 =
34
2
17 =
56
34
=
2 2 2 7
2 17
ve 2 ler basitletirilirse,
2 2 7
17
=
28
17
Ayn ekilde,
119
7
17 = ve
68
2
34 =
34
2
17 = olduundan,
119
68
=
7 17
2 2 17
7 ler basitleir, ve
7
2 2
=
7
4
Bylece,
7
4
ile
28
17
arasnda, en kk ortak blen = 4 17 68 =
olur. Bu yntem epeyce emek youn olduundan, Mathcad 15 ile bu ilem,
lcm 34 68 , ( ) 68 = olarak gerekletirilebilir.
Sonuta
olduundan, 56 2
68
=
112
68
ve ile ve 56
34
119
68
112
68
119
68
rasyonel say iftlerinin birbirlerinin ayn olduu grlr..
8
ki rasyonel saynn toplanmas iin, nce ortak paydalar bulunur, sonra paylar birbirine
eklenir. rnek,
45
89
57
23
+ 2.984 = lcm 89 23 , ( ) 2047 =
2047
89
23 =
2047
23
89 =
45 23 57 89 +
2047
2.984 = olarak bulunur.
Rasyonel saylarn arpm,
a
b
c
d

e
f

a c e
b d f
=
eklinde, blmleri ,
a
b
c
d
a d
b c
=
eklinde gerekleir.
Eer a 0 ise yani payda sfrdan farkl ise, her
a
b
rasyonel saysnn
sonucu x olarak adlandrlabilen tek bir rasyonel saydr. yleki,
a x b =
olarak yazlabilir.
Eer,
a 0 ve b 0 = ise a x 0 = olur.
Bunun gerellemesi iin ya a sfr olmal, ya da x = 0 olmaldr. Verilerde, a sfrdan farkll
olarak tanmlanm olduundan, x eit sfr olur. Bu durumda,
say 0 0 =
olmaldr.
9
Eer,
a 0 = ve b 0 ise, 0 x b = yani, sfr say1 say2 =
ve
say2
0
say1 =
olur. Oysa ki, bu ifade anlamszdr, nk bir saynn sfrla arpmnn sonucu sfr olmaldr.
Sonucun bir baka sayya eit olacann belirtilmesi verilerin tutarszln belirtir. Bir yerde bir
hata olmaldr. Yani
say
0
Anlamsz =
Bu durum, anlamsz bir hali aklar ve asla buna olanak verilmemelidir.
Eer,
a 0 = ve b 0 = ise, 0 x 0 = yani,
0
0
Belirsiz =
Bu, belirsiz bir durumu aklar, nk her x deeri, denklemi salayabilir. Oysa, tanm olarak
sadece tek bir x deeri, denklemi salamaldr.
Yzde fadeleri
Yzde deerleri, % iaret ile belirtilir ve
say
100
deerini tar. rnek olarak, 34 saysnn
%5 deeri,
34
100
0.34 = saysdr.
Problem :
Eer 42 liradan 3 lira vergi kesiliyorsa, bu ilemden yzde ka vergi alnmaktadr?
zm bu ilemin zm, basit orant denilen ve her duruma uygulanabilen bir dnce
ile gerelletirilebilir. Bu dnce,"Eer 34 liradan 3 lira kesiliyorsa bu durumda, 100
liradan ayn oranda kesinti yaplr" eklindedir.
Kesinti yzdesi,
3
34
r = olur. Burada r deeri bilinmektedir ve hesaplanr:
r
3
34
= r 0.08824 = olarak hesaplanr.
100 liradan alnacak vergi miktar x lira olsun. Eer 100 liradan
10
alnacak vergi oran, 34 liradan alnan vergi oran ile ayn olacaksa o zaman,
3
34
x
100
= r =
olmaldr. Buradan, x 100 r = ve x 8.824 = Lira olarak bulunur.
Burada bulunan r, vergi oran % deeridir. Yzde deerinden sayya ,
(% deeri) x 100 olarak geilir. Burada 100 liradan 8.824 lira vergi alnmaktadr ve ilemin
vergi oran, r 0.08824 = % olarak ifade edilir.
Orantnn daha ematik bir ekli,
34 iin 3
100 iin x
eklinde ifade edilebilir ve
34 x 100 3 =
olarak formle edilebilir. Buradan 100 lira iin alnacak deer ,
x
100 3
34
= x 8.824 = olarak bulunur.
Vergi oran ise,
%
Deger
100
= olduundan,
8.824
100
0.088 = olarak bulunur.
Problem :
Bir banka, vadeli mevduat iin % 7.23 vermektedir. Eer bu bankaya 4500 lra yatrlrsa, vade
sonunda parann miktar ne olacaktr ?
zm :
Bilindii gibi,
7.23
100
x
4500
= ve x
4500 7.23
100
= x 325.35 =
lira.
Vade sonunda yatrlan 4500 lirann ulaaca deer :
11
yeni_deger 4500 4500
7.23
100
+ =
yeni_deger 4500 1
7.23
100
+
|

\
|
|

=
yeni_deger 4825.35 =
Bu sonucu genelletirelim,
yeni_deger yatrlan_deger 1 %faiz_oran + ( ) =
Buna soyutlatrma (abstraksion) da denilir. Soyutlatrma da ama, birok benzer duruma
uygulanabilecek bir balantnn gerekletirilmesi ve bu balantdan yararlanarak bir
hesaplama yntemi (algoritma) gelitirilmesidir.
Burada gerekletirilen balantnn etkin olarak uygulanabilmesi iin nce yllk yzde faiz oran
hesaplanr
yllk_yuzde_faiz_oran
7.23
100
=
katsay 1 yllk_yuzde_faiz_oran + =
yatrlan 4500 =
yeni_deger yatrlan katsay =
yeni_deger 4825.35 =
Bir yntem oluturulduktan sonra, ayn yntemi tm benzer ilemler iin uygulama olana bulunur.
rnek oarak, ayn faiz oran uygulandnda, 3500 lirann vade sonunda ulaaca deer,
yatrlan 3500 =
12
yeni_deger yatrlan katsay =
yeni_deger 3753.05 = lira olarak bulunur. Eer daha ksa deiken isimleri
ile ifade edilmek istenirse,
f 7.23 =
k 1
f
100
+ =
y 3500 =
t y k =
t 3753.05 =
Grld gibi, bir ilemi soyutlatrmak iin kullanlacak deiken adlarnn bir
nemi yoktur. Bunlar sadece ilemin genelletirilebilmesi iin kullanlan aralardr. Tornavida
ve penseden farklar yoktur, fakat anlamlarnn ok iyi anlalmas gerekir. Deiken adlar
verilerde nasl tanmlanmsa, uygulamada da kesinlikle yle kullanmak gerekir, yoksa tutarl
sonular alnamaz.
Grld gibi, harferin bir nemi yoktur. nemli olan yntemi anlamaktr.
stersek grek harfleri de kullanabiliriz.
(ksi)
1
7.23
100
+ = 3500 =
(n)
= 3753.05 = (psi)
13
Bir Problemin zmnde Gznne Alnmas Gereken Noktalar
(Polya'nn salk verdii yntemler ve yenileri)
ncelikle problemi iyi inceleyin ve tam olarak anlamaya aln. Anlamadnz problemi
zemezsiniz. Problemin amac nedir ? Veriler ne olmaldr, sonu olarak neler
istenmektedir. Problemi iyi anlamadan problemi zmeye almayn. Soruyu
anlamadnzsa aklanmasn isteyin.
Konunuzu ok iyi aln. Eer temel konularda yeterli deilseniz problemleri
anlayamazsnz.
Problemi zmeye balarken verileri inceleyin. Veriler yeterli ve tutarl olmal, sonularn
hesaplanmas iin yeterli veri olmaldr.
Sonular iyi inceleyin. Sonular beklenen byklklerde olmal, fiziksel beklentileri tam
olarak karlamaldr. 100 lira olmas beklenen sonu, 100 kuru olarak karsa tm
yntem yeniden ve iyice gzden geirilmelidir. Doruluuna tam olarak gvenmediiniz
hibir sonucu aklamayn. Acele yok ! Sakin ve emin admlarla ilerleyin.
Hesap hatas yapmayn. Daima bilgisayar kullann.
Irrasyonel Saylar
Baz saylar rasyonel say olarak belirtilemezler.Bu saylar arasnda ve e saylar da
bulunmaktadr. rrasyonel saylar, birbiri ile ayn olmayan, sonsuz ondalkla aklanabilen
saylardr.
rnek olarak,
3.141592653589793 = e 2.718281828459045 =
Rasyonel ve rrasyonel saylar, birlikte gerel saylar kmesini olutururlar. Gerel
saylar kmesi, eksi ve art sonsuz aras, tm gerel saylar ierir.
Kmeler
Kmeler nesnelerin sakland kutular olarak dnlebilir. Tm kutular gibi, kmeler de,
kapal hacmlardr ve kapa kapal olduu srece, kutunun iine birey giremez ve dna birey
kamaz. Kutulara birbiri ile ilgili eyalar konulduu gibi, kmelere de birbiri ile ilikili nesneler
konulur.
Gerel Saylar (Reel Saylar) Kmesi
Gerel saylar kmesi, eksi ve art sonsuz aras tm gerel (reel) saylar ierir. Bu szel
ifade, her defasnda yazlmak istendiinde ok vakit ve emek harcanmasn gerektirir. Bunu iin ,
matematikte eitli kavramlarn belirtilmesi iin ksayol iaretleri kullanlr.
14
Matematikte kullanlan ksayol iaretleri, kullanclara ok tuhaf geleblir. Bu iaretlerin
bizim canmz skmas iin hibir neden yoktur. Bunlar, bizim yararlanmamz iin
oluturulmulardr. ve bizim iin buradadrlar. Bizim iin kolaylk ve kavram netlii yaratrlar.
Bizim de onlarn anlamlarn iyi bilmemiz, anlamlarn bilmediimiz sembolleri kullanmamamz
gerekir. Yani bu semboller yazm ksal salayan steno iaretleri gibidir.
Gerel saylar kmesi, matematiksel olarak,
R = {tm gerel saylar}
eklinde belirtlebilir. Burada R kmenin addr. {} iaretleri, kmenin snrlarn belirtir. Tm
gerel saylar, R kmesinin elemanlardr. Kme isimleri byk harfle, kme elemanlar kk
harfle yazlr. Kmenin ierebilecei elemanlarn niteliklerinin ok iyi bir ekilde tanmlanm
olmas gerekir. Kme iindeki elemanlar ayrk taneler halinde kme iinde bulunurlar. Yani,
yukardaki gsterim sadece "R kmesi, tm gerel saylar ierir" cmleciinin ksayoldan
aklanmasdr. Hepsi bu kadar ve son derece basit. Artk her defasnda kme tanmn uzun
uzun yazmak zorunda kalmayacamz iin mutlu olmamz gerekir.
Her kutu iinde baka kutular bulunabilir. Kmeler de yle, her kme baka alt kmeleri
ierebilir. Kmeler iin bu kurallar daha skdr. Alt kme elemanlar ierisinde bulundular
kmenin ierik kurallarna uygun olmaldrlar, sadece bu kurallara ek olarak kendi kurallarna da
uyan nesneleri, eleman olarak ierebilirler. rnek olarak bir kutu iinde inciler bulunsun. Bu kutu
iinde bir ekmece de de siyah inciler bulunabilir. Siyah inciler, inci niteliinde olduklarndan,
inci kutusunda bulunabilirler. Ayrca bu inciler ek bir nitelik olarak siyah olduklarndan, inci
kutusunun zel olarak siyah incileri bulunduran bir ekmecesinde saklanabilirler. Yapay bir inci,
inci kutusuna giremez. nk inci kutusunun kurallar, sadece gerek incilerin, gerek inciler
kutusuna girebileceini belirtmitir. Yapay inciler iin, yapay inciler kutusu dzenlenebilir. Bu
kutudaki elemanlarn salamas gereken zellik olarak yapay inci olmalar belirtilirse, bu
kutunun ieriinde sadece yapay inciler olabilir.
Yukarda grlen kme notasyonu, liste notasyonu olarak adlandrlan kme
notasyonudur. Eer bir kmenin elemanlarnn ortak bir zellii varsa, kme zellikler
notasyonu yntemi ile de belirtilebilir.
R = {x| gerel saylar}
yani, R kmesi yle bir kmedir ki, elemanlar gerel saylardr eklinde okunabilir.
15
Bir Kme tanmnda, her trl matematik notasyon kulanlabilir. Bu ekilde,
kme elmanlarnn ortak zellikleri ksa yoldan aklanmaya allr. rnek olarak, bir
kme
R = { x: eksi sonsuzdan art sonsuza kadar tm gerel saylar}
eklinde tanmlanm olabilir. Bu sylem biraz fazla uzun olduundan,
R = {x: -<x<+}
olarak yazlabilir. Her ikisinin okunuu da ayn ve "eksi sonsuzdan art sonsuza kadar tm gerel
saylar" eklindedir. Semboller bizi artmasn. Bunlar sadece sylemin ksaltlmas iin
kullanlan yntemlerdir.
Bu kme iinde bulunan bir eleman ,
14.08 R

olarak belirtilir. Bu ifade en uzun olarak, "-14.08 says, gerel saylar kmesinin bir elemandr"
olarak, daha ksa olarak ise, "14.08 says, R kmesinin bir elemandr" eklinde okunur.
Eer bir nesne bir kmenin eleman deilse,
A R
olarak aklanabilir. Bunun anlam , "A nesnesi, R dizisinin bir eleman olamaz" eklindedir.
Tamsaylar Kmesi
Tamsaylar kmesi, gerel saylar kmesinin sadece tamsay olan elemanlarn ierir.
Tamsaylar kmesi Z kmesi oarak adlandrlr.
Z = {eksi sonsuz art sonsuz aras tm tamsaylar}
veya
Z = {...,-5,-4,-3,-2,-1,0,1,2,3,4,5,...}
olarak belirtilir.
16
Doal Saylar Kmesi (Natural)
Doal saylar kmesi, tamsaylar kmesinin bir alt kmesidir. Doal saylar (Natural)
kmesi, aadaki gibi aklanabilir:
N = 0,1,2,4,5,6,7,8,9,10,11,12,13,14,15,16,...}
veya
N = {x | 0 x , x Z }
yani, "doal saylar kmesinin elemanlar, sfr dahil art sonsuza kadar, tamsaylar kmesinin
elemanlardr", eklinde okunur.
Sayma Saylar Kmesi (Pozitif)
Sayma saylar kmesi, tamsaylar kmesinin bir alt kmesidir. Sayma saylar (Pozitif)
kmesi, aadaki gibi tanmlanabilir:
N
+
={1,2,4,5,6,7,8,9,10,11,12,13,14,15,16,...}
veya
N
+
= {x | 1 x , x Z }
yani, "sayma saylar kmesinin elemanlar, bir dahil art sonsuza kadar, tamsaylar
kmesinin elemanlardr", eklinde okunur. Grld gibi, matematik sembolleri birkez
allnca, ok etkin ve kolaylk salayan bir iletiim yntemi olutururlar.
Kapsam Alanlar
Kme elemanlarnn alabilecei deerler, kapsam alanlar ile aklanr. Kapsam
alanlar, "eksi ondokuz ile art ondokuz aras deerler" eklinde aklanr. Fakat bu szel
aklama, uzun ve sistematik olmaktan uzaktr. Daha ksa ve daha saysal ifade tarz ok
yararl olacaktr.
Kapsam alanlarnn tanmlar, allm mantksal iaretlerden yararlanlarak yaplabilir. rnek
olarak , pozitif saylar kmesinin elemanlarnn kapsam aralnn tanmnda bu yntemi
uygulamtk. Bir baka rnek de,
20 x < 20 <
17
olabilir. Bu rnekte "x deerleri, -19 ile art 19 arasnda deerler alabilir" ifadesinin matematiksel
aklanmas grlmektedir. Fakat bu gsterim de, fazla ak olmayabilir. Bunun iin kapsam
alanlarnn belirtilmesinde, zel bir ksayol iaretleri oluturulmutur. rnek olarak yukardaki
kapsam aral, -19 ile +19 deerlerini ieren ve bularn alt veya stndeki deerlere kapal olan
bir alandr ve -20 ile +20 deki iki duvarla snrlanan bir kapsam alandr . Bu tr kapsam alnlarna
"Kapal Kapsam Alanlar" ad verilir ve
[-20,20]
Bunun anlam, bu kapsam alannn -20 ile 20 deki duvar deerlerinin dnda tm aradaki
deerleri ierdiidir.
Baz durumlarda ak aralk tanmlanmak istenir. Ak aralk, duvar deerini de
kapsar.Ak aralk aadaki gibi gsterilir,
20 20 , ( )
Bunun anlam, -20 (dahil) , 20 (dahil) aradaki tm deerlerin kapsam iinde olduudur. Matematik
olarak,
20 x 20
olarak aklanabilir.
Bazen yar ak kapsam arallklar da tanmlanabilir. rnek olarak
x < 45 <
aral, yar ak bir kapsam aral tanmdr ve ksa olarak,
(- , 45]
olarak yazlabilir. Bu kapsam aral - sonsuzdan 45 'e (45 hari) kadar tm deerleri kapsar.
18
Eitsizlikler
Eitsizlikler ayn eitlik gibi alr, sadece her iki taraf -1 ile arpldnda
eitsizlik yn deitirir.
f a c <
f c a + <
Her iki tarafn da -1 ile arplmas, yani tm
elemanlarn iaretlerinin deitirilmesi
durumunda , eitsizliin yn de deiir.
f c a >
c a + f > Sadece elemanlar taraf deitirirse, eitsizliin
yn de deimez.
Mutlak Deerler
Mutlak deer, saysal deeri belirten iareti gznne almayan bir ksayol gsterim
eklidir. rnek,
2
-2 veya 2 olabilir.ok kullanlan bir mutlak der ifadesi,
x L <
eklndedir. Bu ifadenin alm,
x L <
x L <
x L + <
olabilecei gibi,
19
x L ( ) <
x L + <
x L <
Her iki taraf -1 ile arplr ve iaret yn
deitirir.
x L >
L x <
eklinde de olabilir.
Sonuta,
L ( ) x < L + ( ) <
Kapsam aral elde edlir.Burada deeri, sonsuz kk olarak seilirse, x, L deerine sonsuz
kkten daha yakn fakat asla L deerine eit deildir. L deeri, x deeri iin bir limit deer
anlamna gelir. Burada zerinde durulmas gereken iki ey bulunmaktadr:
Sonsun kk belirl bir say deildir ve her sonsuz kk olarak dnlen saydan daha 1.
kk bir say bulunabilir.
Bir deerin limit deeri asla deere eit olamaz, ama ama sonsuz kk bir farkla 2.
yaklak olabilir.
stel ifadeler
stel ifadelerle almak ok koladr. Sadece aada grlen yntemlerin iyi
bilinmesi yararl olacaktr.
Bir saynn ss, saynn s kadar kendisi ile arplmasndan baka birey deildir.
a
n
a a a a = ... n n kere
3
5
= 3 3 3 3 3 243 =
fadelerin hesaplanmasnda, eer parantez varsa, parantezler kendi ilerinde, da
kapal bamsz bir baka ifade olarak kabul edlir. Paranetezler zerine yaplacak
ilemler tm parantez elemanlar zerine uygulanr. Parantez yoksa, tm ifade, bamsz
bir ifade olarak deerlendirilir. lem ncelii, s alma , arpma veya blme, en son
20
bir ifade olarak deerlendirilir. lem ncelii, s alma , arpma veya blme, en son
olarak toplama veya karma ilemlerindedir. Yani, bir ifade almak isteniyorsa, nce s
alma ilemleri gerekletirilir, sonra arpma veya blmeler yaplr, en son olarak,
toplama veya karma ilemleri gerekletirilir.
Eer,
2 ( )
4
deerinin hesaplanmas isteniyorsa, tm parantezin 4 nc
kuvvetinin hesaplanmas gerekir. Bunun iin yaplacak ilem, parantezi, drt kere kendisi ile
arpmaktr.
2 ( ) 2 ( ) 2 ( ) 2 ( ) 4 4 = 16 =
a b + ( )
2
ifadesinin deerinin hesaplanmas iin de yaplacak i tam olarak ayndr.
Yani parantez kendisi ile arplacaktr.
a b + ( )
2
a b + ( ) a b + ( ) = a a a b + b a + b b + = a
2
2 a b + b
2
+ =
Bu gibi iki elemanl ifadelerin slerinin hesaplanmas iin, Binom (bi =iki
,nom=numara, latince, yani iki elemandan oluan ifadeler anlamna geliyor) almlar
kullanlabilir. Bu basit forml Pascal geni le birlikle ileride greceiz. Her ikisi de sonucu
kolay aklda tutma ynteminden baka bir ey deildir. Asl olan yukarda olduu parantezin
kendisi ile arpmdr.
Fakat, eer x
2
ifadesinin hesaplanmas isteniyorsa, durum deiiktir, nk, burada
parantez yok ve uygulanmas gereken ilemler var. lemler de, ilem ncelik srasna gre
uygulanr. Yani nce s alnr. Burada s alma ilemi sadece x deerini ilgilendirir. nce x 'in
ss alnr. Sonra bulunan deer toplanr veya karlr. Bu durumda rnek olarak,
2
2
2 2 ( ) = 4 = olarak bulunur.
Tekrar edelim : 2 ( )
2
4 = 2
2
4 =
Bunu daima aklmzda tutalm.
21
1 a
m
a
n
a
m n +
= 2
3
2
5
2
8
= 2
8
256 =
2
3
8 = 2
5
32 =
8 32 256 =
a
n
( )
m
a
nm
= 2
2
( )
3
2
6
= 64 = 4
3
64 =
2
3 a b ( )
n
a
n
b
n
= 2 6 ( )
2
12
2
= 144 =
2
2
6
2
144 =
4x
4
y
5
( )
3
dikkat bu ifade , 4 ( ) x
4
( ) y
5
( )

3
olarak verilmitir ve 4
3
x
4
( )
3
y
5
( )
3
4
3
x
12
y
15
=
olara alr. Burada 4 ayr bir terimdir ve teki x ve y terimleri gibi 3 n
kuvveti alnmaldr.
10 z
2
y
4
( )
2
z
3
y ( )
5

Bu ifade 10 ( ) z
2
( ) y
4
( )

2
z ( )
3
y ( )

olarak okunmaldr. Burada ncelik sras s alma ilemindedir. Dikkat edilmesi


gereken konu, -10 teriminin eksi iareti ile birlikte karesinin alnmas gerektiidir.
10 ( )
2
z
2
( )
2
y
4
( )
2
z
3
( )
5
y ( )
5
100 z
4
y
8
z
15
y
5
=
22
= 100 z
11
y
13

100
z
11
y
13

=
a
b
|

\
|
|

n
a
n
b
n
= b 0 ( )
18
3
|

\
|
|

2
6
2
= 36 =
4
18
2
3
2
324
9
= 36 =
5
a
n
a
m
a 0 ( )
eer m n > ise m n ( ) 0 >
a
n
a
m
1
a
m n ( )
=
6
2
6
3
1
6
=
6 6
6 6 6
1
6
=
6
2
6
3
1
6
3 2 ( )
=
1
6
1
=
1
6
=
eer n m > ise n m ( ) 0 >
a
n
a
m
a
n m
=
6
3
6
2
6
3 2
= 6 =
6
3
6
2
6 6 6
6 6
= 6 =
23
6
a
b
|

\
|
|

n
b
a
|

\
|
|

n
=
b
n
a
n
=
5
15
|

\
|
|

3
5
3
3
3
5
3

=
1
3
3
=
1
1
3
3
= 3
3
= 27 =
5
15
|

\
|
|

3
15
3
|

\
|
|

3
= 3
3
= 27 =
7
a b ( )
n 1
a b ( )
n
=
23 6.7 ( )
5.8
1
23 6.7 ( )
5.8
=
1
23 6.7 ( )
5.8
2.045 10
13
=
8
1
a
n
a
n
=
1
5
2
25 =
9
a
n
b
m
b
m
a
n
=
2
2
4
2
16
4
= 4 =
24
10 a
n
b
m

( )
k
a
nk
b
mk
=
3
3
5
2

( )
3
3
9
5
6
+ =
3
9
5
6
307546875 =
3
3
5
2

( )
3
307546875 =
11
a
n
b
m
|

\
|
|
|

k
a
nk
b
mk
=
2
4
5
2
|

\
|
|

3
0.262 =
2
4 3
5
2 3
0.262 =
Grld gibi, alma yntemine alldnda, stel ifadelerle almak son derece
kolaydr.
Radikaller
Tanm :
Eer n says pozitif ve 1 den byk bir say ise, ve a says da bir gerel say ise,
1 n < < n
a
1
a
n
=
a R
Burada n says indeks , iareti, kk veya radikal olarak adlandrlr.
Yukardaki ifade, a saysnn n inci kk olarak okunabilir. Daha anlalr bir ifadesi, hangi
25
Yukardaki ifade, a saysnn n inci kk olarak okunabilir. Daha anlalr bir ifadesi, hangi
saynn n inci kuvveti a saysn verir ? eklinde okunmasdr. rnek olarak,
4
16
ifadesi, "Hangi saynn 4 nc kuvveti, 16 saysn verir?"
eklinde okunabilir. Bu soruya yant kolayca verilemez. Radikalleri hesaplayacak basit
yntemler yoktur. Olan yntemler ise uzun ve zahmetlidir. Gnmzde, hesap makineleri
bu sorunun zmne yardmc olur.
Yukardaki problemim zm, eski blgiler hatrlayarak gerekletirilebilir:
2
2
2
2
2
2
2
2
2
4
= 16 =
4
16
4
2
4
= 2 = ( 4 ler basitleir)
Fakat eski bilgilerle zemeyeceimiz deerler olunca i gleir. Bu sorunu basit hesap
makineleri zer.
3
35
12
1.501 10
6
= 35
12
( )
1
3
1.501 10
6
=
5
24 1.888 =
24
1
5
1.888 =
48.23
1
3.56
2.971 =
En ok kullanlan radikal, kare kk deeridir.
Burada,
36 36
1
2
=
olarak hesaplanr.
Bu ifade, "Acaba hangi deer, kendisi ile arplnca 36 saysn verir ? " diye okunursa,
yant daha kolay bulunabilir. Bu deer 6 dr nk 6 x 6 =36 eder. Yani,
26
36 6 =
Daha karmak kare kklerde, hesap makinesi kullanlabilir.
15.7545954 3.969 =
Radikaller, stel say hesaplama yntemleri ile zmlenirler. En genel radikal tanm,
a
m
n
a
1
n
|

\
|
|

m
=
n
a
( )
m
=
n
a
m
=
veya
a
m
n
a
m
( )
1
n
=
n
a
m
=
olarak tanmlanr.
zellikler :
n pozitif bir tamsay ve a ve b birer pozitif gerel say iseler,
n
a
n
a =
n
a b
n
a
n
b =
n
a
b
n
a
n
b
=
bantlar yazlabilir.
rnek :
28 4 7 = 4 7 = 2 7 =
28 5.292 =
27
2 7 5.292 = olarak hesaplanabilir.
rnek : 3 4 +
( )
5 7
( )
3 5 7 3 4 5 + 28 =
15 7 3 4 5 + 28
3 5 7 3 4 5 + 28
5 3 4 +
( )
7 3 28 27.307 =
Orijinal ifade :
3 4 +
( )
5 7
( )
27.307 =
rnek : y
8
ifadesinin basitletirilmesi,
y
8
( )
1
2
y
8
2
= y
4
=
rnek :
3
t
12
ifadesinin
basitletirilmesi,
t
12 ( )
1
3
t
12
3
= t
4
=
rnek :
2 x 3 ( ) 18 = x deerini bulunuz .
28
Burada iki ayr soruya yant aramak gerek, ,ilki "acaba hangi gerek deerler, mutlak
deer (2 x - 3) deerini oluturabilir ? "dieri de, "acaba hangi deer, kendisi ile
arplnca, kk iindeki terimi oluturulabilir ?".sorulardr
lk sorunun yant
:
2 x 3 ( ) 2 x 3 ( ) veya 2 x 3 ( )
eklindedir.
Gerel Deerler ile alldnda, kk iindeki deer mutlaka pozitif olmaldr. nk
hibir gerel byklk kendisi ile arplnca negatif sonu vermez.
Bu durumda, bamsz deiken x deerinin kapsam araln saptayabiliriz.
2 x 3 ( ) 0 > 2 x 3 ( ) 0 <
3
2
x < x
3
2
<
3
2
x < < x <
3
2
<
Bu durumda fonksiyonun tanm iki farkl kapsam aral iin farkl olmaktadr. Buna paral
(piecewise) bir fonksiyon ad verilir.
Artk fonksiyonlarn zmne geebiliriz.
lk kapsam aral iin,
x <
3
2
<
2 x 3 ( ) 18 =
2 x 3 ( )

2
18
2
=
2 x 3 ( ) 324 =
=
29
2 x 324 3 =
x
321
2
= x 160.5 =
kinci kapsam aral iin, (3/2 < x < )
2 x 3 ( ) 18 =
2 x 3 ( )
2
18
2
=
2 x 3 324 =
2 x 327 =
x
327
2
= x 163.5 =
Dzenlenm Fonksiyonlarn Tanm
lk kapsam aral iin, (- < x < 3/2)
f x ( ) 2 x 3 ( ) 324 = f x ( ) 2 x 3 + 324 =
f x ( ) 2 x 321 =
kinci kapsam aral iin, (3/2<x<)
f x ( ) 2 x 3 324 = f x ( ) 2 x 327 =
f x ( ) if x <
3
2
< 2 x 321 , 2 x 327 ,
|

\
|
|

=
30
x 20 19.9999 , 20 .. =
20 10 0 10 20
330
320
310
300
290
280
f x ( )
x
Dzenlenmi denklemlerden elde edilen kk deerlerinin esas fonkisyonda mutlaka denenmesi
gerekir, nk dzenleme srasnda yaplan ilemler, doru fakat orijinal fonksiyondan farkl bir
bant ile sonulanm olabilir.
lk kapsam aralnda (- < x < 3/2) orijinal fonksiyon
fo x ( ) 2 x 3 ( ) 18 =
kinci kapsam aralnda (3/2<x<) orijinal fonksiyon
fo x ( ) 2 x 3 ( ) 18 =
fo x ( ) if x <
3
2
< 2 x 3 ( ) 18 , 2 x 3 ( ) 18 ,

=
31
x 200 199.99 , 200 .. ( ) =
200 100 0 100 200
20
10
0
10
20
fo x ( )
x
Grld gibi, orijinal fonksiyon, dzenlenmi fonksiyondan farkl, fakat kk deerleri ayn
gibi grlyor. Kesin sonu, ancak saysal kontrol le anlalabilir.
Sonularn Salanmas :
2 x 3 ( ) 18 0 =
2 160.5 ( ) 3 [ ] 18 0 =
32
veya f 160.5 ( ) 0 = Kk 1 geerli !
2 x 3 ( ) 18 0 =
2 163.5 ( ) 3 18 0 =
veya f 163.5 ( ) 0 = Kk 2 geerli
!
Sonu olarak, bu rnek iin, dzenlenmi denklemlerden bulunan kk deerlerinin orijinal
fonksiyonda da geerli olduu grlr. Bu sonu, her durum iin geerli deildir ve
dzenlenm denklemlerden bulunan kk deerlerinin, daima orijinal denklemlerde de
salanmas ve dorulanmas gerekir.
Negatif Radikaller
Eer indis bir ift say ise, negatif radikaller olabilir.
rnek,
3
125
Yani,"Hangi say, kendisi ile kez arplnca, -125 sonucunu
verebilir?"
Yant : (-5) x (-5) x (-5) = -125
yani
3
125 5 =
Radikal indisi tek say olursa, yani 3,5,7 vb... olursa, sonucu gerel say olan negatif
radikaller olabilir. Fakat radikal indisi 2 (karekk) , 4, 6, 8 gibi ift saylar olursa, olay
olumsuzlua dnr. nk negatif bir karekk olamaz, bunun nedeni, hibir gerel
say kendi kendisi ile arplnca negatif bir say veremez.
4
sonucu ne olabilir ? Yani "Hangi say kendi kendisi ile arplnca --4 verir ?"
Yant : Hibir gerel say kendisi ile arplnca negatif bir sonu veremez. Gerel saylar
33
Yant : Hibir gerel say kendisi ile arplnca negatif bir sonu veremez. Gerel saylar
kendisi ile arplnca daima pozitif bir say verir. rnek,
2 ( ) 2 ( ) 4 =
2 ,
4
16 gibi saylar, gerel saylar kmesinde
bulunmaz.
2 ,
4
16 gibi ifadelerin sonular ne olacak ?
O zaman
Matematik ilemlerin yrtlebilmesi iin, tek elemanl yeni bir say kmesi
yaratlm ve buna Sanal Say Kmesi, ad verilmtir. Bu kmede sadece bir eleman bulunur
bu da i saysdr. Sanal say kmesinin tek eleman olan i saysna sanal say ad verilir. Sanal
i saysnn deeri,
i 1 =
olarak tanmlanr. Bu ekilde,
4 2i = 4 4 1 ( ) = 4 1 = 2 i =
olarak bulunur. Bu tanma gre,
i
2
1
( )
2
= 1 ( )
2
= 1 =
sonucunu verir . Bu zellik, birok matematik ilemin gerek say sonucunun bulunmasna
yardm eder.
Sanal saydan yararlanlarak, a + b i eklinde kompleks saylar oluturulabilir. Kompleks
saylar, ax
2
+ b x +c = 0 eklinde, ikinci derece denklemlerin kklerinin bulunmasnda bile
ortaya kabilen, matematik zmler iin ok yararl dzenlemelerdir.
Kompleks Saylar
Kompleks saylar, a+ b i olarak tanmlanr. Burada a gerel ksm, i sanal ksm olarak
isimlendirilir. Her kompleks saynn a - b i eklinde bir elenii (konjgesi) vardr. Baz
34
isimlendirilir. Her kompleks saynn a - b i eklinde bir elenii (konjgesi) vardr. Baz
kompleks saylar sadece 5 i gibi sadece kompleks ksmdan oluabilirler.
Kompleks saylar, cebir ve anatik geometri konularnda aklanamayan ifadelerin
aklanmas iin olutuulmu kolaylk dzenlemeleridir. Negatif saylar gibi, gerekle ilgileri
yoktur.
Kompleks saylarn ifadeleri, ayn normal ifadere gibi dzenlenir. fadelerde i says sabit
bir deer olarak kabul edilir. rnek olarak bir kompleks saynn elenii ile arpm,
a bi + ( ) a bi ( ) a
2
a b i b a i + b
2
i
2
=
ve i
2
1 = olduundan
a bi + ( ) a bi ( ) a b + ( )
2
=
olarak bulunur. lgin, fakat gerek say olmayan bir sonu.
Bir baka rnek :
a b x + ( ) 5 x 5 a x 5 b x
2
+ =
a bi + ( ) 5i 5 a i 5 b i
2
+ =
ve
i
2
1 = olduundan
sonu : 5 a i 5 b 5 a i b ( ) =
Yani, a b x + ile a b i arasnda cebirsel ilem olarak
hibir fark yoktur.
Saylar ve Rakkamlar,
Rakamlar, saylarn olutuulmas in kullanlan iaretlerdir.
Onlu say sistemi (desimal) on tane rakkamdam oluur.
Onlu say sisteminin rakkamlar : 0 , 1 , 2 , 3 , 4 , 5 , 6 , 7 , 8 , 9
Onlu sistemde, bu on tane rakkamdan yararlanlarak, her trl gerel veya kompleks say
35
Onlu sistemde, bu on tane rakkamdan yararlanlarak, her trl gerel veya kompleks say
yaratlabilir.
rnek olarak . 103 says 100 +3 olarak ifade edileblir. 100 says ise 10
2
olarak
yazlabildilinden 103 says, 10
2
3 + olarak yazlabiir.

Ayn ekilde, 10 10
1
= 1000 10
3
= olarak yazlabiir.
0
10
0
1
1
10
1
10
2
10
2
100
3
10
3
1000
4
10
4
1000
|

\
|
|
|
|

ve
devam..
rnek olarak 10023 says 1 10
4
2 10
1
+ 3 10
0
+ 10023 =
Biraz srad, fakat bilgisayarlarn saylar nasl hersapladklarnn anlalmas asndan da ilgin.
Ondalklarn gsterimi, onlu sistemde kolay :
0.1 =
1
10
10
1
= 0.1 =
102.4 1 10
2
2 10
0
+ 4 10
1
+ =
O zaman bir ksayol sistemi oluturup saylar yle yazalm
:
4
10
4
3
10
3
2
2
10
2
3
1
10
1
1
0
10
0
0
1
10
1
0
2
10
2
0
3
10
3
1
4
10
4
6
|

\
|
|
|
|

36
2 10^3 + 3 10^2 + 1 10^1 + 0 10^0 + 0 10^-1 + 0 10 -2 + 1 10 ^-3 + 6 10^-4 = 2310.0016
Grld gibi onlu say sisteminde saynn tamsay ve ondall ksmlar bir toplama ilemi olarak
yazlabiliyor. Bu ilem her say sisteminde gerekletirilebilir.
kili Say Sistemi (Binary)
kli say sisteminde sadece ik rakam vardr. Bu rakamlar 0 ve 1 dir fakat her say bu
iki rakkamla oluturulablir. Modern bilgisayarlarn alma sistemleri ikili alma sistemine
dayanr. Buna ikili mimari (binary architecture) ad verilir. Temel say ikidir.
2
0
1 = 2
1
2 = 2
2
4 = 2
3
8 = 2
4
16 =
2
5
32 = 2
6
64 = 2
7
128 = 2
8
256 =
Bilgisayar sisteminde bu saylar, sra halinde dizilen transistrlerin 0 ve ya 1 karl voltaj alp
almadklar ile belirlenir
8
1
1 2
8

7
1
1 2
7

6
1
1 2
6

5
1
1 2
5

4
1
1 2
4

3
1
1 2
3

2
1
1 2
2

1
1
1 2
1

0
1
1 2
0

\
|
|
|
|

yani ikili 1 1 1 1 1 says, onlu olarak,


1 2
4
1 2
3
+ 1 2
2
+ 1 2
1
+ 1 2
0
+ 31 =
deerine eittir.
Bilgisayarlar ikili say sistemi ile tm saylar hesaplayabilirler. Saylarn tamsay
ksmlar mantis, ondalk ksmlar eksponent adn alr ve tm ayr ayr hesaplanp toplama
ilemi ile saylar oluturulur. Daha nceleri Leibniz ikili say sistemlerinin olanaklarna iaret
etmitir. Fakat modern bilgisayarlarn ilem gc olmadan bu say sistemini pratik olarak
kullanma olana olmamtr.
On Altlk Say Sitemi (Hexagesimal veya ksaca Hex)
37
On altlk say sisteminde 16 tane rakkam vardr ve sistemin temel says 16 dr. Bu
rakkamlar 0 , 1 , 2 , 3 , 4 , 5 , 6 , 7 , 8 , 9 , A , B , C , D , E , F saylardr. Burada, A says 10, B
says 11, C says 12, D says 13, E says 14 ve F says 15 deerini tar. Bu sistem, ikili say
sisteminin saylarn insanlar iin daha kolay anlalr hale getirmek iin oluturulmutur.
Hex saylar 4 l gruplar halinde kullanlr. 16 bitlik sistemlerde saylar tek 4 l grup ile
ifade edilir. 32 bit sistemlerde 4 tane 4 l grup, 64 bitlik sistemlerde 4 tane 4 l grup kullanlr.
3
2
2 16
3

2
A
10 16
2

1
F
15 16
1

0
3
3 16
0

\
|
|
|
|

2AF3 = 2 16
3
10 16
2
+ 15 16
1
+ 3 16
0
+ 10995 =
15 16
3
15 16
2
+ 15 16
1
+ 15 16
0
+ 65535 =
FFFF =
anlamna gelir. FFFF says, 16 bitlik sistemlerde ifade edilebilecek tamsaylarn en yksek
deerini aklar. On altlk say sistemi ile bilgisayar bellek adresleri gibi, donanmla ilgili
deerler belirtilir. Bu say sistemi ile aritmetik ilem yaplmaz.
kili fadeler (Binom)
kili ifadeler, a b + ( )
n
ekl,ndedir ve a b + ( )
2
, a b + ( )
3
, a b + ( )
4
eklinde ifadeler
ile karlalabilir. Bu ifadelerin zm ekilleri en eski olarak Hindistanda , "Meru Dann Merdiveni"
olarak gelitirilmi, inliler, ranl (mer Hayyam geni) en son olarak da Pascal tarafndan
aklanmtr.
gen en tepede 1 ile balar. 1
kinci srada 1 ler bulunur 1 1
nc Srada 3 tane eleman bulunur. 1 1 1 + 2 = 1
38
lk eleman 1 dir ikinci eleman stten soldaki ve sadaki iki elemann toplamdr (1+1) = 2
Burada a b + ( )
2
= a
2
2 a b + b
2
+ almn katsaylar grlr.
Sonra, drdnc srada 4 tane eleman bulunmaldr. En soldaki eleman yine 1 dir. En
soldaki ikinci elemann st sanda nc srann en solundaki eleman olan 1 , st
sanda ise , ikinci eleman olan 2 bulunmaktadr. Bu durumda, drdnc srann ikinci
elemannn deeri, 1+2 =3 olacaktr. Bir sonraki elemann st sanda 2 , st solunda 1
olduundan yine 2+1 = 3 olacaktr. En sada ise yine 1 bulunacaktr. Bylece 4 nc
sra, 1 3 3 1 olarak sralanacaktr.
1 3 3 1
a b + ( )
3
= a
3
3 a
2
b + 3 a b
2
+ b
3
+
Beinci sra ayn ekilde 1 4 6 4 1
olarak hesaplanr.
a b + ( )
4
= a
4
4 a
3
b + 6 a
2
b
2
+ 4 a b
3
+ b
4
+
Altnc srada 6 eleman olacaktr . En solda 1, onun yannda 1+4 =5 , onun yannda 4+6 = 10 ,
onun yannda 6+4 = 10, onun yannda 4+1= 5 , en sada ise yine 1 olacaktr.
1 5 10 10 5 1
a b + ( )
5
= a
5
5 a
4
b + 10 a
3
b
2
+ 10 a
2
b
3
+ 5 a b
4
+ b
5
+
Grld gibi, binom ifadelerinin almlarnn ezberlenmesi gerei yoktur. Bu ilemler iin,
kutsal Meru dann merdivenlerinin anmsanmas yeterli olacaktr.
Denklemler
Denlemleri anlaya alren en yararl olacak ara, bie el terazisinin anmsanmasdr. rnek olarak,
terazinin sol tarafnda 7 portakal, sanda ise yerel arlk birimi olan bir tane demir arlk ktlesi
olsun. Terazinin denk olduu grlyor. Bu durumda
39
7 portakal = 1 birim_ktle
olarak belirtilebilir. imdi bu 7 portakal dan birisi bir ekilde eksilirse, terazi artk denk
olmayacak ve saa, yani deimeden kalan birim_ktlenin bulunduu tarafa ynlenecektir.
5 portakal < 1 birim_ktle
Tekrar denkliin salanmas iin, 2 portakal bulup sol tarafa
eklersek,
5 portakal + 2 portakal = 1 birim_ktle
veya
7 portakal = 1 birim_ktle
olarak belirlenebilir ve terazi yeniden dengeye gelir.
Eer lahana tartyorsak, bir lahanann birim_ktleden daha hafif olduunu gzlemleyebiliriz.
1 lahana < 1 birim_ktle
Terazinn denkliini salamak iin 3 tane mandalina lahanann bulunduu tarafa eklenirse,
terazinin yeniden denk olduu gzlenirse,
1 lahana +2 mandalina = 1 birim_ktle
olarak aklanabilir . Veya 1 lahana 1 birim_ktleden 2 mandalina daha eksik diyebilirizi.
1 lahana = 1 birim_ktle - 2 mandalina
Grld gibi, bir denklikte taraf deitiren byklk eksi iaret kazanyor. Genelletirirsek,
a b c + =
veya
a c b =
olarak yazabiliriz. Negatif iaretli terimlere dikkat edilmelidir. Aslnda
1 lahana = 1 birim_ktle - 2 mandalina
40
ifadesinin hibir gerek durmu yoktur. Bu sadece bir eylem yaplrsa olacak bir durumu
aklamaktadr. Gerekte terazi birim_ktleden yana eilmitir. Gerek durum,
1 lahana + 2 mandalina = 1 birim_ktle
eklindedir. Bu grnen bir fiziksel gerei aklamaktadr. Bunun iin, denklemlerde daima
pozitif iaret oluturacak dzenlemeler yapmak, fiziksel gereklilie daha yakn ifadelerin
olumasn salayacaktr.
Terazinin her iki tarafna ayn nesnelerin eklenip karlmas terazinin denkliini
deitirmez, bu ak. Fakat, arpma ve blme iin dikatli olmalyz. Terazinin denkliinin
bozulmamas iin her iki tarafn tm ayn deer ile blnp arplmaldr. rnek olarak,
terazinin lahann bulunduu tarafna iki yerine 1 mandalina konulursa terazinin denklii
bozulur ve
1 lahana
2 mandalina
2
+ 1 birim_ktle <
yani
1 lahana 1 mandalina + 1 birim_ktle <
olduu grlr. Denkliin salanabilmesi iin, tm terimlerin ikiye blnmesi gerekir.
1 lahana 2 mandalina +
2
1 birim_ktle
2
=
1
2
lahana 1 mandalina +
1
2
birim_ktle =
olarak dzenlenmesi gerekir. arpm yaplrken de ayn zenin gsterilmesi gerekir.
Denklemleri inceleren temel olay daima gznnde bulundurulmaldr. Temelinde
denklem veya denklik,
5 portakal = 5 portakal ( veya u veya bu ekilde edeeri)
41
olarak aklanr. Burada birimler basitleir ve
5 5 =
temel denklem ifadesi olartak kalr. Bu ifade,
5
5
1 =
olarak da belirtillebilir. Bu ifade,
5
2 3 + ( )
1 =
olarak da deitirilebilir. Her iki tarafn karesi alnabilir. Temel denklik deimez.
5
2
2 3 + ( )
2
1 =
5
2 3 + ( )

2
1 =
Daha yksek sler de alnabilir,
5
2 3 + ( )

5
1 =
5
5
2 3 + ( )
5
1 =
Genelde,
a a = ise a
n
a
n
=
olabilir. Fakat geri dnlmyee allrsa iler tam anlam ile sarpa sarar. nk,
Eer A A = ise A A = her zaman doru olmayabilir.
nk a a a
2
= = A ve A a = olabilecei gibi,
a ( ) a ( ) a
2
= = A ve A a = da olabilir.
Genel olarak (KARE KK ZELL):
Eer, p
2
d = ise p = +/- d olur.
42
p d =
Bunun sonucu, p = +/- d ve { olabilir.
p d =
denklemlerin dzenlenmesinde bu zelliin daima gznne alnmas gerekir.
leri ve geri gidilerde daima ayn ifadelere ulamak iin daima pozitif sonular gznne alnr.
Denklem zmleri :
Bir denklemin veya bir eitsizliin zm, denkleme yerletirilince denklemin denkliini, veya
eitsizlie konulunca, eitsizliin ynn salayan deere denilir. rnek
x
2
9 0 =
Bu ifade alnca, x 3 + ( ) x 3 ( ) 0 =
Bu denkliin geerli olmas iin ya x 3 + ( ) 0 =
elde edilir.
olmaldr. Bu durumda,
x 3 0 =
x 3 + 0 = veya x 3 ( ) 0 =
x 3 = x 3 =
Bu denklemin zm kmesi, x1= -3 ve x2= 3 olarak 2 eleman ierir. Bu elemanlarn her ikisi de
denklemin zmlerini oluturur.
Salama :
x
2
9 0 = x
2
9 = x = +/- 9
x 3 = x 3 =
3 ( )
2
9 0 = 0 0 = Birinci zm geerli
3
2
9 0 = 0 0 = kinci zm geerli
rnek : 3 y 1 + ( ) 4 y 5 = Denkleminin kkleri,
nce tm parantezler alr.
+ =
43
3 y 1 + 4 y 5 =
Sonra tm deiken terimleri tek tarafta toplanr :
3 y 4 y 5 1 =
Tm terimlerin iaretlerini deitirelim, yani her terimi -1 ile arpalm :
1 3 y ( ) 1 ( ) 4 y ( ) + 1 5 ( ) 1 ( ) 1 ( ) + =
3 y 4y + 5 1 + =
y 6 =
Salama :
3 6 ( ) 4 6 ( ) 6 =
5 1 6 =
6 6 =
6 6 + 0 =
0 0 = zm geerli !
rnek :
2 z 5 ( ) 4 z Eitsizliinin geerli olmas iin z deeri.
Eitsizliler aynen denklemler gibi allr. Sadece her taraf -1 ile arplrsa, eitsizliin yn
deiir.
2 z 10 4 z
2 z 4 z 10
2 z 10 z 5 ve z 5
Salama :
Eitsizliin geerli olmas iin z deeri -5 den byk bir deer olmal. Eer -4 deeri seilirse,
(hatrlayalm -4>-5 dir), eitsizliin salanp salanamadna bakalm:
44
2 4 5 ( ) 4 4 ( ) 18 16 veya 18 16
esitsizlik salanyor.
Lineer Denklemler
Lineer denklemler, a x +b = 0 eklinde olan denklemlerdir. Bu ekil
yazlm lineer denklemlerin standart yazlm eklidir. Fakat ou zaman lineer denklemlerin
yazlm bu standart ekle uymayabilir.
Lineer deklemlerde deiken olara belirtilen x yerine, z, t, gibi farkl semboller
kullanlabilir.
Liner denklemlerin zmleri iin denklemin denkliini etkilemeyecek dzenlemeler
yaplarak, denklemin denkliini salayacak zm deeri bulunur.
rnek :
3 x 5 + ( ) 2 6 x ( ) 2 x =
zm :
Parantezler alr. Deikenler tek tarafta toplanr ve zm salanr.
3 x 15 + 12 2 x 2 x =
3 x 15 + 12 4 x =
3 x 4 x + 12 15 =
7 x 27 =
x
27
7
=
Salama :
3
27
7
5 +
|

\
|
|

2 6
27
7

\
|
|

2
27
7

\
|
|

=
3 27
7
15 + 12
2 27
7
+
2 27
7
+ =
45
3 27
7
15 + 3.429 = 12
2 27
7
+
2 27
7
+ 3.429 =
zm geerli. zm : {x
| -27/7}
Yani zm, sadece bir elemanl ve elemann deeri -27/7 olan bir kmedir
rnek :
m 2
3
1 +
2 m
7
=
zm.: nce her iki taraf dzenleyeceiz. Sonra en kk ortak payda ile yazacak,
payday kaldracak ve zm bulacaz.
m 2
3
1
3
3
+
2 m
7
=
m 1 +
3
2 m
7
=
7 m 1 + ( ) 3 2 m ( ) =
7 m 7 + 6 m =
7 m 6 m 7 =
m 7 = zm : { m | -7 }
Salama :
7 ( ) 2
3
1 +
2 7 ( )
7
=
2 2 = zm geerli.
rnek :
5
2 y 6
10 y
y
2
6 y 9 +
=
46
zm
:
Eer
,
a
b
c
d
= ise
,
a d b c = yazlabilir.
5 y
2
6 y 9 + ( ) 10 y 2 y 6 ( ) =
5 y
2
30 y 45 + 20 y
2
60 y =
5 y 3 ( )
2
20 y y 3 ( ) = Bunu grmek son derece g
5 y 3 ( ) 20 y =
5 y 15 20 y =
15 y 15 =
y 1 =
5 y
2
30 y 45 + 20 y
2
60 y = Normal olarak devam edelim .
5 y
2
20 y
2
60 y + 30 y 45 + 0 =
15 y
2
30 y + 45 + 0 = 2 inci derece bir denkleme ulatk !!!
a x
2
b x + c + 0 = 2 inci derece denklemin kkleri :
b
2
b
2
4 a c
2
+
a

b
2
b
2
4 a c
2

\
|
|
|
|
|
|
|
|

= = =
47
a 15 = b 30 = c 45 =
b
2
b
2
4 a c
2
+
a
3 =
b
2
b
2
4 a c
2

a
1 =
Burada bir sorunla kar karyayz. Gerek bant, dzenlenince basitleen bir ekle
indirgeniyor. Fakat bunu kolay gremeyeceimiz iin, yapabileceimiz gibi devam ediyoruz.
Bu sefer de ikinci derece bir denklem bulunuyor ve iki tane zm elde ediliyor.
Tek deikenli lineer denlemlerin sadece bir tek zm olur. Burada iki zm
bulundu. Bunlardan birisi mutlaka yanltr. Acaba hangisi ? Bunu ancak salama ile
anlayabiliriz.
Salama
y 3 =
5
2 y 6
10 y
y
2
6 y 9 +
=
5
2 3 ( ) 6
5
0
= Sfra Blme , Anlamsz
!!!
y 3 = denklemin kk deildir. !!!!
y 1 =
5
2 y 6
0.625 =
10 y
y
2
6 y 9 +
0.625 =
Salama tamam. y=-1 denkemin tek zmdr. zm : { y | -1 }
SALAMANIN NE KADAR NEML OLDUU GRLYOR !!!!!!
48
rnek :
2 z
z 3 +
3
z 10
2 + = Denkleminn kklerinin bulunmas:
zm : zm srasnda olas klerden z=-3 ve z=+10 deerlerinden kanmak gerekir,
nk bu deerler 0 a blme yaratrlar bunlarn dnda her deer kk olabilir.
2 z
z 3 +
3
z 10
2
z 10 ( )
z 10 ( )
+ =
2 z
z 3 +
3 2 z 10 ( ) +
z 10
=
2 z
z 3 +
3 2 z + 20
z 10
=
2 z
z 3 +
2 z 17
z 10
=
2 z z 10 ( ) z 3 + ( ) 2 z 17 ( ) =
2 z
2
20 z 2 z
2
11 z 51 ( ) 0 =
51 9 z 0 =
9 z 51 =
z
51
9
= z
17
3
=
Salama :
z
17
3
=
49
2 z
z 3 +
3
z 10
2 + =
2 z
z 3 +
1.308 =
3
z 10
2 + 1.308 =
Salama tamam. !!! zm : { x | 17/3 }
rnek :
x
x 2 +
x
x
2
5 x + 6 +
=
lk nce hangi deerlerin zm olup olamayaca aratrlmal.
x
x 2 +
terimi iin x= -2 olursa sfra blme olur bu nedenle -2 zm olamaz.
x
x
2
5 x + 6 +
terimi iin payday sfr yapan deerler
x
2
5 x + 6 + denkleminin kkleridir.
a 1 = b 5 = c 6 =
b
2
b
2
4 a c
2
+
a
3 =
b
2
b
2
4 a c
2

a
2 =
Burada da -3 ve -2 deerlerinin kk olmayaca belirlenir.
Denklemin zm
:

50
x
x 2 +
x
x
2
5 x + 6 +
=
x
2
5 x + 6 + ( ) x x x 2 + ( ) =
x
2
5 x + 6 + ( ) x x x 2 + ( ) + 0 =
x x 2 + ( ) x 4 + ( ) 0 =
zmler : x 0 = x 2 = x 4 =
Bunlar arasnda x=-2 zm olamaz, x=0 ve x=-4 orijinal denklemde denendikten sonra,
zm olup olmadklar anlalabilir.
Salama : x 0 =
x
x 2 +
x
x
2
5 x + 6 +
=
0
0 2 +
0
0 0 + 6 +
=
0 0 = x 0 = zmdr.
x 4 =
x
x 2 +
x
x
2
5 x + 6 +
=
4
4 2 +
4
16 20 6 +
=
2 2 = x 4 = zmdr.
zmler kmesi : { x | 0, -4 }
51
rnek :
2
x 1 +
4
2 x
x 1 +
= Denkleminin zm
ncelikle, x=-1 deerinin zm olamayaca belirtelim. Bu deerle sfra a blme durumu
ortaya kar.
Denklemin zm :
2
x 1 +
4
x 1 + ( )
x 1 + ( )

2 x
x 1 +
=
2 4 x 1 + ( ) 2 x =
2 4 x 4 + 2 x =
2 4 2 x =
x 1 = Bu deer, denklemin zm olamayacandan,
DENKLEMN ZM YOKTUR:
ki Bilinmeyenli Lineer Sistemler
Bu denklemler,
a
1
y b
1
x + c
1
+ 0 =
a
2
y b
2
x + c
2
+ 0 =
eklindedir. zmleri iin, yntemlerden biri, denklemlerden birinden bir deikenin
deerini bulup dier denkleme yerletirmektir.
rnek :
5 y 3 x 22 =
4 y 2 x + 22 =
zm
nce ilk denklemden y deeri bulunur:
5 y 3 x 22 =

52
y
3 x
5
22
5
+ =
Sonra ikinci denkleme yerletirilir ve x
bulunur.
4 y 2 x 22 =
2 x
5
88
5
+ 22 =
x 99 =
Sonra y hesaplanr :
y
3 99 ( )
5
22
5
+ =
y 55 =
Salama : 5 55 ( ) 3 99 ( ) 22 = Deerler doru.
zm Kmesi : {x | -99 , -55}
Daima salama yaplmaldr.
kinci Derece Denklemler
kinci derece denklemler genel olarak
a x
2
b x + c + 0 =
eklindedir ve burada tek koul x deikeninin karesi olmas ve a 0 olmasdr.
Yerine gre b ve c sfr olabilirler. Bu denklemlerin alablir formlarnn zmleri ok kolaydr.
rnek : x
2
9 0 =
zm : Bu form,
Denkleminin zm :
53
olduu zaman elde edilir. zm son derece basittir
:
genel formuna, a = 1, b = 0 ve c = -9
a x
2
b x + c + 0 =
Burada, denklemi 0 yapacak deiken deerleri yani denklemin kkleri,nin bulunmas iin iki olaslk
bulunmaktadr.
x
2
9 x 3 ( ) x 3 + ( ) = 0 =
Bu denklemin sfra eit olabilmesi iin, ya
x 3 0 = olacak yani x 3 = olacaktr.
Veya
x 3 + 0 = olacak yani x 3 = olacaktr.
zm kmesi : {x | -3 , 3} olarak belirlenir.
En genel durum,
x
2
a
2
0 =
x
2
a
2
=
x a
2
=
Buraya kadar tamam. Fakat bundan sonra x in deeri ne olabilir ?
a
2
yle bir a deeridir ki a a a
2
= olsun .
lk bir olaslk olarak a ( ) a ( ) a
2
= olur. Bu durumda,
a
2
a = ve x a = olur. Tamam ama bu sadece
birinci olaslktr. Bir baka olaslk da olabilir.
kinci bir olaslk da a a a
2
= olabilir. Yani a
2
nin iki sonucu vardr. Bunlardan ikisinin
de saysal deerleri ayn fakat iaretleri deiiktir. Bu durumda, sonucun birisi a , dieri ise -a dr.
Yani,
54
a
a
2
= { a
a x a
2
= x = {
a
olabilir. Bu durumda,
x a
2
= ise , x = +/-
a olur
Zaten bu sonu aktr:
x
2
a
2
0 = denkleminin alm,
x a + ( ) x a ( ) 0 = eklindedir ve denklemin kkleri
x a = ve x a = olarak bulunur.
Denklemin zm kmesi { x | -a ,a } eklinde yazlablir.
zetle , eer p
2
t = ise ,
p = +/- t olur.
Buna, deerin karesi prensibi ad verilir.
rnek : x
2
486 0 = dekleminin zm,
x
2
486 =
x = +/- 486 486 22.045 =
x 22.045 = veya x 22.045 =
Denlemin zm kmesi , { x | -22.045 , 22.045 } olarak belirtilebilir.
55
rnek : a x b 0 = dekleminin genel zmn bulunuz.
zm :
a x b + ( ) a x b ( ) 0 =
x
b
a
= x
b
a
=
rnek : 4 m
2
1 0 = dekleminin kkleri:
Yukardaki zm yntemini uygularsak
x = + / -
1
4
olarak bulunur.
x
1
2
= veya x
1
2
=
Denklemin zm kmesi , { x | -0.5 , 0.5 } olarak belirtilebilir.
Alamayan ikinci derece denklemlerin kklerinin bulunmas yine ok kolaydr.
Denklem,
a x
2
b x + c + 0 =
eklindedir. Bu denklemin iki kk vardr. Bu kklerin ikisi de gerel veya her ikisi de sanal
olabilir. Fiziksel olarak, gerel kkler ile ilgileniriz.
Karenin Tamamlanmas
Genel a x
2
b x + c + 0 = denklemi ,
56
x
2
b
a
x +
c
a
+ 0 =
eklinde yazlabilir. Bu durumda katsaylar yeniden tanmlanrsa, genel ikinci derece denklem,
x
2
b x + c + 0 =
olarak yazlabilir. rnek olarak
12 x
2
7 x 23 0 =
x
2
7
12
x
23
12
0 =
7
12
0.583 =
23
12
1.917 =
x
2
0.583 x 1.917 0 =
x
2
0.583 x 1.917 =
haline gelebilir. Bu aamada, hatrlamamamz gereken bir alm var
:
a b + ( )
2
= a
2
2 a b + b
2
+
x b + ( )
2
= x
2
2 bx + b
2
+
imdi elimizde, x
2
b x + ksm var, sabit terimi de ekleyebilirsek,
kareyi tamamlam olabiliriz. Bu terimin deeri,
b
2
|

\
|
|

2
b
2
4
= deeridir
.
57
x
2
b x +
b
2
|

\
|
|

2
+ x
2
b x +
b
2
4
+ = x
b
2
+
|

\
|
|

2
=
x
b
2
+
|

\
|
|

2
=
b
2
4
b x + x
2
+
Bu ekilde, kare tamamlanm ve genel ikinci derece denklem ifadesinden, bileenlerine
ayrlabilen bir genel ikinci snf denklem modeli elde edilmi olur. Tm ikinici derece
denklemler, karenin tamamlanmas ile, bileenlerine ayrlabilen bir genel ikinci snf denklem
modeli oluturabilirler.
Saysal almamza devam edelim :
x
2
0.583 x 1.917 =
x
2
0.583 x b 0.583 =
b
2
|

\
|
|

2
0.583
2
|

\
|
|

2
= 0.085 =
x
2
0.583 x 0.085 + 1.917 0.085 + =
x
0.583
2

\
|
|

2
2.002 =
x
0.583
2
= + / - 2.002 2.002 1.415 =
lk kk :
x
0.583
2
1.415 =
58
x 1.415
0.583
2
+ =
x 1.707 =
kinci Kk :
x
0.583
2
1.415 =
x 1.415
0.583
2
+ =
x 1.123 =
olarak bulunur.
Salama :
Orijiinal Denklem :
12 x
2
7 x 23 0 =
x 1.123 =
12 x
2
7 x 23 5.452 10
3
=
x 1.707 =
12 x
2
7 x 23 0.017 =
Bu grdmz az ondalkla altmz iin karlatmz duyarlk hatalardr. Her iki sonu
da 0 olarak kabul edilebilir.
Denklemin Kkleri
59
1153
24
7
24
+ 1.706 =
1153
24
7
24
+
7
24
1153
24

\
|
|
|
|
|
7
24
1153
24
1.123 =
olarak bulunur yani saysal sonular dorulanr.
zm Kmesi { x | -1.123 , 1.706 } olarak belirlenir.
rnek :
2 x
2
6 x + 7 + 0 =
Bu denklemi, kanonik hale getirelim. Kanonik latince "Kanuni" demektir. Yani yasal, kabul edilmi
ekillere kanonik ekil ad verilir. Her ikinci derece denklem,
a x
2
b x + c + 0 =
cc
c
a
=
bb
b
a
=
dntrm ile kanonik hale dntrlebilir. kinci derece denklemlerin kanonik hali,
x
2
bb x + cc + 0 =
eklindedir. lk aamada sabit terimi sa tarafa ekelim,
x
2
bb x + cc =
kareyi tamamlamak iin, her tarafa
bb
2
|

\
|
|

2
bb
2
4
= ekleyelim :
x
2
bb x +
bb
2
4
+ cc
bb
2
4
+ =
Sol tarafn karesi tamamland .
60
x
bb
2
+
|

\
|
|

2
bb
2
4
cc =
x
bb
2
+
|

\
|
|

= + /-
bb
2
4
cc
x
bb
2
+
bb
2
4
cc = nce pozitif taraftan devam edelim,
Balang deerlerine geelim :
bb
b
a
= cc
c
a
=
x
b
a
|

\
|
|

2
4
c
a

b
a
|

\
|
|

2
=
x
b
2
4 a
2

c
a

\
|
|
|

b
2 a
=
x
b
2
a 4 a
2
c
4 a
3

b
2 a
=
x
b
2
4 a c
4 a
2

b
2 a
=
x
1
b b
2
4 a c +
2 a
= x
1
2 =
imdi de negatif taraftan devam edelim :
x
2
bb
2
+
bb
2
4
cc =
61
x
2
bb
2
bb
2
4
cc =
bb
b
a
= cc
c
a
=
x
2
bb
2
bb
2
4
cc =
x
2
b a
b
2
4 a
2
cc
a
2
+
2 a
=
x
2
b a
b
2
4 a c
a
2
+
2 a
|

\
|
|
|
|

=
x
2
b
a
a
b
2
4 a c +
2 a
|

\
|
|
|

=
x
2
b b
2
4 a c +
2 a
=
x
2
b b
2
4 a c
2 a
=
x
2
b b
2
4 a c
2 a
= x
2
3 =
62
Bu ifadeler tandk geldi mi ? Evet ezberlediimiz bantlar. Bu bantlar ikinci derece
denklemlerin kanonik (allagelmi) kk bantlardr. Burada
b
2
4 a c terimine, ikinci derece denklemin diskriminant ad verilir ve
ad ile tanmlanr. kinci derece bir denklemin gerel kklerinin bulunabilmesi iin,
diskriminantnn mutlaka pozitif saylar kmesinden bir deer olmas gerekir. nk , ancak
pozitif saylar kmesinin bir elemannn kare kk alndnda, pozitif ve negatif iki gerel say
elde edileblir. Eer ikinci derece bir denklemin diskriminant negatif bir say olursa, bu
denklemin ancak sanal kkleri olabilir. Eer diskriminant, (ayrdedici anlamna) sfr olursa,
denklemin tek ve akk bir kk vardr. Bu akk kk,
x
b b
2
4 a c +
2 a
= x
b 0 +
2 a
= x
b
2 a
=
zetle, ikinci derece bir denklemin kklerinin bulunmas iin kanonik bant,
x
b
2 a
= +/-
b
2
4 a c
2 a
eklindedir. Buradan,
x
1
b b
2
4 a c +
2 a
= x
2
b b
2
4 a c
2 a
=
Olarak analitik zm deerleri bulunur. (Analitik hesap yolu ile anlamndadr.)
Saysal deerlere geelim ve cebirsel bantlarn bulunabilmesi iin cebirsel yolu izleyelim::
2 x
2
6 x + 7 + 0 =
bb
6
2
= cc
7
2
=
63
kareyi tamamlayacak byklk,
bb
2
|

\
|
|

2
6
2
2
|

\
|
|
|

2
=
9
4
=
x
2
3 x +
9
4
+
7
2
9
4
+ =
bb
2
3
2
=
14 9 +
2
2.5 =
5
2
2
5
4
=
x
2
3
2
+
|

\
|
|

2
7
9
2
+
2
=
x
2
3
2
+
|

\
|
|

2
5
4
=
= +/-
5
4
x
3
2
+
x
1
3
2
+
5
4
= x
2
3
2
+
5
4
=
5
4
1
5
2
=
5
2
i =
5
4

5
2
i =
x
1
3
2
+
5
2
i = x
2
3
2
+
5
2
i =
x
2
3
2
5
2
i =
x
1
3
2
5
2
i + =
x
3
2
= +/-
5
2
i
Saysal zm ve bilgisayar program :
Program ba :
2 x
2
6 x + 7 + 0 =
a 2 = b 6 = c 7 =
64
b
2
4 a c = 20 =
x
1
b +
2 a
= x
2
b
2 a
=
x
1
1.5 1.118i + = x
2
1.5 1.118i =
Salamalar :
a x
1
( )
2
b x
1
+ c + 0 =
a x
2
( )
2
b x
2
+ c + 0 =
program sonu.
zm kmesi { x| 1.5 1.118i , 1.5 1.118i + }
Salama souncu beklendii gibi 0 olarak bulunuyor. nk, az sayda saysal ilem
yaplnca, tama hatas az olmaktadr.
Bu program dili allagelmi bir programlama dili deildir. Bu programlama dili cebirsel
ilemler yapabilen Mathcad 15 sembolik prosesrdr. Bu ekilde, ilemler insan hatas
yaplmadan gerekletirilebilmektedir.
Yeni saysal rnek :
3 x
2
2 x 1 0 =
a 3 = b 2 = c 1 =
b
2
4 a c = 16 =
x
1
b +
2 a
= x
2
b
2 a
=
x
1
1 = x
2
0.333 =
Salamalar :
a x
1
( )
2
b x
1
+ c + 0 =
( )
65
a x
2
( )
2
b x
2
+ c + 0 =
program sonu.
zm kmesi
{ x|
1
3
, 1 }
Bu ekilde istediimiz kadar problem zebiliriz.
kinci Dereceye ndirgenebilen Denklemler
nc derece denlemlerin de, ok kar olmakla birlikte, ikinci dereceye benzer
bir analitik zm yntemi var. Fakt drt ve zeri dereceli denklemlerin bir analitik
zm yntemleri yok. Bunun yerine, ok sayda gzel ve etkili saysal zm
yntemleri uygulanabilir. Baz yksek dereceli denlemlerin de cebirsel yntemlerle
ikinci dereceye indirgenip analitik zmlerinin bulunmas olana var.
Bu konuda ileri de greceimiz, deiken dntrm uygulanablir. rnek :
x
4
7 x
2
12 + 0 =
Burada,
x
2
( )
2
x
4
= olduu hatrlanrsa, u x
2
=
dntrm yaplrsa, x
4
u
2
= x
2
u = olur. Denklem u cinsinden
yazrsa,
u
2
7u 12 + 0 = Bu denklem ayrtrlabilir bir denklem olup,
u 4 ( ) u 3 ( ) 0 =
ekline dnr. Fakat bunu ilk bakta grmek kolay deildir. Bunun iin en iyisi genel yntem
uygulanr.
a 1 = b 7 = c 12 =
b
2
4 a c = 1 =
66
u
1
b +
2 a
= u
2
b
2 a
=
u
1
4 = u
2
3 =
Salamalar :
a u
1
( )
2
b u
1
+ c + 0 =
a u
2
( )
2
b u
2
+ c + 0 =
program sonu.
zm kmesi
{ u| 3 , 4 }
Bunan sonra tersine deiken dntrm yaparak x deerlerin bulmalyz.
x
2
u =
x = + / - u
x
1
u
1
= x
2
u
1
=
x
3
u
2
= x
4
u
2
=
Yani , drdnc derece denklem iin en ok drt analitik kk bulunabilir. Sonular,
beklentileri karlyor ( ilemlerin doru gidip gitmediinin anlalmas iin bu en nemli
noktalardan biridir).
Saysal deerler,
x
1
u
1
= x
1
2 = x
2
u
1
= x
2
2 =
x
3
u
2
= x
3
1.732 = x
4
u
2
= x
4
1.732 =
zm Kmesi : { x | -2 , -1.732 , 1.732 , 2 }
67
ok gzel, fakat her zaman byle ansl olmayz.
Radikal iinde terimler ieren denklemler
Bu konu ok dikkat edilmesi gereken bir konudur. Bu konuda daha nce de almalar
yapmtk. Burada da ayn almalara devam edecek ve kacak sorunlar birlikte inceleyeceiz.
rnek : x x 6 + = denklemini znz.
x a = ise x
2
a = yazlabilir. Faat bu konuda
ok dikat edilmesi gerek, nk denklem yeni bir ekil almtr ve her ne kadar iki yazm da
birbirine denk gibi grleblirse de,
x
2
a = dzenlemesi, x a = ya gre yeni zellikler kazanm
olabilir. rnek olarak, oriinal denklemde geerli olmayan yeni br kk gibi. Bunun iin,
dzenlenmi denklemlerdem elde edilen sonular daima orijinal denklemde denenmeli ve
ancak geerli olduklar anlalrsa doru sonu olarak kabul edilmeleri gerekir.
x
2
x 6 + =
x
2
x 6 0 =
Bu deklemin kklerini bulalm :
a 1 = b 1 = c 6 =
b
2
4 a c = 25 =
x
1
b +
2 a
= x
2
b
2 a
=
x
1
3 = x
2
2 =
Salamalar
:
x
1
x
1
6 + 0 = Tamam
68
Tamam Deil !!!
x
2
x
2
6 + 4 =
Yani -2 deeri, dzenlenmi denklemin geerli kk olmasna karsn orijinal denklemin
kk deildir. Bu konuda ok dikkatli olunmal ve ancak orijinal denklemde
denendikten sonra geerli olduu saptanan sonular, orijinal denklem iin geerli
olarak bildirilmelidir.
Orjinal denlemin kk kmesi {x | 3 }
rnek : y y 4 + 4 =
Burada deeri 4 olan sadece bir tane kk var.
zm : Karekk teriminden kurtulmak iin biraz dzenleme yapmak gerekir.
y 4 y 4 =
sadece taraflarn yerini deitirdik ve denklemin hala sadece deeri 4 olan bir tek kk
grnyor.
y 4 ( )
2
y 4 ( ) =
3
4
|

\
|
|

Her ik tarafn karesini aldk, bu son derece yasal, ama sonuta dzenlenmi denkleme, deeri 3
olan yeni bir yeni kk eklendi.
y 4 1 =
Her iki taraf (y-4) ile bldk, bu da gayet yasal, ama bu sefer, orijinal denklemde geerli
olan kk 4 ortadan kalkt, sadece, orijinal denklemde geerli olmayan 3 deeri tek kk
olarak grnyor.
Dzenlemeyi deitirelim :
y y 4 + 4 =
69
y 4 4 y = 4
4 y ( )
2
y
2
8 y 16 +
y 4 4 y ( )
2
=
4
5
|

\
|
|

y 4 y
2
8 y + 16 0 =
y 4 ( ) y 5 ( ) 0 =
Hi deilse, orijinal deklemde geerli olan kk deeri 4 bulunur olarak kalm. Dzenlemeler,
bazan ulalan bantnn orijinal bant ile ilgisinin kesilmesine neden olabilir. Uygun bir
bant bulana kadar dzenlemelerin denenmesi gerekir.
Mutlak Deer Denklemleri
Bir deerin mutlak deeri |p| eklinde yazlr ve mutlak deer p daima pozitif
bir deer olarak kabul edilir. Matematik olarak,
p p = Eer p 0 ise
p p = Eer p 0 < ise
rnek olarak , 7 7 = 56 56 = olarak alr.
34 8 272 = olur. nk ilem yaplren, mutlak deer iindeki
Fakat
terim poztif olarak kabul edilir. Aslnda deer zaten pozitifse, bunu mutlak deer olarak
belirtmek iin hi bir neden yoktur. Yani |6| = 6 olarak alr ve 6*6 = 36 , |6|*6 = 36 olur.
Mutlak deer sembol sylenecek szleri azaltmak iin kulanlr. rnek olarak
"stenilen deerler, y= -(16*x+34) deklemi ile y= (16*x+34) denkleminin zm ile
bulunur". Szckleri yerine,
"stenilen deerler, y = |16*x+34| bantsnn kkleridir. " ifadesi, sylenecek
70
"stenilen deerler, y = |16*x+34| bantsnn kkleridir. " ifadesi, sylenecek
szleri yar yarya azaltr.
Genel kural : Eer |p| = b ise ve b>0 ise, p=b veya p = -b olabilir.
rnek : 2 x 5 9 =
zm : Burada zm iki ayr denklemin zmdr:
2 x 5 9 = 2 x 5 ( ) 9 =
2 x 5 + 9 =
x = 7
x = 2
Yani, iki ayr denklem ve iki ayr sonu, sz konusudur.
rnek : 28 x 9 0 =
28 x 9 0 = 28 x 9 ( ) 0 =
x =
9
28
28 x 9 + 0 =
x
9
28
=
akk kk !!
rnek :
4 x 6 + 5 =
Olumsuz, zm yok, hibir mutlak deerin sonucu eksi bir deer olamaz!!!!!
rnek : x 2 3 x 1 + =
zm :
Bu konuda dikkatli olmalyz, nk hibir mutlak deer teriminin sonucu eksi olamaz.
O zaman ilk yaplacak ey zmn geerlik aralnn saptanmasdr.
+
71
3 x 1 + 0
3 x 1
x
1
3
veya
1
3
x veya
1
3
x
yani , [
1
3
x < )
Sonra, sras ile denklemin olas kkleri bulunur:
x 2 3 x 1 + = x 2 ( ) 3 x 1 + =
3 x x + 2 1 + = x 2 + 3 x 1 + =
4 x 1 =
2 x 3 =
x
3
2
= x
1
4
=
Bu kklerden sadece x
1
4
= zmn geerlilik alan iinde olduundan
{ x | 1/4 }
geerli zm olarak kabul edilebilir.
Not : 3
2
1.5 =
1
3
0.333 = olduundan ve
1.5 0.333 < olduundan, -1.5 x deikeninin geerlik alan iinde deildir.
Geerlilik alan iinde olmas iin ya x= -0.333 veya daha byk bir deerde olmas
gerekir.
Yer Belirleme
stnde yaadmz dnyada, bir nesnenin yerinin herkes tarafndan sadece bir tek
yer anlalacak ekilde belrtilmesi yaamsal nem tar. Her trl tamaclk, kadastro, snrlan
belirtilmesi ve daha saylamayacak kadar ok ilem iin bir yer belirleme sistemine gereksinim
vardr.
72
nsanlarn kullanabilecei bir yer belirleme sistemi nasl olmaldr? Hereyden nce
insanlar snrl sayda boyutu geometrik olarak alglayabilirler.
Bir kibrit kutusu dnelim. Bu kutunun ykseklii, eni ve boyu alglanabilir. Daha baka
bir boyut dnlemez. En, boy ve ykseklik, insanlar sadece bu boyutu alglayabilirler. Bunun
iin yaadmz dnyaya, " Boyutlu (3D) Dnya" ad verilir.
1
2
3
1
2
3
0
2
4
6
8
Yaadmz dnya bir portakal gibi alglanabilir.
Bu portakaln tam orta noktasna "Orijin" ad verilir ve orijinden bir dey , iki de
birbirine dikey yatay eksen geirilir. Tm eksenler eit paralara blnr. Bu koordinatlarda,
st yarmkrede bulunan dey koordinat deerleri pozitif, alt yarmkrede bulunan dey
koordinat deerleri negatif olarak kabul edilir. Sa tarafta olan yatay koordinat deerleri
73
koordinat deerleri negatif olarak kabul edilir. Sa tarafta olan yatay koordinat deerleri
pozitif, sol tarafta olan yatay koordinat deerleri negatif olarak kabul edilir. Bylece tm
dnya iindeki noktalar, belirtilmi olur.
boyutlu ("Kresel veya 3D" koordinat sistemi ok yararl olmasna karn dnyada
ou iler sadece iki boyut kullanlarak yrtlebilir.
imdi portakal yardan kesip, bir kat zerine bastralm. Greceimiz ekil
aadaki gibi olacaktr:
-1.0 -0.5 0.5 1.0
x
-1.0
-0.5
0.5
1.0
y
Burada grdmz portakaln yatay kesitidir. Tam ortada, yani 0 noktasnda orijin
bulunmaktadr. Orijinden geen bir yatay eksen (x ekseni) bir de dikey eksen (y ekseni)
vardr. Bu bir yzey olduundan dey ykseklik ekseni (z ekseni) yoktur. Her iki eksen
birbirine diktir. Eksenler eit birimlik paralara blnmtr ve orjinin sanda bulunan
ksmdaki birimler + , solundaki birimler ise - olarak deerlendirilir.
Dzlemsel koordinat sistemi, 1637 de Fransz matematiki ve filozof Ren Descartes
tarafndan bulunmu ve kendisinin Latince ad olan, "Renatus Cartosius" dan dolay Kartesiyen
Koordinat Sistemi ad verilmitir. Daha sonralar bu sisteme bir de ykseklik ekseni z eklenerek,
gnmzdeki kresel koordinat sistemi oluturulmutur.
74
Kartezyen koordinat sistemi, temelde iki boyutludur. Eksenlerin birimleri istenildii kadar
fazla olablirler. Yani kartezyen koordinatlarn grntleyemeyecei koordinat deerleri yoktur.
Dzlemsel kartezyen koordinat sisteminde her nokta, {x ,y} eklinde, iki elemanl bir kme
ile ifade edilir. rnek olarak {3,5} noktas, x ekseninde saa doru 3 birim, y ekseninde yukar
doru 5 birim olacak ekilde gsterilir.
Kartezyen grafk yntemleri, matematik bantlarn grafik olarak grntlenmesi
amac ile ile kullanlr. Bu konudakii bilim dal "Analitik Geometri" olarak adlandrlr.
rnek olarak 4 nokta oluturalm. Bunlarn koordinatler, A noktas iin x = 3 , y = 5
olsun. Bu koordinatlar A(3,5) olarak gsterlir. A noktann addr, Parantez iinde noktann
koordinatlar gsterilir. Konvansyonel ( allagelmi, ortak kabul grm) ynteme gre
nce x, sonra y, sonra varsa z koordnatlar belirtilir. Bu konvansiyona gre dier noktalar,
B(-3,5), C(3,-5) ve D(-3,-5) olsun. Bu noktalarn 2 boyutlu uzayda kartezyen koordinatlarda
gsterimi aada grlmektedir.
75
boyutlu ortamda da noktalar aada grld gibi grntlenebilir. Sadece,
bu biraz daha zordur ve sonularn alglanmas daha gtr.
Yukarda grdmz 2 boyutlu yzey grafikleri, bu boyutlu dikdrtgen prizmann
sadece x ve y eksenlerini ieren yatay kesitidir.
Peki insan zekas sadece 3 boyutlu verilerle snrl mdr? Szkonusu bile deil! nsan
zekasnn snrlar ok genitir. Matematikte 3 den ok daha fazla boyutta matrislerle allabilir.
Sadece veriler geometrik olarak alglanamaz.Yoksa hesaplar iin bir engel yok. Bilgisayarlar bu
iler iin gelitirilmitir. (Sadece internetten ark indirmek iin deil !!!)
Artk verilerimizi hem 2 hem 3 boyutlu ortama nasl grntleyeceimizi bildiimize
gre, sadece bamsz noktalar deil, birbirleri arasnda bir iliki olan noktalar, yani
fonksiyonlar nasl grntleyebileceimizi dnebiliriz.
Fonksiyonlar
Fonksiyonlarn temeli denklemlerdir. Bir fonsiyon, bir denklemi salayan
deerlerden oluan bir kmedir.
rnek olarak y = 3*x +5 eklinde lineer bir fonksiyon ele alalm. lineer fonksiyonlar
bamsz deiken x 'in ssnn 1 olmasndan dolay , "Birinci Derece Denklemler" ad
verilen ilikiden kaynaklanmaktadr.
y 3 x 5 + =
76
likisine baklnca, x deerlerinin istenldii gibi, hibir kurala bal olmadan
seilebilecei grlr. Bunun iin ,x deikeninine bamsz deiken ad verilir.
rnek olarak, x= -2, 9, -125.88 deerlerini alabilir. Bunun iin hibir engel yok, nk
x bamsz bir deikendir.
Dier taraftan, y deikeninin serbeste seilmesi sz konusu deil, nk
fonksiyonun gerekleemsi deerinin x e bal olarak hesaplanmas gerekir. Bunun iin y
deikenine "Baml Deiken" ad verilir.
Fonksiyonlarn belirtilmesi iin, iki trl konvansiyon kullanlabilir. Bunlardan ilki, klasik
y f x ( ) =
eklinde belirtmektir. Burada y nin baml deiken olduu, deerinin sadece x adl tek
bamsz deikenin deerine gre ve tanmlanm fonksiyonel iliki f() ifadesine gre
hesaplanaca belirtilir. Bu liki de sadece bamsz deiken ( ksaca deiken olarak belirtilir)
says aklanabilir. Deikenin derecesi belirtilemez. rnek olarak,
y 4 x 7 = ve y 45 x
2
=
y f x ( ) = fonksiyonu olarak gsterilir. nk bunlarn her ikisi de tek deikenli
fonksiyonlardr. Ve bu konvansiyonun belirtebildii tek zellik budur.
Bamsz deiiken says birden fazla olan fonksiyonlar da bu konvansiyonla
z f x y , ( ) =
olarak belirtilebilirler. Burada iki bamsz deiken olan x ve nin serbeste seilebilen deerleri
ile z deikeninin deerinin hesaplanabilecei belirtilir. rnek olarak,
t 12 p 7 q + = fonksiyonu, t = f(p,q) olarak belirtilebilir.
Bir fonksiyonun derecesi ile deiken says birbirinden farkl kavramlardr. rnek olarak
y 12 x
2
= ikinci dereceden fakat tek deikenli bir fonksiyondur. Oysa,
z 56 x 22 y + 9 = birinci dereceden, iki deikenli bir fonksiyondur. nk, ss
birden byk hibir deikeni yoktur.
Fonksiyon tanm iin bir baka konvansiyon, f(x,y) eklinde bir tanmdr. Bu tanm , tek
deikenli bir fonksiyonu aklar. Bu fonksiyonun kapal eklidir ve bu fonksiyonda iki deiken
olduunu birinin baml, dierinin bamsz olduunu aklar. Fonksiyonun ak ekli,.
f x ( ) sin x ( ) =
olarak verilebilir.Bu tanmda bamsz deiken bulunmuyor, nk zaten bamsz deiken
fonksiyonun deeridir. Yani f(x) notasyonu, y yerine kullanlan gterili bir deyimden baka ey
deildir ve ayn ekilde g(x), h(x) gibi notasyonlar da kulanlabilir. Bunlarn hepsi y anlamndadr.
Bu ikinci tanm, daha ak, daha komprime, daha dinamik bir tanmdr ve yaygn olarak kullanlr.
77
ZGLER :
Dz bir izginin kapal fonksiyonu aadaki gibi tanmlanr:
f x y , ( ) A x B y + C + =
Burada A, B, C izginin niteliine bal olarak saptanan sabit saylardr. Bunlara, fonksiyonun
parametreleri ad verilir.
Kapal denklemler, daha zet bilgi verebilmelerine karn pratikte fazla yararl deildir.
Denklemin baml deikeni konvansiyonel olarak y , bamz deiken de x olarak seilirse,
f x y , ( ) y =
A
B
x C =
Parametreler yeniden tanmlanrsa,
f x y , ( ) y = mx n + =
Bu fonkiyon ak ekli ile,
f x ( ) mx n + = veya y mx n + = olarak belirtilebilir.
Bir doru nasl izilebilir ?
Ya parametreleri bilirsiniz. Bamsz deiken deerlerini serbeste seer baml
deiken deerlerini hesaplarsnz. Karetezyen koordinatlarda x lere kar gelen y
noktalarn yerletirir, bu noktalar birletirir. Doruyu izersiniz.
Ya da noktalar bilirsiniz ve bu noktalarn kartezyen koordinatlarndan yararlanarak
parametreleri hesaplarsnz. Parametreler hesaplannca doru da izilmi olur.
Ya da elinizdeki noktalar kartezyen koordinatlarda yerletirir. Noktalar birletirir.
izgiyi ekersiniz. Tabii bu durumda, analitik bir bant filan bulamazsnz. Bu banty
bulmak iin grafik zerinden almaya devam etmek gerekir.
lk olarak parametreleri bildiimizi dnelim: rnek olarak m=0, n = 16 olsun
f x ( ) 0 x 16 + =
f x ( ) 16 = (ayn ekilde f(0) olarak da belirtilebir, yani f(0)=16 )
Enteresan bir durum. Bamsz deiken ortadan kalkm, x deeri ne olursa olsun f(x) daima
ayn deeri alyor.
Bu durum ,
f x ( ) Sabit =
zel halidir ve izimi, x eksenine paralel bir izgidir.
.. =
78
x 10 10 .. =
f x ( ) 16 = Saysal rneinin izimi,
10 5 0 5 10
0
10
20
30
40
50
f x ( )
x
Baka bir zel durumu inceleyelim :
Eer n = 0 olursa ,
f x ( ) m x =
olur. Burada olaslk olabilir:
Eer m =0 olursa,
o zaman f x ( ) 0 = olur. Fonksiyon ortadan kalkar ve tm deerler sfr noktasn yani
orijini verir.
79
Eer m = pozitif saylar kmesinden bir deer olursa ,
(Yani bu kst, m= 0 olma olasln ortadan kaldrr)
O zaman x deikeni fonksiyonun tanml olduu tm kapsam aralnda deerler alabilir.
x < < arasdr. Bu durumda , x deikeninin
Lineer bir fonksiyonun kapsam alan
alabilecei deerler arasnda, 0 deeri de vardr ve bu durumda,
f x ( ) 0 m = 0 =
olur. Yani eer, m 0 ( ) ve n 0 = ise, fonksiyon daima orijinden
geer.
imdi deerleri inceleyelim :
x x
1
= iin ordinat deeri, f x
1
( ) olarak hesaplanr.
imdi ikinci bir deer verebiliriz.
Eer, ikinci deer, birinciden bykse yani,
n 0 = ve m 0 > olduunda,
eer
x
2
x
1
> olursa f x
2
( ) f x
1
( ) > olur.
Yani fonksiyon, artan bir fonksiyon olur. rnek
f x ( ) 2 x =
80
10 5 0 5 10
20
10
0
10
20
f x ( )
x
Artan bir fonksiyonun grafii yukar doru ykseli gsterir. Bunu anmsamak iin "Uluda'a
k" diyebiliriz. Fakat iin bir de saysal ksm var.
Acaba ykseli ne kadar keskin ? Matematikte hereyin bir lt var. Bunun iin de bir
keskinlik lt oluturulmu.
eim =
YukarDogruHareket
IleriDogruHareket
ki nokta arasnda erinin (Matemetikte hereye eri denir. Bu eri, yzde yz bir doru da olsa
farketmez) yryn izlemek iin iki nokta alalm :
x
1
5 = f x
1
( ) 2 5 ( ) = 10 =
x
2
5 = f x
2
( ) 10 =
lleri Doru Hareket =
Yukar doru Hareket =
x
2
x
1
10 =
egim
f x
2
( ) f x
1
( )
x
2
x
1

= f x
2
( ) f x
1
( ) 20 =
81
egim
20
10
= egim 2 =
Birinci nokta daha byk, ikinci nokta daha kk olsa bile sonu deimez. Ykselen bir erinn
eimi, ne yaparsak yapalm pozitif kar.
Matematik aan eimin deerini saptamaya alalm:
Eim :
m x
2
n + ( ) m x
1
n + ( )
x
2
x
1

=
m x
2
m x
1
n + n
x
2
x
1

x
2
x
1
( ) m
x
2
x
1
( )
Eim = m
=
Bir dorunun eimi, doru denkleminin m parametresine eittir.
Bir negatif emli azalan bir fonksiyon izleyelim
:
f x ( ) 3 x =
10 5 0 5 10
40
20
0
20
40
f x ( )
x
x 10 9.999 , 10 .. =
imdi n parametresinin ne anlama geldiini saptamaya alalm:
Doru denklemi, f(x) = m * x + n olduuna gre
Eer x=0 olursa, f(0) = n olur. Burada n deeri, dorunun y eksenini (ordinat) kestii yerdir.
82
Tm parametreleri olan tam bir doru denkleminin izimini izleyelim:
f x ( ) 3.5627 x 15.27924 =
10 5 0 5 10
40
20
0
20
40
f x ( )
x
imdi, elimizde veri noktalar var iken doruyu nasl izebileceimize bakalm:
stersek veri notalarn kartezyen koordinatlara yerletirir, noktalar bir cetvel ile
birletiririz. Ama, daha iyi bir yntem var.
Doru denklemi, y = m*x +n eklindedir. Burada bilinmeyenn iki deiken m ve m
deerleridir. Eer, iki bamsz denklem bulabilirsek, iki bilinmeyenli iki denklemden oluan
bir lineer denklem sistemi oluturur ve m ile n deerlerini hesaplayabiliriz.
Elimizde deneysel veri noktalar var ve her nokta bamsz olarak doru denklemini
salar. (Not: Deneysel veri noktalarnn tek ayn ve doru zerinde olduunu, hi kimse
garanti edemez. kinci, nc derece, zellikle stel ilikier de olabilir. Burada varsayalm ki
noktalar ayn doru zerinde olsun, yani iliki lineer ksn).
Elimizdeki noktalar, x
1
2 = y
1
13 =
x
2
6 = y
2
32 =
y
1
m x
1
n + =
iki bilinmeyenli (m ve n) denklem sistemi
y
2
m x
2
n + =
83
ilk denklemden n deikenini ekelim :
y
1
m x
1
n =
kinci denklemde yerine yerletirelim :
y
2
m x
2
y
1
m x
1
( ) + =
Tek bilinmeyenli (m) tek bir denklem kald. Dzenleyelim
y
2
y
1
m x
2
x
1
( ) =
m
y
1
y
2

x
1
x
2

= m 4.75 =
Buradan n deerini de hesaplayalm :
n y
1
m x
1
=
n 3.5 =
Doru denklemi
:
f x ( ) 4.75 x 3.5 + = x 10 9.999 , 10 .. =
10 5 0 5 10
40
20
0
20
40
f x ( )
x
Salama : f 2 ( ) 13 = f 6 ( ) 32 = Sonular doruland.
84
Buraya kadar, en ok karlalan dorusal fonksiyonlarn incelemesini yaptk. Bunun
amac fazla kuramsal bilgilere gmlmeden, pratik adan bilgi ve deneyim kazabilmekti. Fakat,
yava yava salam kuramsal temeller zerinden bilgilerimizi biriktirmeliyiz.
Fonksiyonlarn Pratik Adan Tanm
"Fonksiyonlar denklemlerdir. Bamsz deiken veya bamsz deikenlerin tek bir deeri,
fonksiyonun sonucunda tek bir deer oluturursa, bu denklem bir fonksiyon olarak
tanmlanabilir."
Bu tanm, pratik olmasna karn salam bir tanmdr, fakat deneysel bir yntemdir. Yani,
deerlerin yerletrilmesi ve sonucunun incelenmesi yntemine dayanr. Hereyden nce bir
denklemin fonksiyon olup olmadnn denenmesi iin tm gerel saylarn kullanlabilmsesi
olana yoktur.
Gerel saylar kmesinin eleman says ok genitir. Art sonsuzdan, eksi sonsuza kadar
says bile insan dncesinin snrlarn aan sayda eleman ierebilir. Yine de, 2 gibi saylarn
gerel saylar kmesinde karl yoktur. Baz deerler de sfra blme gibi tanmsz sonular
verebilir. Bunun iin, belirli sayda deer kullanp sonular olumlu olursa, denklemin fonksiyon
olduu dnlebilir. Olumsuz sonu, denklemin kesinlikle bir fonksiyon olamayacan ortaya
karr.
rnek :
x
1
0 =
y 4 x 23 + =
f x ( ) 4 x 23 + =
Fazla deneme gerei yok nk daima tek deer kar. Bu bir lineer (dorusal) foksiyondur.
Yntemi tamamlamak iin ilikinin grafii izilmeli ve belirli bir x deerine dik izilip,
fonksiyonun bu diki tek noktada kestiine emin olunmaldr.
f x
1
( ) 4 0 23 + = f x
1
( ) 23 =
x 10 9.9999 , 10 .. =
10 5 0 5 10
20
0
20
40
60
80
f x ( )
x
85
Grafikten, tm dikmelerin sadece bir noktada grafii kestii grlyor. Bu ilikinin snand
deerler arasnda fonksiyon olarak kabul edilebilecei sylenebilir.
rnek : y 4 x
2
=
En iyisi grafi izip sonucuna bakalm :
f x ( ) 4 x
2
=
10 5 0 5 10
0
100
200
300
400
f x ( )
x
Kesinlikle bir fonksion.
rnek : y
2
5 x 3 =
zm: Bu sefer durum sakat,
p
2
d = p =
d
d
|

\
|
|

x 2 =
y
2
5 2 3 = y
2
7 =
y 7 = veya y 7 =
Bamsz deikenin tek deeri, denklemin iki tane sonucunu veriyor. Kesinlikle fonksiyon
deil.
86
rnek :
y
2
x
2
+ 4 =
Bunun da bir fonksiyon olamayaca ak. izimine bir bakalm :
y 4 x
2
= f x ( ) 4 x
2
=
g x ( ) 4 x
2
=
x 3 2.99 , 3 .. =
2 1 0 1 2
2
1
0
1
2
f x ( )
g x ( )
x
Bu bir ember denklemi, emberler asla fonsiyon olamazlar. Ama yarm emberler farkl tabii.
Tanm Alan ve Kapsam Alan
Bir fonksiyonun tanm alan, fonksiyonun anlaml br tanm olmas iin, bamsz
deiken (x) kapsam alandr. (Domain) . Bunun iin Trke Domen demek daha belirgin
olacaktr.
Bir fonksiyonun tanm alan, baml deikenin kapsam alandr. (Range)
Fonksiyon Deerlendirmeleri
rnek : f x ( ) x
2
2 x 8 + = g x ( ) x 6 + =
a - f(3) ve g(3) deerlerini bulunuz.
Tek yaplacak ey, x yerine 3 koymak ve hesaplamak.

f x ( ) 3
2
2 3 8 + = f x ( ) 9 6 8 + = f x ( ) 11 =
g x ( ) 3 6 + = g x ( ) 9 = g x ( ) 3 = veya g x ( ) 3 =
87
p x ( ) t
2
= p x ( ) = +/- t
b - g 10 ( )
g x ( ) 10 6 + = g x ( ) 4 = Bu bir gerel say deil ve
g x ( ) fonksiyonun domeni -10 deerini iermiyor.
En iyisi bir kapsam aral saptamas yapalm :
x 6 + 0 > x 6 > x 6 < 6 x <
Fonksiyonun domeni, [-6 , )
c - f 0 ( ) Dikkat !
f 0 ( ) 0
2
2 0 8 + = f 0 ( ) 8 = Yanlp da sfr sanmayalm !!!
d - f t ( ) t
2
2 t 8 + = f x 1 + ( ) ve f t 1 + ( ) 'i
hesaplaynz.
zm : Burada, x yerine x+1 ve t yerinde de t+1 yerletireceiz.
f t ( ) t 1 + ( )
2
2 t 1 + ( ) 8 + = f t ( ) t
2
2 t + 1 + 2 t 2 8 + =
f t ( ) t
2
7 = f t ( ) t 7 ( ) t 7 + ( ) =
f(x) de ayn
!!.
rnek : 3 t
2
4 + eer t 4 ise
g t ( ) = 10 eer 4 t < 15 is
e
1 6 t eer t 15 > ise
g 6 ( ) g 4 ( ) deerlerini
hasaplaynz.
zm : Bu aslnda paral bir fonksiyon, ama tek bir fonksiyon. Sadece, farkl
88
zm : Bu aslnda paral bir fonksiyon, ama tek bir fonksiyon. Sadece, farkl
domen alanlarnda, farkl fonksiyonel ilikiler var.
6 Hangi Domende ? 6 4 < O zaman uygulanacak fonksiyonel iliki
g x ( ) 3 t
2
4 + =
g x ( ) 3 6 ( )
2
4 + = g x ( ) 108 4 + = g x ( ) 112 =
4 (-4 her iki domende de var, fakat sadece en st domen, t= -4 deerini
kapsyor.
g 4 ( ) 3 4 ( )
2
4 + = g 4 ( ) 3 16 4 + = g 4 ( ) 52 =
Fonksiyonlarn Birbirlerine Eklenmeleri , karlmalar , arplmalar ve Blnmeleri
Fonksiyonlar cebirsel ifadelerdir. Bunlar ile her trl ilem yaplabilir. rnek
olarak,
3 x 4 + ( ) 5 x
2
7 ( ) + ileminin sonucu, 5 x
2
3 x + 3 olur.
Yani,
eer
f x ( ) 3 x 4 + ( ) = ve g x ( ) 5 x
2
7 = ise
,
f x ( ) g x ( ) + 5 x
2
3 x + 3 = olur. Bu ilem,
f x ( ) g x ( ) + = (f+g)(x) veya basite f+g olarak belirtilir.
Ayn ekilde, f(x) - g(x) = (f-g)(x) = f - g = 3 x 4 + ( ) 5 x
2
7 ( ) 3 x 5 x
2
11 + =
olarak
hesaplanr.
Fonksiyonlarn birbirleri ile arplmas, f(x) g(x) = (fg)(x) = f g =
3 x 4 + ( ) 5 x
2
7 ( )
buradan fg = 15 x
3
20 x
2
+ 21 x 28 olarak bulunur
Fonksiyonlarn birbirleri ile blnmesi de ayn ekilde yrtlr.
f x ( )
g x ( )
=
f
g
(x) =
f
g
=
3 x 4 + ( )
5 x
2
7 ( )
Sonular, bamsz deiken cinsinden ifade edilebilir.
89
f
g
0 ( )
4
7
=
Fonksiyon Yaplandrlmas
Fonksiyonlarn birletirilmesi i in baka bir yntem, fonksiyon yaplandrlmas (
fonksyon kompozisyon) ad verilen bir yntemdir. Fonksiyonlarn yeniden yaplandrlmasnn
amac, karmak bir yapdan daha kolay bir yapya geebilmektir.
Fonksiyon yaplandrlmasnn tanm :
Bir f(x) fonksiyonu ile bir g(x) fonksionun yaplandrlmas (burada sra nemlidir !)
fg ( ) x ( ) = f[g(x)]
Tam aksi, yani nce g sonra f gelirse,
gf ( ) x ( ) = g[f(x)]
Bu konuda dikkat edilmesi gereken nokta bu ilemin sraya baml bir ilem olduudur.
kinci bir nokta ise, bu ilemin ASLA BR FONKSYON ARPIMI OLMADIIDIR. Bu ilem sadece bir
fonksiyon deerlendirilmesi ilemidir.
Fonksiyon yaplandrlmas ilem youn bir almadr ve gnmzde bu almalar artk
insanlar deil, bilgisayarlar (sembolik prosesrler) gerekletirir.
rnek olarak , f x ( ) 2 3 x + x
2
= g x ( ) 2 x 1 =
fg(x) ve fg ( ) x ( ) deerlerini hesaplayn aradaki fark grn.
f g = f x ( ) g x ( ) = 2 3 x + x
2
( ) 2 x 1 ( ) = 7 x
2
2 x
3
x + 2
ve ilem sras burada nemli deildir.
fg ( ) x ( ) f g x ( ) ( ) f 2 x 1 ( ) = Son derece basit !
=
ve
f x ( ) 2 3 x + x
2
= olduuna gre,
f 2 x 1 ( ) = 2 3 2 x 1 ( ) + 2 x 1 ( )
2
= 10 x 4 x
2
2
Esas soru, F x ( ) 10 x 4 x
2
2 =
ise
,
yle iki f(x) ve g(x) fonksiyonu bulunuz
ki F x ( ) = fg ( ) x ( ) olarak aklansn . Bu baya bir sorun olurdu ve
90
geni sembolik prosesr kapasitesi olmazsa zm olana olmayabilirdi.
Bu arada, f g ileminin sonucu farkl olacaktr.
f g = 2 3 x + x
2
( ) 2 x 1 ( ) = 7 x
2
2 x
3
x + 2
Invers (Ters) Fonksionlar
Tanm:
Bir f(x) ve g(x) fonksiyonu verilmi olsun, g(x) fonksiyonunun f(x)
fonksiyonunun inversi veya f(x) fonksiyonun g(x) fonksiyonun inversi olabilmesi iin,
fg x ( ) gf x ( ) = x = olmas gerekir
Eer bir g(x) fonksiyonu, bir f(x) fonksiyonun inversi ise,
g x ( ) f
1
x ( ) =
olarak yazlr. Eer bir f(x) fonksiyonu, bir g(x) fonksiyonunun inversi ise,
f x ( ) g
1
x ( ) =
olarak yazlr.
Inverse foksiyonlarla alldnda, f
1
x ( ) ifadesinin kesinlikle
1
f x ( )
olmad hatrlanmaldr.
f x ( ) 3 x 2 = olduuna gre, f
1
x ( ) fonksiyonunu bulunuz.
rnek :
zm : lk olarak f(x) yerine y kullanalm :
y 3 x 2 =
sonra x ve y lerin yerlerini deitirelim x 3 y 2 =
sonra y cinsinden zelim
:
y
x
3
2
3
+ =
Bylece inverse fonsiyon bulunmu olur.
f
1
x ( )
x
3
2
3
+ =
91
Salama : f f
1
x ( )

=
f
x
3
2
3
+
|

\
|
|

3
x
3
2
3
+
|

\
|
|

2 x = Salama tamam.
rnek : h x ( )
x 4 +
2 x 5
= h
1
x ( ) ifadesini bulunuz.
x
y 4 +
2 y 5
=
y
5 x 4 +
2 x 1
=
h
1
x ( )
5 x 4 +
2 x 1
=
Salama :
h h
1
x ( )

h
5 x 4 +
2 x 1
|

\
|
|

=
5 x 4 +
2 x 1
4 +
2
5 x 4 +
2 x 1
5
= x = (O.K.)
Bilgisayar olmadan bu ie girimek akl sal iin zararl olabilir.
Bir fonksiyon ile inversinin grafikleri arasnda ilgin bir iliki vardr. Bu grafikler y=x ekseninin
iki tarafnda birbirinin aksidir.
f x ( ) 3 x 2 = g x ( )
x
3
2
3
+ = s x ( ) x =
4 2 0 2 4
3
2
1
0
1
2
3
f x ( )
s x ( )
g x ( )
x
92
f x ( ) e
x
= s x ( ) x =
4 2 0 2 4
3
2
1
0
1
2
3
f x ( )
s x ( )
g x ( )
x
g x ( ) log x ( ) =
ok Rastlanlan Fonksiyonlarn izimleri
Buraya kadar yaptmz almalarda, dorularn ve emberlerin izimlerini incelemitik.
Burada ilk olarak paral fonksiyonlarn izimi ile incelemelerimize devam edeceiz.
Paral fonksiyonlar, tek fonksiyonlar olmakla beraber, fonksiyonel iliki farkl
domenlerde farkl olur. Bu yzden domen sonu ve balarnda, erinin iziminde atlamalar
grlr. Her domen farkl bir fonksiyonel iliki ve bunun sonucu olarak da farkl bir izime
neden olur.
rnek olarak, Br f(x) fonsiyonu x<2 olduunda , 2 x x>2 olduunda ise, 2 x 3 olarak
tanmlanyor. Bu erinin izimi aada grlmektedir. Fonksiyonun x=2 noktasnda bir
sreksizlii vardr. Bu atlama noktalar bir doru ile birletirilmemelidir. Atlama noktalar ii bo
noktalar halinde gsterilebilir.
f x ( ) 2 x x 2 < if
2 x 3 x 2 > if
=
93
4 2 0 2 4
10
5
0
5
f x ( )
x
0 1 2 3 4 5
10
5
0
5
f x ( )
x
Farkl domenlerde, farkl fonksiyonel iliki gstermelerine karn, paral
fonksiyonlar tek bir fonksiyondur ve izimleri de tek bir fonksiyonun izimidir.
Parabol
Bir paraboln izimi aada grlmektedir.
94
Paraboln iziminden de grlebilecei gibi bir parabol, yukardan aaya, veya
aadan yukar hareket gsterebilir. Yukar kan bir paraboln en yksek noktasna tepe
noktas, aaya alan bir paraboln en alak noktasna taban noktas ad verilir. Genel olarak
bu noktalar vertex olarak adlandrlr.
Parabol ortadan blen eksene, simetri ekseni ad verilir.
Parabol erisinde ani dnler izlenmez. Aaya doru inen bir parabol daima
aaya doru iner.
Parabol erisinin, karakteristik noktalar , vertex noktasnn koordinat, varsa erinin absis ve
ordinat kestii noktalardr.
Bir paraboln en ak denklemi ,
f x ( ) a x h ( )
2
k + =
eklindedir.Fakat ne yazk ki, bu ak denklem her zaman ele gemez.
l bakta, parabol denkleminin ikinci derece bir denklem olduu anlalr. Buradaki a
katsays, paraboln ibkey (Concave) veya dbkey (Convex) olduunu belirtir. Bu katsaynn
deeri pozitifse, parabol dbkey, aksihalde ibkeydir.
Paraboln verteks noktas (h,k) noktasdr ama iaretlere dikkat edilmesi gerekir.
Paraboln x-eksenini (absis) kestii
noktalar,
f x ( ) 0 =
a x h ( )
2
k + 0 =
denkleminin kkleridir. Bu deerler,
a x h ( )
2
k + 0 =
h
k i
a
+
h
k i
a

\
|
|
|
|
|

a 0 if
0 a 0 = k 0 = if
AND VE = = ( )
95
Eer gerek bir kk bulunamazsa, parabol absisi kesmiyor anlamna gelir. Eer gerek a,
gerekse k sfra eitse, verteks orijinde olacaktr.
a h
2
k + 0 =
0 a 0 = k 0 = if
k i
a
k i
a

\
|
|
|
|
|

a 0 if
f x ( ) 2 x 3 + ( )
2
8 =
Vertex : h 3 = h 3 =
k 8 = Vertex = (-3,-8)
a 0 > Eri Yukar doru hareketli
x 100 99.999 , 100 .. =
20 10 0 10 20
1 10
4

0
1 10
4

2 10
4

3 10
4

f x ( )
x
96
4 3.5 3 2.5 2
10
9
8
7
6
f x ( )
x
Dip Noktasnn Koordinatlar (-3,-8)
Erinin x ekenini kestii koordinatlar
2 x 3 + ( )
2
8 0 =
1
5
|

\
|
|

f 1 ( ) 0 =
f 5 ( ) 0 =
Absisi kestii koordinatlar : (-1 , 0) ve (-5 , 0)
Ordinat kestii noktann koordinatlar (not : ordinat bir dikmedir ve bir fonksiyon bir dikmeyi
sadece bir kez kesebilir)
y 2 0 3 + ( )
2
8 =
y 10 = x 0 =
97
Ordinat kesim noktas, (0 , 10)
5 0 5
100
50
0
50
100
f x ( )
x
Genel Parabol Denklemi
Genel Parabol Denklemi,
f x ( ) a x
2
b x + c + =
eklindedir. Burada a parametresinin iareti, paraboln yukar veya aa almasn belirler.
Paraboln verteks noktasnn koordinatlar
b
2 a
f
b
2a
|

\
|
|

,
|

\
f x ( ) c = olur
Eer x = 0 ise,
Eer y = 0 ise, denkleminn kkleri, standart ikinci derece denklem zmnden bulunur.
98
rnek : g x ( ) 3 x
2
6 x 5 + = a 3 = b 6 = c 5 =
Parabol Yukar Doru alr. ( a=3)
g 0 ( ) 5 = Ordinat kestii yer.
b
2 a
1 =
f
b
2 a
|

\
|
|

24 =
Verteks = (1,24) yukar doru alan bir paraboln dip noktas x ekseninin
stnde ise, bu parabol x eksenini kesmez.
Elips
Elips, yumurta gibi kapal bir eridir ve daha nce incelediimiz ember, elips'in zel bir
halidir.
Elips'in genel denklemi aadaki gibidir
:
x h ( )
2
a
2
y k ( )
2
b
2
+ 1 =
Standart genel denklemde sa taraf daima 1 olur. Elips erisinin nemli noktalar, merkez noktas,
en sa nokta, en sol nokta, en alt nokta ve en st noktadr.
Merkez Noktas koordinatlar (h , k)
En sa nokta koordinat (h+a , k)
En sol nokta koordinat (h-a , k)
En alt nokta koordinat (h-b , k)
En st nokta koordinat (h+b , k)
olarak bulunur.
rnek :
x 2 + ( )
2
9
y 4 ( )
2
25
+ 1 =
zm : Burada en kritik ilem, h ve k nn iaretlerininin doru olarak saptanmasdr. Eer bilinli
ve sistematik allrsa, bu konuda hibir sorun kmaz.
x h x 2 + = y k y 4 =
h 2 =
k 4 =
h 2 = k 4 =
99
Elipsin merkez noktas (-2 , 4) noktasdr.
a 3 =
a
2
9 =
lk ilem olarak daima pozitif
olaslklar gznne alnr.
b
2
25 = b 5 =
Elips'in nemli noktalar :
En sa nokta , absis = h a + 1 = ordinat = k 4 =
En sol nokta , absis = h a 5 = ordinat = k 4 =
En alt nokta , absis = h 2 = ordinat = k b 1 =
En st nokta , absis = h 2 = ordinat = k b + 9 =
Drt tane nokta elde ettik ama bir izim iin daha fazlas gerek. Bu nedenle elipsin y ye gre
ak bir denklemini elde etmeye alalm:
x h ( )
2
a
2
y k ( )
2
b
2
+ 1 =
k
b a h + x a h x +
a
+
k
b a h + x a h x +
a

\
|
|
|
|
|

Bu sonutan, elips denkleminin aslnda paral bir fonksiyon oluturduunu ve


f x ( ) k
b a h + x a h x +
a
+ y k if
k
b a h + x a h x +
a
y k if
=
olmas gerektii anlalr. Ayrca bu fonksiyonun domeni (bamsz deiken kapsam alan)
100
a h + x 0
x h a
x h a +
a h x + 0
x h a
Yani , h a x h a + olarak bulunur.
a 3 = b 5 = k 4 =
f x ( ) k
b a h + x a h x +
a
+ f x ( ) k if
k
b a h + x a h x +
a
f x ( ) k if
=
Kolay izilmesi iin bu paral foksiyonu iki tane tek fonksiyon olarak
tanmlayalm.
h a 5 = h a + 1 =
x 5 4.9999 , 1 .. =
g x ( ) k
b a h + x a h x +
a
+ =
h x ( ) k
b a h + x a h x +
a
=
10 8 6 4 2 0 2 4 6 8 10
10
8
6
4
2
0
2
4
6
8
10
g x ( )
h x ( )
x
Grld gibi yumurtaya benziyor.
101
Euclid Kuram :
c
2
a
2
b
2
+ =
a
2
b
2
+
a
2
b
2
+
|

\
|
|
|

c a
2
b
2
+ =
Eer c dorusu, x
1
y
1
, ( ) Noktasndan balar ve x
2
y
2
, ( )
tamamlanrsa, b y
2
y
1
( ) =
noktasnda a x
2
x
1
( ) =
olur. Bu durumda, herhangibir doru parasnn uzunluu,
d y
2
y
1
( )
2
x
2
x
1
( )
2
+ =
olarak belirlenir.
(sonucun pozitif kmas iin, daima kk olan deer, byk olan deerden kartlr),
Bir ember, merkezden ayn uzaklkta olan noktalarn geometrik yeri olarak
tanmland iin, ember denklemi,
r x h ( )
2
y k ( )
2
+ =
veya
r
2
x h ( )
2
y k ( )
2
+ =
veya
olarak tanmlanr.
x h ( )
2
r
2
y k ( )
2
r
2
+ 1 =
Buradan, emberin a = b olmas durumunda elipsin zel hali olduu grlyor.
102
ember deklemini y deikeni bakmndan
aalm.
x h ( )
2
r
2
y k ( )
2
r
2
+ 1 =
x h ( )
2
y k ( )
2
+ r
2
=
h
2
2 h x k
2
+ 2 k y x
2
+ y
2
+ r
2
=
k h r + x ( ) r h x + ( ) +
k h r + x ( ) r h x + ( )

rnek : r 3 = ve merkezi (-4 , 2) olan bir emberi iziniz.


h 4 = k 2 =
h k 6 = h k + 2 =
x 10 9.9999 , 2 .. =
circleup x ( ) k h r + x ( ) r h x + ( ) + =
circledown x ( ) k h r + x ( ) r h x + ( ) =
10 7.6 5.2 2.8 0.4 2
2
0.4
2.8
5.2
7.6
10
circleup x ( )
circledown x ( )
x
103
8 6.4 4.8 3.2 1.6 0
2
0.4
1.2
2.8
4.4
6
circleup x ( )
circledown x ( )
x
Hiperboller
Hiperbollerin olas izimleri aada grlmektedir:
104
Burada mavi ile belrtilen izgiler, hiperboln asimptot izgileridir. Asimptot izileri,
hiperbol erisinin bir paras deildir. Hiperbol erisinde x +/- sonsuz deerine yaklarken,
fonksiyon da asimptot izgisine yaklar fakat asla eit olmaz. Asimptot izgileri hiperpol
erisinin sreksizlik gsterdii limit deerleridir. Asimptotlarn kesime noktas, hiperboln
merkezidir.
Hiperboller, parabole benzeyen bir ekilde, yukar ve aaya doru alrlar. Her iki
almn da verteks noktalar vardr.
Hiperbol erisinin standart formlar,
Form
x h ( )
2
a
2
y k ( )
2
b
2
1 =
y k ( )
2
b
2
x h ( )
2
a
2
1 =
Merkez (h ,
k)
(h ,
k)
Alm
saa ve
sola
Yukar ve
aa
Verteks Noktalar (h + a , k) ve (h - a ,
k)
(h , k - b) ve (h , k +
b)
Asimptotun eimi : +/-
b
a
+/-
b
a
Asimptotun denklemi
y = k +/-
b
a
x h ( ) y = k +/-
b
a
x h ( )
Temel denklemler ayn, sadece yukar ve yana alan hiperboller arasnda bir iaret fark
bulunuyor.
rnek :
x 3 ( )
2
25
y 1 + ( )
2
49
1 = Hiperbolnn izimini yapnz.
zm : Hiperboln genel grafiinin izilebilmesi iin, ak bir denklemini karalm. Bu hiperbol,
birinci tipten yani saa ve sola alan bir hiperboldr ve ak forml,
x h ( )
2
a
2
y k ( )
2
b
2
1 =
olarak verilmitir. Bu ifade alrsa,
105
h
2
a
2
x
2
a
2
+
2 h x
a
2

k
2
b
2
y
2
b
2
+
2 k y
b
2

\
|
|
|

1 =
h
2
a
2
k
2
b
2

x
2
a
2
+
y
2
b
2

2 h x
a
2

2 k y
b
2
+ 1 =
Son ifadenin y cinsinden zm, iki yane zm bants ile sonulanr:
b
2
k
b
2
a h + x ( ) a h x + ( )
a b
+

b
2
k
b
2
a h + x ( ) a h x + ( )
a b

Bu zm ile elde edilen bantlar basitletirilebilir ve her bant ayr bir fonksiyona
atanabilir:
f x ( ) k
b h
2
a
2
2 h x x
2
+
a
+ =
1 ( )
2 ( )
g x ( ) k
b h
2
a
2
2 h x x
2
+
a
=
Buradan grlebilir ki, tam hiperbol bir fonksiyon deildir. nk her x iin iki
tane y oluur. Ama yarm hiperbol, yani f(x) ve g(x) fonksiyonlar birer fonksiyondur.
nk tek deerlidir. (Bir x e bir y kar gelir)
nce izelim, sonra hesaplayalm. Bylelikle hesapladmz deerlerin doruluunu
kontrol edebiliriz.
Saysal rnee geri dnelim.
x 3 ( )
2
25
y 1 + ( )
2
49
1 =
y zm,
106
7 x 2 + x 8
5
1
7 x 2 + x 8
5
1
|

\
|
|
|
|
|

f1 x ( )
7 x 2 + x 8
5
1 =
f2 x ( )
7 x 2 + x 8
5
1 =
x 20 19.999 , 20 .. =
20 10 0 10 20
20
10
0
10
20
f1 x ( )
f2 x ( )
x
10 8 6 4 2 0 2 4 6 8 10
10
8
6
4
2
0
2
4
6
8
10
f1 x ( )
f2 x ( )
x
107
a 5 = b 7 =
Hiperboln Merkezi : x 3 x h =
h 3 =
y k y 1 + =
k 1 =
H iperboln Merkezi, (3 , -1) noktas olarak belirlenir.
Verteks Noktalar : h a 2 = k 1 = (1 , 2)
h a + 8 = k 1 = (5 , 2)
Asimptotlarn Eimleri .
b
a
1.4 =
b
a
1.4 =
Asimptot Denklemleri :
y1 x ( ) k
b
a
x h ( ) + =
y2 x ( ) k
b
a
x h ( ) =
x 20 19.999 , 20 .. =
108
20 10 0 10 20
20
10
0
10
20
f1 x ( )
f2 x ( )
y1 x ( )
y2 x ( )
x
Sonuta izim tamamland. Hiperbol izimi, emek youn bir ilemdir. Fakat, hesap makineleri
ilemleri kolaylatrr.
Bir baka rnek yaparak pratiimizi arttralm :
rnek:
y
2
9
x 2 + ( )
2
1 =
zm : x terimi negatif olduundan eri yukar aa alacaktr.
y ak denklemleri : (sadece zm izlemek amac ile)
y
2
9
x 2 + ( )
2
1 =
109
3 x
2
4 x + 5 +
3 x
2
4 x + 5 +
|

\
|
|
|

f x ( ) 3 x
2
4 x + 5 + =
g x ( ) 3 x
2
4 x + 5 + =
x 6 5.999 , 2 .. =
6 5.2 4.4 3.6 2.8 2 1.2 0.4 0.4 1.2 2
10
8
6
4
2
0
2
4
6
8
10
f x ( )
g x ( )
x
110
standart eri :
y k ( )
2
b
2
x h ( )
2
a
2
1 =
rnek :
y
2
9
x 2 + ( )
2
1 = veya
y
2
9
x 2 + ( )
2
1
1 =
k 0 = b 3 = a 1 =
x 2 + x h = h 2 =
Merkezin Koordinatlar : h k , ( ) 2 0 , ( )
Eriden kontrol : Doru gibi grnyor.
Verteks noktalar : h k b + , ( ) ve h k b , ( )
k b + 3 = k b 3 = 2 3 , ( ) ve 2 3 , ( ) noktalardr .
Eriden kontrol : Doru gibi grnyor.
Asimptotlarn eimi : +/-
b
a
b
a
3 =
b
a
3 =
Asimptot Denklemleri : y k
b
a
x h ( ) + = y k
b
a
x h ( ) =
y1 x ( ) k
b
a
x h ( ) + = y2 x ( ) k
b
a
x h ( ) =
111
6 5.2 4.4 3.6 2.8 2 1.2 0.4 0.4 1.2 2
10
8
6
4
2
0
2
4
6
8
10
f x ( )
g x ( )
y1 x ( )
y2 x ( )
x
Herey tamam.
Karekk Fonksiyonu
y x ( ) x =
geerlilik alan (domen) tm pozitif x ler ve x=0
x 0 0.0001 , 10 .. =
112
0 2 4 6 8 10
2
0
2
4
6
y x ( )
x
y x ( ) x 5 =
geerlilik alan
(domen)
x 5 0
x 5 5.0001 , 10 .. =
5 6 7 8 9 10
1
0
1
2
3
4
y x ( )
x
x 5
113
nc derece fonksiyon y x ( ) x
3
=
Bu fonksiyonun geerlilik alan (domen) x < <
x negatif olunca fonksion da negatif olur (-x)(-x)(-x) = -x
x = 0 olunca fonksiyon da sfr olur.
x pozitif olunca fonksyion da pozitif olur
.
yani, x = 0 , fonksiyonun bkm noktasdr.
x 10 9.999 , 10 .. =
10 5 0 5
1 10
3

500
0
500
1 10
3

y x ( )
x
llk defa bir fonksiyonun bkm noktasna rastlyoruz.
TRANSFORMASYONLAR (DNM)
Transformasyonlar (dnmler), bir fonksiyonun fonksiyonal iliki deimeden daha
basit formlara dntrlmesi anlamna gelir.
Dikey Kaydrma
114
Bir fonksiyonun dikey kaydrlmas, +c birim yukar veya -c birim kadar aaya
telenmesi ile gerekleir.
rnek : t x ( ) 1.8 x 12 + = Fonsiyonunu 5 birim yukar, 5 birim aa teleyiniz.
zm : Burada temel (baz) konum veya sfr konumu, t x ( ) 1.8 x 12 + = denklemidir.
5 birim yukar teleme, sabit terime 5 birim eklenmesi ile gerekletirilir.
ust x ( ) 1.8 x 12 + 5 + = sonu : ust x ( ) 1.8 x 17 + =
5 birim aa teleme, sabit terimden 5 birim karlmas ile
gerekletirilir.
alt x ( ) 1.8 x 12 + 5 = sonu : alt x ( ) 1.8 x 7 + =
Hepsini birlikte izelim :
10 5 0 5 10
20
10
0
10
20
30
40
ust x ( )
t x ( )
alt x ( )
x
Bir dnm oluturmak ok kolay deildir. Fonksiyonun ve hareket sisteminin iyi incelenmi
115
Bir dnm oluturmak ok kolay deildir. Fonksiyonun ve hareket sisteminin iyi incelenmi
olmas gerekir.
necelikle amacn iyi saptanmas nemlidir. rnek olarak fonksiyon dikey kaydrlacaksa, bu
ynde bir hareketi gerekletirebilecek yntemlere younlalmas gerekir.
Bundan sonra ilk yaplacak ey, fonksiyonel ilikinin iyi anlalmas ve fonksiyonun esas iliki
ve parametreler olarak yeniden yazlmasdr.
rnek olarak, y 3 x 6 + = fonksiyonun dnelim: Bu fonksiyonda esas ilikinin,
y x = olduu ve y k x b + =
eklinde ifade edilebilecei grlebilir.
Bundan sonra parameterlerin ekileri iyi incelenmeli ve istenilen ynde telemeyi salayan bir
parametrenin olup olmad anlalmaya allmaldr. Eer mevcut parametrelerden birisi,
istenilen ynde hareketi salayamyorsa, yeni bir parametre eklenerek istenilen ynde hareket
salanp salanamayaca zerinde bilgi edinilmeye allmaldr.
rnek olarak, nce y x = erisini, sonra da k parametresinin etkisini anlamak iin k=3 ve k=6
deerindeki erilerini izdirip farka bakalm:
y x ( ) x = y1 x ( ) 3 x = y2 x ( ) 6 x =
0 2 4 6 8 10
0
2
4
6
8
y2 x ( )
y1 x ( )
y x ( )
x
116
k Parametresi, erinin geniliini arttryor fakat yukar teleme iin etkisiz.
Bundan sonra b parametresinin etkisine bakalm
:
y3 x ( ) x 6 = y4 x ( ) x 6 + =
10 5 0 5 10
0
1
2
3
4
y3 x ( )
y4 x ( )
x
Bu bilgilerle, b parametresinin de, sadece yatay telemeyi kontrol edebildii
grlyor. Ama biz u anda sadece dikey teleme ile ilgileniyoruz.
Fonksiyona yerletirdiimiz parametrelerin hibiri dikey telemeyi komtrol
edemediine gre, dikey telemeyi kontrol eden bizim farkna varmadmz bir
parametrenin daha bulunmas kanlmazdr. Bu parametreyi de gznne alan denklemin
yazlm,
y x ( ) a k x b + + =
olabilir.
y5 x ( ) 3 x + =
117
0 2 4 6 8 10
0
2
4
6
8
10
y5 x ( )
x
te bu kadar. Aradmz parametre a parametresi imi. Bylece, a {- , }arasnda
deiebilen a parametresinin deeri, erinin dikey telemesini salayan parametre deeri
olduu grlyor.
x ve y eksenleri etrafnda erinin aksetmesi
Bir f(x) erisinin, x eksenine gre aksi, -f(x) erisidir.
Bir f(x) erisinin y eksenine gre aksi, f(-x) erisidir.
rnek : g x ( ) x
2
= erisinin x eksenine gre aksini bulunuz.
x eksenine gre aksi :
f x ( ) x
2
=
10 5 0 5 10
100
50
0
50
100
f x ( )
g x ( )
x
118
rnek : h x ( ) x = erisinin y eksenine gre aksini
bulunuz.
zm : Bu fonksiyonun
h x ( ) x = olmas bizi engellemez. Fonksiyonun domeni,
x 0 veya x 0
h 4 ( ) 4 ( ) = ve bu da 4 olduundan anlamszdr. Yani hibir
gerel say birbiri ile arplnca negatif bir say vermez. Yani,
olmaldr.
h 4 ( ) 2 ( ) 2 ( ) = 4 = 2i = Fakat negatif saylar iin byle deil. Yani
h 100 ( ) 100 ( ) = 10 ( ) 10 ( ) = 10 = deerini verir. Yani iki
negatif gerel saynn arpm pozitif bir say verir. 2 ( ) 2 ( ) 2 =
t x ( ) x =
10 5 0 5 10
0
1
2
3
4
h x ( )
t x ( )
x
119
Simetri
Bir f(x) erisinde, bir (a , b) noktas iin, (-a , -b) noktas bulunuyorsa, bu eri y
ekseni boyunca simetriktir.
Bir f(x) erisinde, bir (a , b) noktas iin, (a , -b) noktas bulunuyorsa, bu eri x
ekseni boyunca simetriktir.
Bir f(x) erisinde, bir (a , b) noktas iin, (-a , -b) noktas bulunuyorsa, bu eri , orijin
etrafnda simetriktir.
f x ( ) x
2
=
10 5 0 5 10
0
20
40
60
80
100
f x ( )
x
120
Simetri Testleri :
Simetrik erilerin tannmas iin, baz ok basit testler uygulanabilir.
Bir eride tm y lerin, -y ile yer deitirdii yeni bir eri bulunabiliyorsa, bu eri x ekseni 1.
121
Bir eride tm y lerin, -y ile yer deitirdii yeni bir eri bulunabiliyorsa, bu eri x ekseni 1.
etrafnda simetriktir.
Bir eride tm x lerin -x ile yer deitirebildii, alternatif bir baka eri bulunabiliyorsa, bu 2.
eri x ekseni etrafnda simetriktir.
Bir eride tm x lerin -x ile tm y lerin de -y ile yer deitirdii yeni bir eri bulunabiliyorsa, 3.
bu eri orijin etrafnda simetriktir.
Rasyonel Fonksiyonlarn izimleri
f x ( )
1
x
= Erisinin izimi
Bu erinin sreksizlik gsterdii tek nokta, x = 0 noktasdr. Bunun dnda tm gerel x
ler iin, fonkiyon sreklidir.
x 10 9.9 , 10 .. =
f x ( )
1
x
x 0.1 < if
1
x
x 0.1 > if
=
10 5 0 5 10
4
2
0
2
4
f x ( )
x
122
Dey ve yatay asimptot izgileri x ve y eksenleridir.
rnek : y
3 x 6 +
x 1
=
zm : lk olarak fonsiyonun sreksizlik noktasn saptayalm :
x 1 0 = x 1 = fonksiyon, bu noktada sfra blnme belirsizli
gsterdiinden bu noktada sreksizdir.
f x ( )
3 x 6 +
x 1
x 1 < if
3 x 6 +
x 1
x 1 > if
=
x 10 9.9999 , 10 .. =
5 3.5 2 0.5 1 2.5 4 5.5 7 8.5 10
20
16
12
8
4
0
4
8
12
16
20
f x ( )
x
Eksenleri kestii noktalar:
x eksenini kestii nokta (lar) : f x ( ) 0 =
+
123
3 x 6 +
x 1
0 = 3 x 6 + 0 x 1 ( ) =
3 x 6 + 0 =
3 x 6 = x 2 =
Erinin iziminden bu noktay saptayabiliyoruz. Saysal olarak, f 2 ( ) 0 =
y eksenini kestii nokta (lar) : f 0 ( ) 6 =
Erinin iziminden bu noktay da saptayabiliyoruz.
Yatay ve Dikey Asimptot(lar)
Einin sreksizlik gsterdii x = 1 erisi ayn zamanda dikey asimptottur.
Yatay asimptotun saptanmas daha zordur. Eer, pay ve paydann eksponenti 1 ise
bu erinin yayay bir asimptotu vardr. Bu yatay asimptot,
y
3
1
= y 3 = izgisindedir
124
10 9 8 7 6 5 4 3 2 1 0 1 2 3 4 5 6 7 8 9 10
8
7.2
6.4
5.6
4.8
4
3.2
2.4
1.6
0.8
0
0.8
1.6
2.4
3.2
4
4.8
5.6
6.4
7.2
8
f x ( )
x
125
Polinomlar
Bir polinom, P x ( ) a
0
a
1
x + a
2
x
2
+ a
3
x
3
+ = eklinde ifade edilenilen
ifadelerdir. Poinomlar, ierdikleri x deerinin ssne gre ,
a
0
Sfrnc dereceden
polinom
a
0
a
1
x + Birinci dereceden
polinom
eklinde adlandrlrlar.
P1(x) ve P2(x) eklinde polinomlar, birbirleri ile toplanr, kartlr, arplr veya blnebilir.
ki polinomun blnmesi iin uzun ve emek youn yntemler olmasna karn, bunlar
gemite kalmtr. Gnmzde bu ilemleri bilgisayarlar gerekletirmektedir.
126
rnek :
5 x
3
x
2
6 +
x 4
acaba, ilk polinomun ilk teriminin bulunmas iin, x-4 teriminin ka ile arplmas
Yant : 5 x
2
( ) x 4 ( ) 5 x
3
20 x
2
=
gerekir?
Bunu ilk polinomdan
karabiliriz.
5 x
3
x
2
6 + ( ) 5 x
3
20 x
2
( ) 19 x
2
6 + =
Buna geriye kalan ad verilir. Yani,
Eer bir P(x) polinomunun derecesi en az bir ise, derecesi bir dk olan yle bir Q(x)
polinomu, r says ve R geriye kalan terimi vardr ki,
P x ( ) x r ( ) Q x ( ) R + =
olsun.
rneimiz iin :
5 x
3
x
2
6 + x 4 ( ) 5 x
2
19 x + 76 + ( ) 310 + =
5 x
3
x
2
6 +
x 4 ( ) 5 x
2
19 x + 76 + ( ) 310 +
1 =
salama :
Mathematica 8 ile elde edilen bu sonucun el ile elde edilmesi ok zordur.
ilk olarak , P(x) polinomnun ilk terimini, x-4 n ilk terimi ile bleriz:
5 x
3

x
5 x
2
=
sonra 5 x
2
ile x 4 arplr. 5 x
3
20 x
2

127
sonra P x ( )
5 x
3
20 x
2
den karlr veya
5 x
3
20 x
2
terimleri iaret deitirilir 5 x
3
20 x
2
( ) 5 x
3
20 x
2
+ =
ve P(x) ile toplanr :
5 x
3
x
2
6 +
+
5 x
3
20 x
2
+
-----------------------------
19 x
2
6 +
imdi
19 x
2

x
19 x = yukars : 5 x
2
19 x + olur.
x 4 ( ) 19 x ( ) 76 x 19 x
2
=
19 x
2
6 +
+ 19 x
2
76 x +
---------------------------------
76 x 6 +
76 x
x
76 = yukars 5 x
2
19 x + 76 + olur
x 4 ( ) 76 76 x 304 + =
76 x 6 +
76 x 304 +
+
-------------------------------------
310
(Geriye Kalan)
128
Sonu ayn. Gnmzde nasl karekk deerini hesapla bulmaya almyor ve hesap makinesi
kullanyorsak, bu deerin de elle hesaplanmas anlamszdr.
Bu konuda daha kolay olduu belirtilen sentetik blme yntemi de uygulanabilir.
Burada P(x) polinomunun, a x
3
b x
2
+ c x + d + eklinde olduu
dnlr ve x 4 ( ) 5 x
3
x
2
6 + ( ) arpmnn (not: 5 x
3
x
2
0 x + 6 + ) sadece
katsaylar
4 5 1 0 6 olarak yazlr.
5 aaya ekilir.
4 5 1 0 6
5
4 ile 5 arplr ve
4 5 1 0 6
20
5 19
toplanr :
19 ile 4 arplr ve
4 5 1 0 6
310
20 76
5 19 76 316
yani sonu : Q x ( ) 5 x
2
19 x + 76 + =
Kalan : 316
Ayn sonu.
129
Polinomlarn Kkleri (Sfrlar)
Polinomlarn kkleri veya sfrlar, bu polinomum deerin sfr yapan x deeri veya
deerleridir.
Eer bir r deeri, bir P(x) polinomunun kk ise, P(r) = 0 olur. Bir baka ifade ile, r
deeri, P(x) = 0 denkleminin kkdr.
Polinomlarn sfrlarnn bulunmas, aslda P(x) = 0 denkleminin kknn bulunmasdr.
Bu konuda, birinci ve ikinci derece polinomlarn kklerinin bulunmasn yapmtk. nc
dereceden polinomlarn da ok karmak ve aklda tutulmaz olmakla birlikte bir analitik zm
vardr.
nc derecenin stndeki polinomlarn analitik zmleri yoktur. Bunun iin, saysal
yntemler uygulanmas gerekir. Saysal yntemlerden bazlar olaanst basit, bazlar ise daha
detayl olabilir.
Temel Cebir Teoremi (Kuram)
Eer P(x) n inci dereceden bir polinom ise, bu polinomun tam olarak n tane kk vardr.
Bazlar da tekrarlanabilir. Yani, bir polinomun kklerinin toplam, polinomun derecesine eittir.
Bu kuramn sonular bazan ok yararl olabilir. nk, toplam kk saynn st snr
bilindii iin, gereksiz yere olmayan kkler aranmaz.
Faktr Kuram
1-Bir P(x) polinomunun eer r bir sfr ise, x-r bu polinomun bir arpandr.
2-Eer x-r bir polinomun arpan ise, r bu polinomun bir kkdr.
Bu kuramn iki tane sonucu olur.
Sonu - 1 :
Eer, P(x) n inci derece bir polinom ise ve r bunun sfr ise, bu polinom,
P x ( ) x r ( ) Q x ( ) =
eklinde yazlabilir. Burada, Q(x), n-1 derece olan yeni bir polinomdur. Q(x), P(x) polinomunun,
x-r ile blnmesi ile elde edilir.
Sonu - 2 :
Eer, P x ( ) x r ( ) Q x ( ) = ve x = t, Q(x) 'in bir kk ise, t ayn zaman da P(x) 'in de bir
kkdr.
130
rnek : P x ( ) x 2 ( ) x 3 ( ) x 7 ( ) =
Poinomunun grafiini iziniz:
x 2 ( ) x 3 ( ) x 7 ( )
zm : P x ( ) x
3
12 x
2
41 x + 42 = olarak hesaplanr. Bu polinomun
kkleri : x 2 = , x 3 = ve x 7 = olarak bulunur.
0 1 2 3 4 5 6 7 8 9 10
20
17.5
15
12.5
10
7.5
5
2.5
0
2.5
5
P x ( )
x
Kklere Dikkat !!!
rnek :
P x ( ) x
3
2 x
2
+ 5 x 6 = Polinomunun kklerinden biri ,
x 2 = dir. Dier kklerini bulunuz.
131
zm :
1 - (Ufku Amak Bakmndan)
P(x) ,nc dereceden bir polinomdur ve nc derece polinomlarnn kklerininin
analitik zmleri vardr. Yani, katsaylarnn konularak analitik kkleri veren forml
vardr. Fakat, bu forml uzun ve karmaktr ve el ile hesaplama iin uygun deildir.
Ama bilgisayar kullanm ile kolaylkla uygulanabilir. Bu bilgisayar programlar, 1990
lara doru gelitirilmitir. Yani, bu tarihten nce yazlm kitaplar okuyanlar, bu
zmleri bilemezler. Bu da bilimde daima gnceli kovalamann ne denli nemi
olduunu ortaya koyar.
te dier kkler : Forml bilmenize bile gerek kalmyor.
x
3
2 x
2
+ 5 x 6 0 =
1
2
3
|

\
|
|
|

teki kkler de -3 ve -1 olarak bulunuyor.


Sonucu grafikle kontrol edelim
:
5 4 3 2 1 0 1 2 3 4 5
10
8
6
4
2
0
2
4
6
8
10
P x ( )
x
x 2 =
Sonu tamam. Fakat bu zm sadece akademik ortamlar iin geerli.
132
2 - (Daha uygulanabilir bir zm )
P x ( ) Polinomunun bir kk olduuna gre,
x 2 ( ) Q x ( ) 0 =
eitlii yazlabilir. Burada Q(x) derecesi P(x) den bir derece aa olan bir polinomdur.
P(x) nc derece bir polinom olduuna gre, Q(x) ikinci derece bir polinomdur ve
ikinci derece polinomlarn kklerinin analitik zmleri kolaylkla uygulanabilecek bir
forml olarak eldedir. Sorun Q(x) polinomunun saptanmasdr. Bu da P(x) polinomunun
x-2 polinomu ile blnmesi ile elde edilir.
Polinomlarn blnmesini fazla okumam insanlara sorsak ya uzun blme, ya da
sentetik blme olarak aklarlar. Oysa bu yntemlerin ikisi de modas gemi eski
yntemlerdir. Polinomlarn birbirine blnmesi, bilgisayar kullanm ile gerekletirilir. Bu i
iin en uygun Mathetmatiaca 8 programdr. Buradan ,
Q x ( ) 3 4 x + x
2
+ =
olarak bulunur.
Salama :
x 2 ( ) 3 4 x + x
2
+ ( ) = x
3
2 x
2
+ 5 x 6
Q(x) polinomunun doru hesaplanm olduu doruland.
Q(x) in kkleri de,
3 4 x + x
2
+ 0 =
1
3
|

\
|
|

olarak bulunur. Olay zmlenir. Q(x) polinomunun sentetik daha da kts uzun
blme yolu ile hesaplanmasn aslnda kime brakmak gerektii herkes tarafndan
bilinir ama ne yazk ki hammaliye hep rencilere kalr. stelik de yapamadklar
zaman kt not alrlar. Oysa, birlikte bu kadar polinom zdk ve hi byle
yntemler kullanmadk ve artk sonsuza kadar da hi kullanmayacaz.
Kklerin Tekrarll
Polinomlarn kkleri bazen tek bir kk olabilir. rnek olarak,
133
3 4 x + x
2
+ 0 =
ikinci derece denkleminin x=-1 ve x=-3 olarak iki kk vardr. Her iki kk de tekrarlanmaz. Eer
bir kkn tekrarll k ile belirtilirse, her ikisinin de tekrarl k = 1 olarak belirtilir. Bu ekilde
tkrarll 1 olan tekrar etmeyen kklere basit kkler ad da verilir. Yukardaki polinom
denkelemi, ikinci derecedir. Yani, toplam iki kk bulunabilir. Her kkn tekrarl da 1
olduuna gre sistemin kklerinin toplam tekrarll yani k lar toplam da 1+1=2 olur. Bu
durumda kklerin tekrarll, kuramsal kk saysna toplanr.
rnek : P x ( ) x 5 ( ) x 5 ( ) = polinomunun sfrlarnn tekrarlln bulunuz.
zm : Bir polinomun sfr olmaz. P(x) = 0 denkleminin sfrlar (kkleri) olur. Burada,
x 5 ( ) x 5 ( ) 0 =
denkleminin sadece deeri x = 5 olan bir tane kk vardr. Bu kkn tekrarllk katsays k=2
dir. nk ayn kk iki kez tekrarlanmaktadr. Bylece kklerin toplam tekrarl 2 dir ve
sistemin kuramsal kk says olan 2 ye eittir.
Polinomlarn Grafikleri
Polinomlarn grafikleri yumuak gidili izimlerdir. Bu grafiklerde ani dnler izlenmez.
Dnler yumuak hatl ve nceden kestirilebilir bir dzendedir.
Polinom erileri yn deitirebilir. Yani, yukar karken yavaa aaya doru veya tersi
bir hareket gsterebilirler. Bu tr yol deiimlerine "Dn Noktalar" ad verilir.
Polinom erilerinde, polinomun derecesinin bir altnda sayda dnm noktalar
gzlenebilir. rnek olarak aada ikinci dereceden polinomlarn erileri grlmektedir. Bu
erilere "Parabol" ad verilir. Parabol erileri ikinci derece bir paraboln izimleri olduklarndan
sadece bir tek dnm noktas ierebilirler. Bu dnm noktasnn durumuna gre iki tr polinom
olabilir. Bunlardan biri yukarya doru alan, dieri de aaya doru alan parabollerdir.
rnek : Yukar doru alan parabol
Parabol1 x ( ) 2 x
2
5 x + 8 + =
x 10 9.99 , 10 .. =
134
10 8 6 4 2 0 2 4 6 8 10
0
30
60
90
120
150
180
210
240
270
300
Parabol1 x ( )
x
Dnm Noktas
:
2 a x b + 0 =
x
b
2 a
=
2 2 x 5 + 0 =
x
5
4
= x 1.25 = Parabol1 1.25 ( ) 4.875 =
x 10 9.99 , 10 .. =
1.5 1.45 1.4 1.35 1.3 1.25 1.2 1.15 1.1 1.05 1
4.87
4.871
4.872
4.873
4.874
4.875
4.876
4.877
4.878
4.879
4.88
Parabol1 x ( )
x
135
rnek : Aaya doru alan parabol
Parabol2 x ( ) 3 x
2
4 x 9 =
10 8 6 4 2 0 2 4 6 8 10
100
88.5
77
65.5
54
42.5
31
19.5
8
3.5
15
Parabol2 x ( )
x
Dnm Noktas : a 3 = b 4 =
dn
b
2 a
= dn 0.667 =
0.8 0.65 0.5 0.35 0.2 0.05 0.1 0.25 0.4 0.55 0.7
10
9.6
9.2
8.8
8.4
8
7.6
7.2
6.8
6.4
6
Parabol2 x ( )
x
Grafikler incelenince, ilk grafiin a katsays + olduundan paraboln yukar doru
136
Grafikler incelenince, ilk grafiin a katsays + olduundan paraboln yukar doru
ald, dnm noktasnnda fonksiyonun deerinin 0 n stnde olduundan paraboln x
eksenini kesmedii, bu durumda paraboln gerel kklerinin olmad grlr.
kinci eeri iin de durum ayn fakat aksi yndedir. Paraboln a katsaysnn iareti
eksi olduu iin parabol aaya doru almaktadr. Dnm noktasnda fonksiyonun
deeri eksi olduundan, fonksiyon x eksinini kesmez. Bu yzden paraboln gerel kkleri
yoktur.
2 x
2
5 x + 8 + 0 =
3 x
2
4 x 9 0 =
Kkleri :
2
3

23 i
3
+
2
3

23 i
3

\
|
|
|
|
|

5
4

39 i
4
+
5
4

39 i
4

\
|
|
|
|
|

Her iki paraboln de sadece sanal kkleri bulunmaktadr.


Biraz da bir paraboln izimleri zerine genel prensiplerden bahsedelim. Bir polinom,
a
0
a
1
x + a
2
x
2
+ ... +
eklindedir. Burada her x deeri iin, polinomun bir deeri hesaplanabilir. Bu polinoma ksa
olarak P(x) polinomu diyebiliriz. P(x) belirli bir polinomu belirtir. Bu P(x) polinomu, bir kez P(x)
olarak tanmlandnda, her x deeri verildiinde hesaplanabilen P(x) deerinin hesaplanma
biim deimez. Yani belirli bir P(x) polinomunun yaps sabittir. Bir P(x) polinomununda her x
deeri iin bir ve tek bir P(x) deeri hesaplanabildiinde gre, x deerlerine bamsz
(deerleri baka bir koula bal olmadan seilebilen) deiken ad verilir. P(x) in deeri,
bamsz deiken x deerine bal olduundan, P(x) de baml deiken adn alr. Bamsz
deiken x deerlerine karlk, baml deiken P(x) deerleri, kartezyen grafik sistemi
zerinden nokta, nokta gsterilebilir. Bu grafiklere P(x)/x grafikleri denilir. Yukardaki rnekte
de byle grafikler grlmektedir.
P(x)/x grafiklerinin ilgin noktalar, dnm noktalar, grafiin x eksenini kestii yer
veya yerler, grafiin y eksenini kestii yer (kesinlikle bir tanedir), varsa, yatay, dikey ve eik
asimptotlar (polinomlarda yoktur), sreksizilk noktalar (polinomlarda yoktur) bilgileridir.
Dnm noktalar, polinomlarda polinomun derecesinden bir eksiine kadar olabilir.
Grafiin y ekseninini kestii yer f(0) deeridir. Baz fonksiyonlar sadece pozitif alanda
tanml olabildiklerinden y eksenini kesmeyebilirler.
Grafiin x eksenini kestii yerler, f(x) = 0 denkleminin kkleridir. Polinomlarda 3 nc
derecenin stnde analitik kk bulma algoritmalar yoktur. Kkler saysal yntemle veya yaklak
olarak grafik izimleri ile bulunabilir. Mhendislikle yaklak yntemler ounlukla yeterlidir.
Kklerin tekrarllk katsaylar kkn etkisi zerinde bilgi verir. Eer x= r , P(x) polinomunun
tekrarllk katsays k olan bir kk ise,
Eer k tek bir say ise, x=r deerine kar gelen x , absisi keser. 1.
Eer k ift bir say ise, x=r deerine kar gelen x , absisi kesmez, sadece dokunur. 2.
137
Ayrca, eer k>1 ise, grafik x=r noktasnda dzleir.
Bundan baka, x deeri sonsuz kk ve sonsuz byk olduunda grafiin davran
incelenmelidir. Polinomlarda bu durumlarda P(x) deeri snrsz ykselebilir veya azalabilir.
Aadaki test bu davrann incelenmesinde yararl olur.
Baat Katsay Testi
Bir P(x) polinomunun derecesi n olduunda, polinomun yazl,
P x ( ) a
0
a
1
x
2
+ = ... a
n
x
n

eklindedir. Burada polinonumun eksterm deerlerdeki davran polinomun baat (dominant) terimi
olan ss en yksek olan x teriminin katsays olan a
n
deeri ile kontrol edilir.
Eer, a
n
>0 ve n bir ift say ise, P(x) polinomunun erisi, her iki taraftan da snrsz ykselir.
Bu konuda en iyi rnek, y = x
2
eisidir.
P x ( ) x
2
= x 1000 99.99 , 1000 .. =
1 10
3
800 600 400 200 0 200 400 600 800 1 10
3

0
3 10
4

6 10
4

9 10
4

1.2 10
5

1.5 10
5

1.8 10
5

2.1 10
5

2.4 10
5

2.7 10
5

3 10
5

P x ( )
x
Eer, a
n
>0 ve n bir tek say ise, P(x) polinomunun erisi, sol taraftan snrsz azalr, sa
taraftan da snrsz azalr. Bu konuda en iyi rnek, y = x
3
eisidir.
138
P x ( ) x
3
=
1 10
3
800 600 400 200 0 200 400 600 8001 10
3

1 10
9

8.4 10
8

6.8 10
8

5.2 10
8

3.6 10
8

2 10
8

4 10
7

1.2 10
8

2.8 10
8

4.4 10
8

6 10
8

P x ( )
x
Eer, a
n
<0 ve n bir ift say ise, P(x) polinomunun erisi, her iki taraftan da snrsz azalr. Bu
konuda en iyi rnek, y = -x
2
eisidir.
P x ( ) x
2
=
1 10
3
800 600 400 200 0 2004006008001 10
3

3 10
5

2.7 10
5

2.4 10
5

2.1 10
5

1.8 10
5

1.5 10
5

1.2 10
5

9 10
4

6 10
4

3 10
4

0
P x ( )
x
Eer, a
n
<0 ve n bir tek say ise, P(x) polinomunun erisi, sol tarafta snrsz ykselir ve sa
tarafta snrsz azalr. Bu konuda en iyi rnek, y = -x3 eisidir.
139
P x ( ) x
3
=
1 10
3
800 600 400 200 0 200 400 600 8001 10
3

6 10
8

4.4 10
8

2.8 10
8

1.2 10
8

4 10
7

2 10
8

3.6 10
8

5.2 10
8

6.8 10
8

8.4 10
8

1 10
9

P x ( )
x
Bir Polinom Fonksiyonun Yaklak Bir iziminin Yaplmas in Yntem :
nce, polinomun tm sfrlarn ve tekrarllk katsaylarn bulunuz. Bu ekilde, erinin x
eksenini kestii noltalarn bulunmas ve eer erinin dzletii noktalar varsa bunlarn bulunmas
salanm olur.
Sonra, erinin y eksenini kestii noktay bulunuz. (0 , P(0))
Sonra baat katsay testi ile erinin ekstrem deerlerdeki davrann saptaynz.
Sonra karakteristik noktalar ierecek eitli (x ,P(x)) noktalar hesaplayarak bunlar
kartezyen grafikte uyglaynz ve aralarnda yumuak bir izgi ile birletiriniz. Ne kadar nokta
hesaplanrsa o kadar geree yakn bir eri elde edilir.
rnek : P x ( ) x
4
x
3
6 x
2
= erisinin grafiini iziniz.
zm : Eeri 4 nc dereceden bir polinom. En ok 3 tane (n-1) dnm noktas olabilir.
Erinin y eksenini kestii nokta (0 ,0) noktas olan orijin dir. Bu durumda P(0) ayn zamanda absisi de
kesen orijin olmaktadr.
Erinin x eksenini kestii noktalar bulmaya alalm . Eeri 4 nc derece ve ak bir analitik
kk algoritmas yok. Kklerini bulmak kolay olmayacaa benziyor.
Aslnda deil, uygulamalarda ve snavlarda ounlukla kolay indirgenecek rnekler verilir. Yoksa iin
iinden klamaz. Bu rnek de x2 ,le blnnrse ok kolay bir ifadeye indirgenir:
P x ( ) x
2
x
2
x 6 ( ) =
Absisi kesebilecek noktalar, x
2
x
2
x 6 ( ) =0 dekleminin kkleridir. Bu kklerden
140
birinin x=0 olmas gerektii daha ilk bakta grlr nk bir arpmn sonucunun sfr olmas
iin, mutlaka arpanlardan birinin sfr olmas gerekir. Dier kkler ise, dier arpan olan ikinci
derece polinomun kkleridir. Bunlar da, belirli olan formlden yararlanlarak kolaylkla
hesaplanabilir. Fakat bu polinom da aslnda x 3 ( ) x 2 + ( ) olarak yazlabildiinden tm
kkler belli olur. Denklemi yeniden yazalm:
x x x 3 ( ) x 2 + ( ) 0 =
Kkler : x = 0 Tekrarl k = 2 (ift kk) (eri, burada dzleir)
x = -2 k= 1
x = 3 k=1
Baat terim katsays ( yani x
4
teriminin katsays ) a
n
: 1 yani pozitif ve tek say, bu durumda
eri her iki utan da snrsz ykselir.
En sonunda baz noktalar :
P 3 ( ) 54 = P 1 ( ) 4 = P 1 ( ) 6 = P 4 ( ) 96 =
Ve grafik :
x 10 9.99 , 10 .. =
4 3 2 1 0 1 2 3 4 5 6
20
16
12
8
4
0
4
8
12
16
20
P x ( )
x
141
stel Fonksiyonlar
stel fonksiyonlar,
f x ( ) b
x
=
olarak tanmlanrlar. Burada b baz olarak tanmlanr, x her zaman olduu gibi bamsz
deikendir. Gerek b gerekse x gerel saylardr. Baz b ise ayrca pozitif, sfrdan ve birden
farkl gerel deerlerdir .
Baz b nin deeri sfr olamaz nk o zaman,
f x ( ) 0
x
= (Eittir Sfr) olur.
Baz b nin deeri 1 de olamaz nk o zaman,
f x ( ) 1
x
= (Eittir Bir)
olur.
Gerek f(x) = 0 , gerekse f(x) = 1 sabit fonksiyonlardr ve deiim
gstermezler.
Ayrca, baz b nin deeri negatif de olamaz nk o zaman sanal fonksiyon
deerleri ile karlalr. rnek olarak,
f x ( ) 4 ( )
x
=
olarak tanmlansa,
f
1
2
|

\
|
|

4 ( )
1
2
= 4 =
olur ki bu sanal bir saydr (2i). Fonksiyon deerlerinde sanal saylar istenmediinden,
negatif bazlar kullanlmaz.
Bir balang olarak , f x ( ) 2
x
= ve g x ( )
1
2
|

\
|
|

x
= fonksiyonlarnn grafiklerini izelim.
ncelikle, g x ( )
1
x
2
x
= olarak ve g x ( )
1
2
x
= ve sonu olarak g x ( ) 2
x
= olarak ifade
edilebilir.
142
10 8 6 4 2 0 2 4 6 8 10
1
0.1
1.2
2.3
3.4
4.5
5.6
6.7
7.8
8.9
10
f x ( )
g x ( )
x
imdi erileri inceleyelim :
f(x) erisi , b deeri ne olursa olsun, daima (0 , 1) noktasn ierecektir. nk, 1.
f 0 ( ) b
0
= 1 = olmaktadr.
Her olas b iin b
x
> 0 olacaktr . Yani, rnek olarak 2
10
1
2
10
= 9.766 10
4
= 2.
(sfra yakn, fakat pozitif bir deer). Bu gzlem, b ninin sfrdan fakl olmas gerektiini
de belirtmektedir. nk bu durumda,
1
0
x
1
0
= Belirsiz = bir durum olacaktr.
Eer 0<b<1 olursa, b
x
erisi soldan saa doru gidildiinde azalan bir eri olacaktr. 3.
Bunu g(x) fonksiyonunun grafiinden gzleyebiliriz.
Eer b>1 olursa, b
x
erisi soldan saa doru gidildiinde artan bir eri olacaktr. Bunu 4.
f(x) fonksiyonunun grafiinden gzleyebiliriz.
143
f(x) fonksiyonunun grafiinden gzleyebiliriz.
Eer b
x
= b
y
olursa, x = y olur. Bunu imdilik dorulamayz. 5.
e Says ve e
x
Fonksiyonu
Matematiin temel saylarndan biri e saysdr. e says irrasyonel bir saydr ve deeri,
e 2.718281828459045 = ... olarak ifade edilebilir.
Matematikte stel Fonksiyon (Eksponansiyel Fonksiyon) olarak, f(x) = e
x
erisi
anlalr. Bazen bu f(x) = exp(x) olarak da belirtilir. imdi, f x ( ) exp x ( ) = ve
g x ( ) exp x ( ) = erilerinin grafiklerini inceleyelim :
10 8 6 4 2 0 2 4 6 8 10
1
0.1
1.2
2.3
3.4
4.5
5.6
6.7
7.8
8.9
10
f x ( )
g x ( )
x
Burada baz b = e olmaktadr. Buna "Doal Temel" (Natural Base) ad da verilir.
144
rnek : q x ( ) 5 e
1 x
4 = fonksiyonunun grafiini iziniz.
2 0.8 0.4 1.6 2.8 4 5.2 6.4 7.6 8.8 10
10
5
0
5
10
15
20
25
30
35
40
q x ( )
x
Bu eri, incelemi olduumuz stel erilerde gzlemlediimiz sonulardan farkl
deerler veriyor. Dorudur, ilk eriler basit stel erilerdi. Buradaki q(x) erisi ise, karmak
bir stel eridir ve grafii, basit stel erilerden farkldr.
Logaritmik Fonksiyonlar
Eer b >0 ve b 1 olacak ekilde herhangibir say ve x>0 ise,
y log
b
x = ifadesi b
y
x = ifadesine edeerdir. lk ifade, "y , x in b temeline logaritmasdr"
eklinde okunur ve "logaritmik ekil" olarak adlandrlr. kinci ifade ise "stel ekil" olarak
adlandrlr.
y log
b
x = ( b 0 > , b 1 ve x>0 )
"x in b temeline gre logaritmas y dir". (logaritmik ekil))
x b
y
= (stel ekil)
145
Tekrar edelim (ezbere bilinecek)
y log
b
x = ise x b
y
= dir.
rnek : 2 saysnn 10 temeline gre logaritmasn hesaplaynz.
zm : Lltfen hesap makinesi veya yeterlibir bilgisayar program kullannz !)
y log
10
2 = log
2 10
y
=
log 2 ( ) 0.30103 =
imdi terminoloji asndan sorunlu bir ksma geliyoruz. Ama nce tarihel bir inceleme
gerekli.
nsanlarn gerek anlamda saysal hesaplara, ancak 9 uncu asrda, Badat
niversitesinde, kltrel olarak Farslam, Trk asll, Horasanl bilim adamlarnn cebir adn
verdileri aratrmalarla baladn ve bu aratrmalarn Avrupada Rnesans (yeniden Dou)
saladn biliyoruz. Fakat, Leibniz in abalar dnda, saylarn ilemlerini yapabilecek cihazlar
yoktu. Mhendislik ise saylarla yaplr. Sonunda kogaritma bulundu ve insanlar 10 temelli saylar
ile altndan 10 temelli logaritmalar kullanld. Logaritmalar aslnda saysal hesap
makinelerinin kt 1980 lere kadar tm bilim dallarnda aktif olarak kullanlmtr. 1980 lerde ise
ad kalmlardr. Logaritmann kullanmnda nceleri log(8) 10 temelli logaritma iin
kullanlm fakat, matematikiler (Jean Napier) bunun iyi bir ey olmadn ve logaritmalarn
doal temelinin e says olmas gerektiini belirtmilerdir. Bu yzden log sylemi bo
olmadndan e tabanna gre logaritmaya doal logatma, (Logaritmus Naturalis) veya
(Logarithme Nperien) ifadesinden kaynaklanan ln terimi kullanlmtr. Bylece ln(8), 8 saysnn
doal logaritmas olarak ifade edilir. Bu durum 1970 lere kadar iyi kt bu ekilde gitti, fakat
1970 lerde Japon transistrlerinden yararlanlarak retilen saysal el hesap makineleri knca,
logaritma bir hesap yntemi olarak kullanmdan kalkmtr. Logaritma artk bir hesaplama
yntemi olarak deerini yitirdiinden ve logaritma cetvelleri kullanmdan kalktndan beri, log ve
ln arasndaki ayrm da gereksiz hale gelmi ve doal logaritma nem kazanmtr. Bugn
alkanlklardan dolay henz 10 tabanna gre logaritmaya log, doal logaritmaya ln denmekte
fakat durumun ksa sre sonra deiecei ve log olarak sadece doal logaritmann kullanlaca
konusunda eilimin yaygnlaaca dnlmektedir. Biz u anda 10 temeline gre logaritmaya
log, doal logaritmaya ln demeye devam ediyoruz.
Bu nedenle ,
log 2 ( ) 0.30103 = 10
0.30103
2 = olmakta,
ln 2 ( ) 0.693 = e
0.693
2 =
146
olmaktadr. leride deiik temelli logaritmalar birbirine dntrme yntemlerini de
greceiz.
rnek : deerini hesaplaynz.
log
4
16
zm :
a log
4
16 = stel ekle dnelim :
16 4
a
=
Bundan sonras kolay deil, nk devam etmek iin, her iki tarafn da a kknn
alnmas gerek
a
16 4 =
Buradadan saysal hesaba devam edemiyoruz nk a deerini bilmiyoruz. Ama
bir snama-yanlma almas yapabiliriz. Sorulacak soru u, acaba hani kk 16 nn deer,i 4
e ettir? Bakalm 16 nn kaekk nedir ? 4 eit mi ?
2
16 4 =
Tam isabet! Arama sonuland aranan a deeri 2 imi, yani
log
4
16 2 =
Grlyor ki, doal ve 10 temelli logaritmalar dndaki temellerle almak kolay olmuyor. En
iyisi temelden temele dnmenin yantemini bir an nce incelemek.
rnek : log 1000 ( ) deerini hesaplaynz.
zm : =n temelli logaritmalarla almak kolay, nk nceden hesaplanm cetvelleri var.
y log
10
10
3
=
imdi soralm : eer 10
3
10
y
=
10
3
10
y
=
ise y katr ? Ak ve seik olarak 3.
147
Buradan eer a
x
= a
y
ise x = y dir olarak belirlenir.
Yant : log(1000) = 3
rnek : log
1
100
|

\
|
|

deerini hesaplaynz.
Yant :
a log
10
1
100
=
1
100
10
a
=
10
2
10
a
=
a 2 =
rnek : ln
1
e
|

\
|
|

deerini hesaplaynz.
zm :
a ln
1
e
|

\
|
|

= a ln e
1
( ) = a 1 =
rnek : ln e ( ) deerini hesaplaynz.
a ln e
1
2
|

\
|
|

=
a
1
2
=
rnek : log
34
34 deerini hesaplaynz.
zm :
a log
34
34 =
34 34
a
=
148
Sonu : Kesinlikle 1. nk sadece 34
1
1 = dir. Bu sonu tm, b b
a
= problemleri iin
geerlidir.
rnek : log
8
1 deerini hesaplaynz.
zm : a log
8
1 = 1 8
a
= Sonu kesin 0 dir. nk sadece 8
0
n
deeri 1 olabilir. Bu sonu tm 1 b
a
= problemleri iin geerlidir.
Not: altmz Mathcad 15, Bu problemleri,
log 8 8 , ( ) 1 =
log 34 34 , ( ) 1 =
log 1 ( ) 0 =
ln
1
e
|

\
|
|

1 =
ln e ( ) 0.5 =
eklinde zer. amzda yntemlerin ok gelimi olduunu bu sonular aklayabilir.
Log(x) ve ln(x) fonksiyonlarnn grafiklerini izelim.
f x ( ) log x ( ) = g x ( ) ln x ( ) =
x 0 0.01 , 10 .. =
149
0 1 2 3 4 5 6 7 8 9 10
5
4
3
2
1
0
1
2
3
4
5
f x ( )
g x ( )
x
Bu grafiklerden de kontrol ederek baz logaritma zelliklerini izleyelim:
1 - b
0
1 = olduundan her durumda, log
b
1 0 = dr.Yani 1 in her temelde logaritmas 0 dr.
2 - b
1
b = olduundan her durumda, log
b
b 1 = dir.Yani, her temelde temelin logaritmas 1
dir..
3- log
b
b
x
x = dir. Buradan, log
b
b
f x ( )
f x ( ) = olarak da belirtilebilir.
4 - b
log
b
x
x = dir. Buradan, b
log
b
f x ( )
f x ( ) = olarak da belirtilebilir.
Temel Deitirme
f x ( ) 3 4
0.7 x
=
g x ( ) 3 e
0.7 ln 4 ( ) x
=
Fonksiyonlarnn grafiklerini izelim :
150
0 0.5 1 1.5 2 2.5 3 3.5 4 4.5 5
0
35
70
105
140
175
210
245
280
315
350
f x ( )
g x ( )
x
Bu iki grafik akk kt Bu ilgin !!!. Fonksiyonlar inceleyelim :
f x ( ) 3 4
0.7 x
=
g x ( ) 3 e
ln 4 ( ) 0.7 x
=
Eer, e
ln 4 ( )
4 = olduu hatrlanrsa f x ( ) ve g x ( )fonksiyonlarnn
Ayn fonksiyonun deiik ekilde yazm olduu ortaya kar.
Bu durumda, tm doal olmayan temelleri, doal temel zerinden yazma olana olduu
grlr.
Bir y = log
b
x ifadesi, x = b
y
ifadesi ile edeerdir. Fakat,
e
ln b ( )
b = olduundan bu ifade, x e
ln b ( ) y
= olarak yazlabilir.
Buradan ln x ( ) ln b ( ) y =
y
ln x ( )
ln b ( )
=
151
olarak elde edilir.
rnek : log
2
23 deerini bulunuz.
zm :
y
ln 23 ( )
ln 2 ( )
4.5235619561 = =
2
4.5235619561
23 =
Sonu : Eskiden sadece doal ve 10 temeline gre logaritma cetvelleri vard. Bunun dnda
mesela 2 temeline gre logaritma cetvelleri yoktu, Bu uygulama ile her temele gre logaritma
deerlerinin hesaplanabildii grlmektedir.
f x ( ) log x ( ) = ve g x ( ) exp x ( ) = fonksiyonlar birbirinin inversi (ters fonksiyonu) dur.
Aadaki grafik bunu belirtmektedir.
0 0.1 0.2 0.3 0.4 0.5 0.6 0.7 0.8 0.9 1
5
4
3
2
1
0
1
2
3
4
5
f x ( )
g x ( )
x
152
Logaritma uzun sre tek saysal hesap yntemi olarak hkm srmtr. te ona bu
yetenei kazandran zellikler:(Bu zellikler iin, x>0 ve y>0 olduu varsaylmaktadr)
1 - log
b
x y ( ) log
b
x log
b
y + =
2 - log
b
x
y
|

\
|
|

log
b
x log
b
y =
3 - log
b
x
r
r log
b
x =
4 - Eer log
b
x = log
b
y ise x = y dir.
Grld gibi ok kolaylk salayan bantlar ama doru kullanmak gerek. Bu
zellikler yznden, insanlar logaritma cetvellerinden yararlanarak saysal hesaplarn yaklak
1970 e kadar logaritma zerinden yrtmlerdir. 1970 den sonra bu i kesinlikle sona ermitir.
Bu zellikler zerinde saysal uygulama yapmayacaz,nk bu samaln artk sona
ermesini istiyoruz. Bir ey mi hesaplayacaksnz? Hesap makinesi veya bilgisayar kullann!
Logaritmaya gereksiniminiz yok.
rnek : k
e
log 2 67 , ( )
32e
5

4
e
3
= deerini hesaplaynz.
k 0.933 = Logaritmaya gereksiniminiz oldu mu ?
stel Denklemlerin zm
stel denklemlerin zm iin, stel terimlerin ortadan kaldrlarak, denklemin
zlebilir duruma getirilmesi gerekir. Bunun iin tel ifadelerin zelliklerinden yararlanlr.
rnek : 5
3 x
5
7 x 2
= denklemini znz.
zm : Bu konuda fazla bir sorun yok, nk, denklemin her iki taraf da ayn temelin sleri
eklinde, b
x
b
y
= ise, b = y olduunu biliyoruz. Bu ekilde,
3 x 7 x 2 = ve x =
1
2
olarak bulunur.
153
rnek : 3
z
3
2
( )
z 5 +
=
Dzenleme : 3
z
3
2 z 5 + ( )
=
slerden Kurtarma : z 2 z 10 + =
zm : z 10 =
rnek : 4
5 9x 1
8
x 2
=
zm: Denklemin her iki taraf da farkl temeller zerinde. Normal olarak, bu gibi
denkelemler zlemez fakat, genelde snavlarda bir zm yntemi olan ifadeler sorulur.
Burada da pf noktas gerek 4 gerekse 8 temelinin iki temelinin katlar olmas bu ekilde bir
dzenleme ile,
2
2 5 9x ( ) 1
2
3 x 2 ( )
=
Hatrlayalm :
a
x 1
a
x
=
154
2
2 5 9x ( )
2
3 x 2 ( )
=
olarak bulunur.
10 18x 3 x 6 + =
Not : Modern bilgisayar programlar (sembolik prosesrler) Bu problemi kolaylkla zerler.
x
4
15
=
2
2 5 9x ( ) 1
2
3 x 2 ( )
=
x iin bilgisayar zm
4
15
Hepsi bu kadar !!
rnek :
5
2 5 9x ( ) 1
7
3 x 2 ( )
=
log 9765625
3814697265625
343
,
|

\
|
|

log 117649
3814697265625
343
,
|

\
|
|

Bunu elle zmek adeta olanaksz !!! Bulduu temellere baknz, sper yani, saysal
sonucu hesaplatmaya alalm :
log 9765625
3814697265625
343
,
|

\
|
|

log 117649
3814697265625
343
,
|

\
|
|

0.191 =
nanlmaz !!!. Bu hesab tutucu matematik hocalarna gstermek gerek.
rnek : 7
x
9 = denklemini znz.
155
zm :
Her iki tarafn doal logaritmasn alalm :
ln 7
x
( )
ln 9 ( ) =
x ln 7 ( ) ln 9 ( ) =
x
ln 9 ( )
ln 7 ( )
=
ln 9 ( )
ln 7 ( )
1.129 =
x 1.129 =
rnek : 2
4y 1 +
3
y
= denklemini znz.
zm :
Her iki tarafn doal logaritmalarn alalm : ln 2
4y 1 +
( )
ln 3
y
( )
=
Dzenleyelim : 4y 1 + ( ) ln 2 ( ) y ln 3 ( ) =
4y 1 + ( )
y
ln 3 ( )
ln 2 ( )
=
4 y
y
1
y
+
ln 3 ( )
ln 2 ( )
=
4
1
y
+
ln 3 ( )
ln 2 ( )
=
1
y
ln 3 ( )
ln 2 ( )
4 =
y
1
ln 3 ( )
ln 2 ( )
4
|

\
|
|

=
y 0.414 =
156
rnek : e
t 6 +
2 = denklemini znz.
Yant : Her iki tarafn doal logaritmas alnr : ln e
t 6 +
( )
ln 2 ( ) =
Hatrlayalm : ln e
f x ( )
( )
f x ( ) =
t 6 + ln 2 ( ) =
t ln 2 ( ) 6 =
t 5.307 =
rnek : 5e
2z 4 +
8 = denklemini znz.
Dzenleyelim : e
2z 4 + 8
5
=
Her iki tarafn doal logaritmasn alalm : ln e
2z 4 +
( )
ln 8 ( ) ln 5 ( ) =
2z 4 + ln 8 ( ) ln 5 ( ) =
z
ln 8 ( ) ln 5 ( ) 4
2
=
z 1.765 =
Logaritmik Denklemlerin zm :
Logaritmik denklemlerin zm eksponansiyel denklemlerin zmlerine benzer.
rnek : 2 log
9
x ( ) log
9
6 x 1 ( ) 0 = denklemini znz.
Dzenleyelim : log
9
x ( )
2
log
9
6 x 1 ( ) =
log
9
x log
9
6 x 1 ( ) =
x 6 x 1 =
157
x
1
5
=
rnek : ln x ( ) ln x 1 ( ) + ln 3x 12 + ( ) =
Dzenleyelim :
ln x x 1 ( ) [ ] ln 3x 12 + ( ) =
x x 1 ( ) 3x 12 =
x
2
x 3x 12 + =
x
2
x ( ) 3x 12 + ( ) 0 =
x
2
4 x 12 0 =
2
6
|

\
|
|

imdi yaplmas gereken ey mutlaka bu iki kk orijinal denklemde deneyip negatif sonu
vermeyeceklerinin kontoln yapmaktr.
x 2 =
ln x ( ) ln x 1 ( ) + ln 3x 12 + ( ) =
ln 2 ( ) [ ] 0.693 3.142i + =
x 2 =
x 6 =
zm deil !
ln x ( ) ln x 1 ( ) + 3.401 =
ln 3x 12 + ( ) 3.401 =
x 6 = tek zm!!
Matris Notasyonu
158
Stun 1 Stun 2 Stun 3
Sra 1 a
11
a
12
a
13
Sra 2 a
21
a
22
a
23
Sra 3 a
31
a
32
a
33
ki Sral Determinantlar
Matrislerin en basit uygulamalar iki sral determinantlardr.
Stun 1 Stun 2
Sra 1 a
11
a
12
Sra 2 a
21
a
22
Burada,
a
11
a
22
deerlerine, ana diyagonal,
159
a
12
a
21
deerlerine tali (ikincil) diyagonal denilir.
Determinant deeri,
Ana diyagonal deerleri arpm - tali diyagonal deerleri arpm
a
11
a
12
a
22
- a
21
olarak hesaplanr.
rnek :
Det1
12
46
3
5
|

\
|
|

= olarak tanmlanyor, determinant deerini


Hesaplaynz.
zm :
160
Det1 = Ana diyagonal deerleri arpm - tali diyagonal deerleri arpm
Det1 12 5 ( ) 46 3 ( ) = Det1 78 =
Bu kadar basit !!!
Sral Determinantlar
sral determinantlar prensip olarak ayn iki sral determinantlar gibi zmlenir.
Burada sadece minr kavramnn zmsenmesi gerek, hepsi bukadar.
Bir elemann minr arpld ikili determinanttr.
l bir determinantn yazlm,
a
11
a
12
a
13
a
21
a
22
a
23
a
31
a
32
a
33
eklindedir. Burada ilk satrn ilk eleman a
11
, nndeki minr
a
22
a
23
a
32
a
33
161
eklindedeki ikili determinanttr. Bu minre M1 diyelim. lk arpan
a
11
M1 olur.
lk satrn ikinci eleman a
12
nndeki minr :
a
21
a
23
a
31
a
33
dir. Bu minre de M2 diyelim. kinci arpan deeri,
a
12
M2 olur.
lk satrn nc eleman a
13
nndeki minr :
a
21
a
22
a
31
a
32
dir. Bu minre de M3 diyelim. nc arpan deeri,
a
13
M3 olur. l determinantn deeri ise
Det = Birinci arpan Deeri - kinci arpan Deeri + nc arpan Deeri
162
Det a
11
M1 a
12
M2 a
13
M3 + =
olarak hesaplanr. Bir rnek yapalm
rnek : Det2
22
56
48
2.95
12
46
3.77
3
5
|

\
|
|
|

= olarak tanmlanyor, determinant deerini


Hesaplaynz.
zm :
Det = Birinci arpan Deeri - kinci arpan Deeri + nc arpan Deeri
Det a
11
M1 a
12
M2 a
13
M3 + =
M2
56
48
3
5
|

\
|
|

= M2 136 =
M3
56
48
12
46
|

\
|
|

= M3 2 10
3
=
56 46 48 12 2 10
3
=
a
11
M1 22 78 = 22 78 1.716 10
3
=
163
a
12
M2 2.95 136 ( ) = 2.95 136 ( ) 401.2 =
a
13
M3 3.77 2000 = 3.77 2000 7.54 10
3
=
Determinant 1.716 10
3
401.2 ( ) 7.54 10
3
+ =
Determinant 9.657 10
3
=
Det2 9.657 10
3
= (Bilgisayar Sonucu)
Matrislerin determinant deerleri, gnmzde bilgisayar programlar ile hesaplanr. l kare
matrislerin determinantlarndan daha yukar sayda determinantlarn hesaplanmas pratik deildir.
Matrislerin Elemanter Satr lemleri
Matrislerin elemanter satr ilemleri toplam olarak tane ilemdir. Bunlar,
Bir satrn baka bir satr ile deiimi 1.
Bir satrn sabit bir say ile arpm 2.
Bir satrn ilem grm eklinin baka bir satra eklenmesi 3.
Bir satrn baka bir satr ile deiimi ok basittir. Bu konuda kullanacamz notasyon,
R
i
R
j
.
Bunun anlam i satrnn j satr yer deitirmesidir. R (Row) satr anlamndadr.
rnek : Aadaki matrisin birinci satr ile nc satrnn yerlerini detirininiz ve Matrisin orijinal
ve deimi hallerinin determinant deerlerini hesaplaynz.
Mat1
22
26
3.56
8.45
5
8.66
16
44.9
97.16
|

\
|
|
|

=
164
Mat1 3.535 10
4
=
R
1
R
3
.
Mat2
3.56
26
22
8.66
5
8.45
97.16
44.9
16
|

\
|
|
|

=
Mat2 3.535 10
4
=
Satr deiimi sonunda, matrisin determinant deeri deiiyor.
Bir satrn sabit bir say ile arplmas ok basittir. Bu konuda kullanacamz notasyon,
cRi veya blme yaplacaksa
1
c
R
i
olabilir.
rnek : Mat2 matrisinin nc satrn 16 ile arpp matrisin yeni determinant deerini
hesaplaynz.
zm :
Mat2
3.56
26
22
8.66
5
8.45
97.16
44.9
16
|

\
|
|
|

=
Mat2 3.535 10
4
=
16 R
3

16 22 352 =
8.45 16 135.2 = 16 16 256 =
Mat3
3.56
26
352
8.66
5
135.2
97.16
44.9
256
|

\
|
|
|

=
Mat3 5.656 10
5
=
165
Mat3
Mat2
16 =
Deer deiti.
Bir satra n ilemler yapp baka bir satra ilemek son derece basittir. n ilemler
sabit saylarla arpma blme gibi ilemler olabilir. Bu konuda kullanlacak notasyonun ilemleri
tam tanmlayabilmesi nemlidir.
rnek olarak bir matrisin j satrn sabit bir say ile arpp, i satr ile toplayp oluan elemanlar
i satrnn yerine yerletirmeyi dnelim. Bu konuda kullanlabilecek notasyon,
R
i
+ cR
j
R
i
.
olabilir. Bu konuda bir uygulama yapalm.
rnek : Mat3 matrisinin ikinci satrn 45 ile arpp birinci satra ekleyelim ve oluan elemanlar
birinci satr ile yer deitirelim.
zm:
Mat3
3.56
26
352
8.66
5
135.2
97.16
44.9
256
|

\
|
|
|

=
R
1
+ 16R
2
R
1
.
nce ikinci srann elemanlarn 16 ile arpalm:
26 16 416 = 5 16 80 = 44.9 16 718.4 =
Bu deerleri, ilk srann elemanlar ile toplayalm:
3.56 ( ) 416 + 412.44 = 8.66 ( ) 80 ( ) + 88.66 =
166
97.16 718.4 ( ) + 621.24 =
Oluan elemanlar ilk srann elemanlar ile yer deitirelim.
Mat4
412.44
26
352
88.66
5
135.2
621.24
44.9
256
|

\
|
|
|

=
Mat4 5.656 10
5
=
Mat3 deeri deimedi.
Matrislerin Baz Determinant Deerleri zellikleri :
Eer bir matrisin iki iki satr veya stunu birbirine eitse, o zaman matrisin determinant 1.
deeri 0 dr.
Eer bir matrisin satr veya stunlar, ayn sabit rakkamla arplrsa matrisin determinant 2.
deeri bu sabit kadar artar.(kinci uygulama)
Eer bir matrisin satr veya stunlarnn birine, baka bir satr veya stunun bir sabit say 3.
ile arplm elemanlar eklenirse, matrisin determinant deeri deimez. (Son uygulama)
Matrislerin Inversleri
Matrislerin Inversleri en son inceleyeceimiz matris konusudur. Bir saynn arpm inversi,
2
1
2
1 =
olarak tanmlanr ve inverse 2
1
1
2
= olarak belirtilir. Sfr dnda tm saylarn inversleri
vardr. Matris inversleri de benzer ekilde tanmlanr. Bir A matrisinin inversi yle bir A-1
matrisidir ki, A*A-1 = I (Birim Matris) verir.
Birim matris, ana diyagonalinin tm elemanlar 1 olan ve boyutlar orijinal matrise eit
olan bir matristir.
Bir kare matris A, eer determinant A 0 olduunda invert edilebilir. Invert
edilebilen matrislere Invertibl, invert edilemeyen matrislere Singular (Tekil) matris ad verilir.
Tanm kolay, uygulama zor.
Matris inversiyonu, yntemleri belli olan ok kolay ilemlerdir. Ne var ki bu ilemler
ok emek youndur. yle ki, elle bu almalara giriildiinde hata yapma olasl ok
yksektir. Bu yzden gnmzde, kimse bir matrisi elle invert etmek cahilliinde bulunmaz.
Bunun iin dnya kadar bilgisayar program bulunmaktadr. Matris inversiyon yntemlerine
sadece matris inversiyon rutinleri yazacak programclarn gereksinmesi vardr. Bizim
gereksinmemiz sadece invert edilmi matristen ibarettir ve onu da elimizdeki bilgisayar
167
gereksinmemiz sadece invert edilmi matristen ibarettir ve onu da elimizdeki bilgisayar
program yardm ile kolayca gerekletirebiliyoruz.
rnek : Aada verilen Mat5 matrisinin inversini alnz ve sonucun salamasn yapnz.
Mat5
23
56
99
66
39
75
88
55
46
|

\
|
|
|

=
Mat5
1
0.014
0.017
2.047 10
3

0.022
0.046
0.029
1.196 10
3

0.022
0.017
|

\
|
|
|
|

=
Mat5 Mat5
1

1
0
0
0
1
0
0
0
1
|

\
|
|
|

=
Elle yapmaya kalksaydk hap yutardk !!!
Matris matematii zerine gereksinmemiz olan balang bilgileri bu kadar. Bu
bilgileri iyi zmseyelim, konularda gereksinmemiz olacak.
Lineer Denklem Sistemleri
Doa bilimcileri srekli olarak doadaki olaylar inceler ve bu olaylarn matematiksel
olarak ifade edilebilecek sistematiini bulmaya alrlar. Buna genel olarak "Matematik
Modelleme" denilir ve gnmzde matematiin renimi iin en nemli nedenlerden biridir.
rnek olarak bir kazanda bir kimyasal reaksiyon gerekletirilecek olsun. Bu reaksiyonun
asit alanda (pH<7) oluaca ve ortam ne kadar asitli olursa olsun, reaksiyon verimini ve hzn
etkilemedii anlalm olsun. Fakat asit pahaldr ve gerektiinden fazlasn kullanmak hem
ekonomik deildir, hem de evre sal asndan asitli bir atk atlmasna msaade
edilmediinden letmeye ek artma yk getirir.
Bu durumda yle bir nokta aranr ki, hem reaksiyon olsun, hem de ortam ar asitli
olmasn. Sistem olanaklar lsnde deikenler dnda sabit hale getirilmeye allr. Sistem
hacm yaklak 10 metrekptr ve eklenecek az miktarda asit hacmnn sistem hacm yannda
ihmal edilebilecek kadar az olaca grlr. Yani V bir parametre deil bir sabittir. D basn 1
atm, (1013 milibar) scakl sabit yaklak 25 C , olarak belirlenir deieken parametre olarak
eklenen asit hacm (ml) ve sistem pH deeri sabit kalr. Tm dier byklkler sabit iken, kazana
belirli ml asit eklenir ve kazan pH s llr. bir pH/ml grafii elde edilir. Dier taraftam
reaksiyonun olmas iin en uygun pH da aranr ve en uygun pH deerinin 4 olduu gzlenir.
Yani reaksiyon pH s sabit bir deerdir.
Bundan sonra pH/ml deerleri incelenir. Elde sadece deneysel noktalar var fakat bu
noktalar birbirine balayan matematiksel iliki yok. Bunun iin eri uydurma denilen
168
noktalar birbirine balayan matematiksel iliki yok. Bunun iin eri uydurma denilen
yntemlerle, deneysel noktalara matematiksel iliki oluturmaya allr. Aratrmalar, aranan
ilikinin dorusal (lineer) olduunu gsterir. Aratrma ekibi rahat bir nefes alr. Artk i hemen
hemen bitmitir.
Bundan sonra kartezyen ortamda pH/ml grafii izilir. Bu lineer azalan bir eridir. (ml
arttka pH der). Dier taraftan pH eksenine pH=4 noktasndan bir dik ekilir bu pH= 4
erisidir.
En uygun ml deeri reaksiyon pH sn (4) salayan ml deeridir. ki erinin birbini
kestii yerde her iki deikenin ortak deerinin saland noktadr. Bu deer, optimum pH iin
gereken ml deerini verir. Bylece optimum (en uygun) iletme koullar belirlenmi olur.
Zaman iinde aratrma ekibi bu matematik modeli bir uluslaras konferansa sunmaya
karar verir. Bildiri kabul edilir. Sunum srasnda dinleyiciler, eri uydurma iinde yetersizlik
olduunu farkeder ve sunucular topa tutarlar. Sunucular yeterli matematik renimi
yapmadklarna bin piman olur ve derhal eksikliklerini gidermek iin almaya balarlar.
Matematik modelin yaym ok daha baarl olur. Aslnda bilimsel kongrelerin yarar da budur.
Sunucular uyarlr, hatalar belirtilir ve iin daha baarl olmas iin ilk uyarlar verilmi olur.
Matematik modele geri dnelim, eri uydurma sonunda, pH/ml erisinin lineer olduu
ortaya kt iin erinin ilikisi yledir :
pH m ml n + =
Azalan bir eri olduu iin saysal deerler yle kmtr :
pH 0.23 ml 7.0 + =
Optimum reaksiyon pH ise bir sabittir:
pH 4 =
imdi bu denklemleri, kartezyen grafikte gsterbilmek iin iki fonksiyon halinde dzenleyelim.
f ml ( ) 0.23 ml 7.0 + =
g ml ( ) 4 =
Bu durumda f(x) ve g(x) ortak ordinat eksenini (pH eksenini), ml deerleri de absis (x ekseni) ni
paylaacaklar. Grafii grelim:
169
0 2 4 6 8 10 12 14 16 18 20
0
1.4
2.8
4.2
5.6
7
8.4
9.8
11.2
12.6
14
f ml ( )
g ml ( )
ml
Sonu ok ok ilgin. Sistem zm yani iki dorunun birbirini kestii noktada, benim
grafikten grdm yaklak 13 ml. Mhendislik almas asndan bunu hata pay iin biraz
arttrp 14 ml olarak kullanabiliriz.
Bu doal olarak yaklak bir zm. Elde saysal deerler olduktan sonra biraz sonra
greceimiz gibi kesin deerler verecek analitik yntemler de kullanabiliriz. Ama unutmamak
gerek ki, bu saysal verileri aldmz eri uydurma ilemleri de yaklak saysal sonu veriyor.
Yani veriler yaklak ise zm de yaklak olabilir hatta olmaldr da. Eer yaklak verilerden
(lmlerde daima bir hata pay vardr) lgnca hassas matemetik yntemler uygulayp
ahkam kesmeye kalkarsak bizi yine bombardmana tutarlar. Yani matematiki-fizikiler,
yaptklar ilemlerin nn-ardn son derece detayl dnmeli, yetersiz olduklar alanlarda
mutlaka uzman tavsiyeleri almaldrlar. Bilim, bir ortak almadr.
Artk lineer sistemlerin saysal zmlerine geebiliriz. Ama bu sefer daha genel
alacaz.
ki Deikenli Lineer Sistemler
iki deikenli lineer sistemler, iki bamsz denklemde, iki deiken ieren lineer
denklem takmdr. Bir denklem sistemi eer denklemlerde deikenler zerinde s bulunmazsa,
deikenler sadece pay zerinde bulunursa ve deiken arpm ieren terimler bulunmazsa
lineer olarak nitelendirilir.
ki deikenli lineer bir denklem sistemi, aada grld gibi formile edilebilir:
a
11
x
1
a
12
x
2
+ a
13
=
+ =
170
a
21
x
1
a
22
x
2
+ a
23
=
ifade ekli bizi artmasn burada x
1
= y ve x
2
= x denilirse, geleneksel x-y denklem sistemi
olduu grlr.
Bu denklem sisteminin biraz nce grdmz ve prensip olrak yaklak grafik
zmnden baka, analitik (kesin sonulu) saysal zm yntemleri vardr. Bu yntemler, ikili
denkelem sistemlerine uygulanan
Yerine yerletirme (substitution) yntemi. 1.
Eliminasyon yntemi. 2.
olarak belirtilir. Daha yksek sayda deiken/denklem ieren sistemlere uygulanan baka
yntemler de var. Onlar ileride inceleyeceiz.
Yerine Yerletirme (Substitution) Yntemi
Bu yntem ok basittir ve her sayda lineer denklem sistemlerine kolayca uygulanabilir. lk
nce ilk denklemden x
1
'in deeri bulunur. Sonra ikinci denklemde yerine yerletirilir. kinci
denklemde tek kalan deiken x
2
bulunur. Bulunan x
2
nin deeri, ilk denklemde kullanlarak x
1
de
bulunur. lemlerin yry :
a
11
x
1
a
12
x
2
+ a
13
=
a
21
x
1
a
22
x
2
+ a
23
=
--------------------------
a
11
x
1
a
11
a
12
x
2
a
11
+
a
13
a
11
=
x
1
a
13
a
11
a
12
x
2
a
11
=
-------------------------------
a
21
a
13
a
11
a
12
x
2
a
11

\
|
|

a
22
x
2
+ a
23
=
a
22
x
2

a
13
a
21

a
11
+
a
12
a
21
x
2

a
11
a
23
=

171
a
22
x
2

a
12
a
21
x
2

a
11
a
23
a
13
a
21

a
11
=
a
11
a
22
a
12
a
21
( ) x
2

a
11
a
23
a
13
a
21

a
11
=
a
11
a
22
a
12
a
21
( ) x
2
a
23
a
11
a
13
a
21
=
x
2
a
23
a
11
a
13
a
21

a
11
a
22
a
12
a
21

=
----------------------------------------------
a
11
x
1
a
12
a
23
a
11
a
13
a
21

a
11
a
22
a
12
a
21

+ a
13
=
x
1
a
12
a
11
|

\
|
|

a
23
a
11
a
13
a
21

a
11
a
22
a
12
a
21

|

\
|
|

+
a
13
a
11
=
x
1
a
13
a
11
a
12
a
11
|

\
|
|

a
23
a
11
a
13
a
21

a
11
a
22
a
12
a
21

|

\
|
|

=
rnek : 2 y 12 x 21 =
7 y 5 x + 16 =
denklem sistemini znz.
zm: Tm zm yntemi oluturulmu durumda, tek yaplacak i, saylar deerleri
yerletirmek.
a
11
2 = a
12
12 = a
13
21 =
a
21
7 = a
22
5 = a
23
16 =

172
y
a
13
a
11
a
12
a
11
|

\
|
|

a
23
a
11
a
13
a
21

a
11
a
22
a
12
a
21

|

\
|
|

=
y 3.16 =
x
a
23
a
11
a
13
a
21

a
11
a
22
a
12
a
21

=
x 1.223 =
salama : 2 3.16 12 1.223 ( ) 21 4 10
3
= (ondalk hatas)
a
11
y a
12
x + a
13
0 = (zm Doru)
a
21
y a
22
x + a
23
0 =
(zm
Doru)
Not :
Bu yntem bir kez gelitirildikten sonra bir bilgisayara programlanr ve tm iki bilinmeyenli 1.
denklem takmlarna uygulanabilir. Bu saysal rnekte de bu yntem kullanlmtr.
Bu gelitirme sonucunda bulunan algoritmalar, sadece 2.
a
11
x
1
a
12
x
2
+ a
13
=
a
21
x
1
a
22
x
2
+ a
23
=
eklinde dzenlenmi matematik model iin geerlidir. Eer veriler,
12y 34x 3 0 =
eklinde verilmise, ncelikle verinin
12y 34x 3 =
ekline konulmas ve deiken deerlerini veren yukarda gelitirilmi algoritmalarn bundan
sonra uygulanmas gerekir. Eer baka bir matematik model kullanlyorsa, tm deiken
deerlerinin bu modele gre yeniden hesaplanmas gerekir. Eer baka bir model iin
gelitirilmi algoritmalara baka bir modelin verileri uygulanrsa sonu yanl kar.
173
gelitirilmi algoritmalara baka bir modelin verileri uygulanrsa sonu yanl kar.
3 . Bilgisayarlarla ok sayda arpma blme, toplama zellikle karma gibi ilemler yaplrken
ara sonularn ifadesi iin kullanlan ondalk says yetersizse sonular zerinde hatalar
bymeye balar. Bilgisayar almalarnda en byk sorunlardan biri budur fakat bizim
bamza farkl bir ey gelmitir. Bilgisayarn gerekte hesaplad deer daha yksek ondalk
says ierirken grntlenen ondalk says sadece 3 le snrlandrlmtr. Gerek deerleri
grnt formatlarn deitirerek grntlemek ve dorulamada kullanmak gerekir. Burada
dorulamada gerek sonular kullanlnca dorulama gerek sonu ne ise onu vermitir.
4. Denklem sistemlerinin zm sonrasnda elde edilen deerlerin verilerle tutarl olup
olmadnn belirlenmesi iin, mutlaka salamalar yaplmaldr.
5. zm srasnda hata birikimi yakndan izlenmeli ve hatalarn allan verilerin dzeyinden
byk olup olmad alnan sonularda hata paynn ne olduunun izlenmesi, hatta baz
katsaylar azaltp oaltlarak verilerin dzeyinin sonulara nasl etki ettii saptanmal, yani her
ey kontrol altnda tutulmaldr.
Uygulamay, bir de sonularn grafik olarak incelenmesi ile kapatmak doru olur.
f x ( ) 6 x
21
2
+ =
x 1000 999.001 , 1000 .. =
g x ( )
5 x
7
16
7
+ =
f 10 ( ) 49.5 =
f 10 ( ) 70.5 =
174
10 8 6 4 2 0 2 4 6 8 10
10
8
6
4
2
0
2
4
6
8
10
f x ( )
g x ( )
x
zm (-1 , 3)
(Yaklak zm)
Grafik zm saysal zm doruluyor fakat doal olarak, grafik sadece saysal olarak
hesaplanm deerlerin grsel olarak dorulanmas asndan ie yarar. Saysal olarak
hesaplanmam deerlerin bilimsel almalarda kullanlmas sz konusu deildir. Yani
herey cebirde biter. Fakat, sonularn, verilerle tutarl olmayan yapay duyarllklarla
ifadesi, okuyucuyu bilerek veya bilmeyerek yanltmak anlamna gelir. nce veriler iyi
incelenecek ve duyarlklar belirlenecek, sonra hesaplar, olabildiince duyarl olarak
hesaplatlacak (yani deneysel hatann stne bir de gereksiz saysal hata eklenmeyecek),
fakat sonular belrtilirken kesinlikle verilerin tadndan daha yksek bir duyarlkla
sonu verilmesinden kanlacaktr. rnek, lebildiimiz en kk hacm 0.05 ml dir (
oda pahal saysal bretlerle), hesap sonucu 12.8816 ml karsa ve siz de bunu
belirtirseniz okuyucular sizin iin kt fikirlere sahip olurlar. Baktnz 12.8816 ml kt,
bunu llebilecek 12.9 ml olarak ifade etmek gerekir. Her ite de byle...
rnek :
3 x y 7 =
2 x 3 y + 1 =
sistemini znz.
zm : Hereyden nce, verileri, gelitirdiimiz matematik modele uygun olacak ekilde
dzenlemeliyiz. Yoksa gereksiz yere, bu verilere gre yeni bir yntem gelitirmek zorunda
kalrz.
175
y 3x + 7 =
--------------------
y 3x 7 =
x
1
3x
2
7 =
3 y 2x + 1 = 3x
1
2x
2
+ 1 =
Katsaylar tantrsak zm hazrdr :
a
11
1 = a
12
3 = a
13
7 =
a
21
3 = a
22
2 = a
23
1 =
y
a
13
a
11
a
12
a
11
|

\
|
|

a
23
a
11
a
13
a
21

a
11
a
22
a
12
a
21

|

\
|
|

=
y 1 =
x
a
23
a
11
a
13
a
21

a
11
a
22
a
12
a
21

=
x 2 =
salama :
a
11
y a
12
x + a
13
0 = (zm Doru)
a
21
y a
22
x + a
23
0 =
(zm
Doru)
Grafik Kontrol :
176
Not : Modern bilgisayar programlar bu gibi denklemleri, lgnca kolay zebilirler.
M
1
3
3
2
|

\
|
|

=
v
7
1
|

\
|
|

=
soln lsolve M v , ( ) = soln
1
2
|

\
|
|

=
Bu durumda biz boa m uram oluyoruz ?
Yant : Kesinlikle hayr. Burada olan ey birilerinin gelitirdiimiz sistemi daha nce gelitirip
bilgisayara yklemi olmasndan ibaret. Biz istersek kendi sistemimizi istersek daha nce
yklenmi sistemi kullanabiliriz. Sorun yok !!!
Sorun , nceden yklenmi birok rutini kullanabilecek iken niin kendimiz altmzdr?
Bunun yant kolay deildir ve ne amala altmz, kimliimizi iereren geni bir alan
kapsar.
Bir kere matematik almann amac kendini gelitirmektir ve her ii bilgisayaralara brakmak
ok yanltr. nsann bilgisayarsz kalmas olas deildir. Fakat bilgisayarlarn altmz
konuda programlanmam olmas veya olan programn bizi tatmin etmemesi kuvvetle olasdr.
O zaman i baa decektir ve o zaman kendimizin gelimi olmas gerekecektir. Yaam
altmz basit problemlerden olumuyor. Daha karmak sorunlar var.
Ben genellikle hem kendim zyor hem de bilgisayarla kontrol ederek doru zm yapp
yapmadm kontrol ediyorum.
177
yapmadm kontrol ediyorum.
Artk, trev, limit integral gibi basit eyleri kendim yapmyorum bilgisayara yaptryorum. Ama
bunlarn uyguland matematik yntemleri en ince noktasna kadar kendim gelitiriyorum.
Yani entellektel ksm ile ben ilgileniyorum, iin emek youn ksmnda bilgisayardan
yararlanyorum.
Ama bu benim tarzm ve benim kiiliim. Olaylar bir bilim adam gz ile yorumluyorum.
Belki, salt bir mhendis olsam, hazr bilgisayar rutinleri bana yetip artacakt. Sylediim gibi,
bu kiilikle ve amalarla ilgili bir sorun...
rnek :
4 y 5 x + 1 =
6 y 3 x 2 =
sistemini znz.
zm :
Katsaylar tantrsak zm hazrdr :
a
11
4 = a
12
5 = a
13
1 =
a
21
6 = a
22
3 = a
23
2 =
y
a
13
a
11
a
12
a
11
|

\
|
|

a
23
a
11
a
13
a
21

a
11
a
22
a
12
a
21

|

\
|
|

=
y 0.167 =
x
a
23
a
11
a
13
a
21

a
11
a
22
a
12
a
21

=
x 0.333 =
salama :
a
11
y a
12
x + a
13
0 = (zm Doru)
a
21
y a
22
x + a
23
0 = (zm Doru)
Grafik Kontrol :
178
f x ( )
1
4
5 x
4
=
g x ( )
x
2
1
3
= x 10 9.9999 , 10 .. =
0 0.1 0.2 0.3 0.4 0.5 0.6 0.7 0.8 0.9 1
1
0.9
0.8
0.7
0.6
0.5
0.4
0.3
0.2
0.1
0
f x ( )
g x ( )
x
Saysal zm grsel olarak da dorulanyor.
Eliminasyon (ortadan kaldrma) yntemi
Denklem sistemlerinin zm iin, uygulanabilecek bir baka yntem eliminasyondur.
Bu konuda bir rnekle birlikte alalm. Son saysal rnek,
179
4 y 5 x + 1 =
6 y 3 x 2 =
eklindeydi. Burada tm sistemde y katsays ayn olsun istersek, birinci denklemi, 3 ile ikinci
denklemi ise 2 ile arpacaz,
3 4 y 5 x + ( ) 1 3 = 12 y 15 x + 3 =
2 6 y 3 x ( ) 2 2 = 12 y 6 x + 4 =
--------------------------------------------
Taraf tarafa toplayalm : 21 x 7 =
x
7
21
=
1
3
= x
1
3
=
x 0.333 =
Geri yerletirme:
12 y 6
1
3
+ 4 =
Buradan,
y
1
6
= y 0.167 =
olarak bulunur.
ki bilinmeyenli denklem sistemlerinin saysal zmnde , elle zm yapldnda,
ounlukla ilk metot yani yerine yerletirme metodu uygulanr. Bilgisayar kullanldnda,
hangi programa eriim salanabilirse o program kullanlr, metodun nemi yoktur.
ki bilinmeyenli sistemleri tamamam oluyoruz. Fakat baz zel durumlar incelemek
gerekli olacaktr.
zel Durumlar
180
y x + 6 =
2 y 2 x 1 =
Bu denklem sistemini zmeye alalm. Yerine koyma yntemini uygulayalm.
y x 6 =
ikinci denklemde yerletirelim:
2 x 6 ( ) 2 x 1 =
12 1 = Felaket , Deiken kayboldu, imdi bunu nasl zeceiz?
lk i grafikten bunun nasl bir sistem olduunun incelenmesidir.
f x ( ) x 6 =
g x ( )
1 2 x +
2
=
x 10 9.999 , 10 .. =
181
3 2.4 1.8 1.2 0.6 0 0.6 1.2 1.8 2.4 3
10
8.7
7.4
6.1
4.8
3.5
2.2
0.9
0.4
1.7
3
f x ( )
g x ( )
x
imdi anlald. Matematikiler hata yapm !!!
Bu iki denklemi oluturan lineer fonksiyonlar birbirine paralel, bu durumda, bu iki denklemin
ortak zm yok. Bu durumda tutarl bir sistem de yok. Bu sistem tutarsz. Yani sistemi
oluturan denklemler birbirinden bamsz deil. Yani matematikiler, modeli olutururken,
bamsz sandklar bir ilikiyi kullanmlar ama bu iliki bamsz deilmi. Bazen insanlar iyi
matematiki ama kt doa bilimci olunca byle eyler yaanr. Tanr korusun !!!
imdi baka bir denklem takmn inceleyelim.
y
2 x
5

1
5
=
2x 5y + 1 =
10 x 25y 5 = y
2 x
5

1
5
=
Her iki denklem de birbirinin ayn bu tek br denklem!!!
Burada baka bir sorun var. Sistemde iki tane deiken var (x ve y). Ama sadece bir tek
denklem tanmlanm. Oysa, tutarl iki deikenli bir sistem iin iki tane birbirinden bamsz
denklem gerekir. Oysa burada sadece bir tane denklem var!!!.
Ne yaplmas gerek? Hereyden nce, matematik modeli yapanlarn, x ve y deikenleri
arasnda, bir baka bamsz iliki bulmalar gerekli. Umarz bulurlar. Bulamazlarsa, bu sistem
eksik tanmlanm bir sistem olarak nitelendirilir. Eksik tanmlanm sitemler, sistemde bulunan
deiken saysndan daha az sayda bamsz denklem ieren sistemlerdir.
182
deiken saysndan daha az sayda bamsz denklem ieren sistemlerdir.
Eksik tanml sistemler tek bir zmden yoksundur. Bu sistemlerin sonsuz zm
vardr. rnek olarak altmz bu sistemd, iki deiken iin sadece tek bir denklem
tanmlanabilmitir. Deikenlerden birinin her deeri iin denklemi salayacak dier deikenin
ayrk bir deeri vardr. Deikenlerden birine parametre ad verilir. Parametrenin sabit bir
deeri yoktur. Kullanclar, kendi isteklerine gre parametreye deerler verirler ve baml
deiken de bu deerleri iin ayrk bir deer alr. Sistemin sonsuz zm vardr, e yarar
veya yaramaz, o ayr bir sorun.
Bilinmeyenli Lineer Denklem Sistemleri
Gerek lineer denklem sistemleri, bilinmeyenli denklemlerle balar. ki bilinmeyenli iki
denklem aslnda oyuncak gibi bir sistemdir. Fakat sistemde bilinmeyen iin bamsz
denklem olunca iler ciddileir.
bilinmeyenli denklemden oluan tural sistemleri incelemek iin saysal bir rnekten
yararlanmak uygun olabilir. rnek olarak kullanacamz sistem,
x 2y 3 z + 7 =
2x y + z + 4 =
3 x 2y + 2z 10 =
denklem sistemidir.
Bu denklem sistemi zorda kalnrsa, yerine koyma yntemi ile zlebilir, fakat analitik
bir bant oluturmak iin ne yerine koyma ne de eliminasyon yntemi kullanlmaz. Bu
konuda ok terimli karmak bantlarla uramak gerekir ki bunlarla hata yapma olasl
yksektir.
Bu konuda, uygulayabileceimiz nc yntem, Cramer Kural olarak adlandrlr.
Cramer Kural
Cramer Kural, bir matris yntemidir. lk olarak sistemin matrisi yazlr. Sistem matrisi,
katsaylar matrisidir. Buna Det adn verebiliriz.
Det
1
2
3
2
1
2
3
1
2
|

\
|
|
|

=
Bu determinantn deerini hesaplamak kolay,
=
183
Det 15 =
Bundan sonra, hesaplanacak her deiken iin, deikenin stnu yerine sa taraf stununu
koyarak tane matris yazarz. Bunlara, Detx,Dety, Detz diyebiliriz. Her birinin determinant
deerini hesaplamalyz.
Detx
7
4
10
2
1
2
3
1
2
|

\
|
|
|

=
Detx 30 =
Dety
1
2
3
7
4
10
3
1
2
|

\
|
|
|

=
Dety 15 =
Detz
1
2
3
2
1
2
7
4
10
|

\
|
|
|

=
Detz 15 =
imdi her deikenin deeri,
184
x
Detx
Det
= x
30
15
= x 2 =
y
Dety
Det
= y 1 =
z
Detz
Det
= z 1 =
olarak bulunur. Salama yaparken, saysal hatalardan kanmak iin, grntlenen deerleri
deil,bilgisayarn belleindeki deerleri kullanmak daha doru olur.
Salama
Det
0 0 ,
Detx
Det
Det
0 1 ,
Dety
Det
+ Det
0 2 ,
Detz
Det
+ 7 0 =
Det
1 0 ,
Detx
Det
Det
1 1 ,
Dety
Det
+ Det
1 2 ,
Detz
Det
+ 4 0 =
Det
2 0 ,
Detx
Det
Det
2 1 ,
Dety
Det
+ Det
2 2 ,
Detz
Det
+ 10 + 0 =
Salama Tamam !
Cramer Kural gzel bir yntemdir, Fakat taneden ok deikenli sistemler iin
mutlaka bilgisayar kullanmak gerekir. Bilgisayar kullanlacak olunca, en son inceleyeceimiz
matris inversiyon yntemi daha geneldir.
185
Arttrlm Matris Yntemi
Arttrlm Matris ynteminde sistem matrisi tm sistemi, sa taraf ile birlikte tek bir
matris halinde belirtilir.
Sistem
1
2
3
2
1
2
3
1
2
7
4
10
|

\
|
|
|

=
Sonra bu matris elemanter satr ilemleri ile,
zm
1
0
0
0
1
0
0
0
1
x
y
z
|

\
|
|
|
|

=
haline getirilir. son stundaki deerler deikenlerin zm deerleridir Bu zme,
Gauss-Jordan Eliminasyon yntemi denilir..
Yorum : Tanmlama kolay, uygulama zor. Bilgisayar olmadan bu ilere giriilmez.
Matris Inversiyon Yntemi
Matris Inversiyon yntemi, lineer denklem takmlarnn zm iin en genel uygulanan
yntemdir. Bu yntemde, iki tane matris kullanlr. Bunlardan birisi katsaylar matrisi, dieri de sa
taraf vektrdr.
KatsayMat
1
2
3
2
1
2
3
1
2
|

\
|
|
|

= SagTaraf
7
4
10
|

\
|
|
|

=
Katasylar matrisinin inversi alnr,
KatsayMat
1
0.267
0.067
0.467
0.133
0.467
0.267
0.333
0.333
0.333
|

\
|
|
|

=
186
Sa taraf vektr ile (tek stundan oluan matris) arplr. Sonu, sistem zmdr.
KatsayMat
1
SagTaraf
2
1
1
|

\
|
|
|

=
yani x = 2, y = -1 ve z = 1. Gzel, ama bu alma bilgisayar programlar
olmadan yaplmaz.
Non-Lineer Sistemler
Non- Lineer sistemlerde deikenlerden bazlar stnde stel deerler bulunabilir,
veya x y arpmlar bulunabilir. Bu sistemlerin zm iin, hibir ntanml yntem yoktur.
Yerine koyma yntemi ile deikenler teker zmlenmeye allr. Bazen zm ok kolay
olabilir.
Kaynaka
Bu kitap tamamen orijinaldir. Amac rencilere kendi kendilerine matematik
konularn gelitirme olana salamaktr. Prof. Dr. Bedri Doan Emir'in zgn retim yntem
ve konularn iermektedir.
Konular bir mfredat izleme amac ile seilmemitir. Konular temel bilgileri oluturma
amac ile dzenlenmitir. Temel bilgiler olutuktan sonra, renciler ders kitaplarndan
izleyerek her trl konuyu kendi kendilerine gelitirebilrler.
Bu kitapta birok Trk ve Amerikan niversitesinin ders kitaplarndan yararlanlmtr.
En ok yararlanlan http://tutorial.math.lamar.edu/Classes/Alg/Alg.aspx de Yaynlanm Paul
Dawkins Algebra ders notlardr.
Internetten ndirme Adresi :
http://www.bedriemir.com/math
letiim Adresi : bedri@bedriemir.com (ltfen anlamadnz konular sorunuz)
Tm Okuyuculara Candan Baarlar Dilerim :
Prof. Dr. Bedri Doan Emir
187

You might also like